Senior ACS Weekly

¡Supera tus tareas y exámenes ahora con Quizwiz!

A 43-year-old woman with a past medical history notable for hypertension, cholecystectomy, and appendectomy presents to the emergency department with right lower quadrant pain, fever, and leukocytosis. She is hemodynamically stable, and her abdomen is soft, with tenderness to palpation in the right lower quadrant. She is not diffusely tender. A urinalysis is notable only for ketones, and a pelvic sonogram demonstrates a normal uterus and ovaries. She undergoes a CT scan with oral and rectal contrast that demonstrates multiple diverticula throughout the colon, thickening of the cecum, and inflammation of the adjacent cecal fat. What is the next most appropriate step in the management of this patient? A) Admission to the hospital, NPO, intravenous fluids, and antibiotics B) Exploratory laparotomy C) Diagnostic laparoscopy D) Discharge to home with a referral to a gastroenterologist for outpatient follow-up E) Admission to the hospital for ileocecectomy

To demonstrate knowledge about cecal diverticulitis Answer A is correct In the United States, diverticulitis rarely involves the cecum or the right colon. Right-side diverticula occur in only 15% of patients in Western countries, compared with 75% in Singapore. The incidence of cecal diverticulitis appears to be related to the number of diverticula present. A classification system has been proposed that divides cecal diverticulitis into four grades to facilitate comparisons between different clinical series and to help surgeons formulate treatment plans in the operating room. Some cecal diverticula are true diverticula, containing all layers of the bowel wall, but the majority are pseudodiverticula. Diverticulitis of the hepatic flexure and the transverse colon is even less common and can present with symptoms suggesting appendicitis. Patients with right-side disease tend to be younger and to have less generalized peritonitis than patients with left-side diverticulitis. Because they typically present with right lower quadrant pain, fever, and leukocytosis, acute appendicitis is usually suspected. CT scans are helpful for differentiating cecal diverticulitis from appendicitis or colon cancer. If cecal diverticulitis is suspected (as in a patient who has previously undergone appendectomy or in a patient with known right-side diverticulosis who has experienced similar attacks in the past), medical management with observation and antibiotics is generally the favored strategy, just as with simple sigmoid diverticulitis. In Japan, where right-side diverticulitis is more common, medical treatment has been successfully used for recurrent attacks of uncomplicated right-side diverticulitis. After a few weeks, colonoscopy should be performed to rule out a colonic neoplasm.

A 71-year-old man presents to the emergency department with several days of left lower quadrant pain, fever to 38.5°C (101.4°F), and urinary urgency. On further questioning, he reports episodes of pneumaturia. A diagnosis of diverticulitis with colovesical fistula is suspected. Which of the following statements about colovesical fistula is not true? A) Women are less likely to experience colovesical fistula because the uterus protects the bladder B) Cystoscopy is required to confirm the diagnosis C) Possible CT findings include thickening of the bladder, air in the bladder, and sigmoid diverticula D) Resection of a colovesical fistula is usually undertaken given the associated risk of urinary sepsis E) None of the above

To demonstrate knowledge about the evaluation of colovesical fistulas resulting from diverticulitis Answer B is correct Some diverticular abscesses rupture into adjacent tissues or viscera, resulting in the formation of fistulas. The fistulas most commonly seen in this setting (50 to 65% of cases) are colovesical fistulas. This complication is less common in women because of the protection afforded by the uterus. Symptoms of colovesical fistulas tend to involve the urinary tract (e.g., pneumaturia, hematuria, and urinary frequency). When a colovesical fistula occurs, contrast CT with narrow cuts in the pelvis can be very helpful. The classic findings are sigmoid diverticula, thickening of the bladder and colon, air in the bladder, opacification of the fistula tract and bladder, and, possibly, an abscess. Cystoscopy is less specific, showing possible edema or erythema at the site of the fistula. Colovesical fistulas are usually resected because of the risk of urinary sepsis and the concern that a malignancy might be overlooked. Preferably, the operation is done when the acute inflammation has subsided.

A 28-year-old morbidly obese female presents to your office to discuss bariatric surgery options. Her BMI is 42 kg/m2. Which of the following is true regarding the various bariatric surgery procedures? A) Patients typically lose weight more quickly after a laparoscopic GBP than after a laparoscopic adjustable gastric band (LAGB) B) LAGB achieves weight loss through malabsorption of food C) Jejunoileal bypass and laparoscopic GBP have similar complication rates D) GBP and vertical banded gastroplasty (VBG) result in similar amounts of weight loss E) The reoperation rate after LAGB is significantly lower than after laparoscopic GBP

To demonstrate knowledge of the advantages and disadvantages of bariatric surgery procedures Answer A is correct LAGB achieves weight loss by being a restrictive, not a malabsorptive, procedure. Studies show that laparoscopic GBP patients lose weight more quickly than LAGB patients. For instance, in one study, patients who underwent laparoscopic GBP initially lost weight more rapidly: their mean percentage of excess body weight lost (%EBWL) was 65% during postoperative year 1 compared with 39% for LAGB patients. Thereafter, weight loss slowed, remaining nearly unchanged at 3 years (63%). Patients who underwent LAGB initially lost weight more slowly, but the ongoing weight loss was continuous, eventually approaching that of laparoscopic GBP. At 3 years, the %EBWL for LAGB patients was 55%. Complication rates and reoperation rates were similar between LAGB and laparoscopic GBP. In multiple studies, up to one third of LAGB patients need to have their bands removed, because of complications or inadequate weight loss. Randomized, prospective trials have conclusively shown that GBP is as effective for weight control as the malabsorptive jejunoileal bypass is, while resulting in significantly fewer complications. Jejunoileal bypass is associated with a substantial incidence of both early complications (e.g., acute cirrhosis, electrolyte imbalance, and fulminant diarrhea) and late complications (e.g., cirrhosis, interstitial nephritis, arthritis, enteritis, nephrocalcinosis, and recurrent oxalate renal stones). Multiple randomized, prospective trials have found VBG to be significantly less effective in achieving long-term weight loss than standard GBP. The failure rate is thought to be high after VBG because patients experience no symptoms (such as dumping syndrome) when eating candy or drinking nondietetic sodas.

A 28-year-old morbidly obese female presents to your office to discuss bariatric surgery options. Her body mass index (BMI) is 42 kg/m2. Which of the following is true regarding the various bariatric surgery procedures? A) Patients typically lose weight quicker after a laparoscopic GBP than after a laparoscopic adjustable gastric band (LAGB) B) LAGB achieves weight loss through malabsorption of food C) Jejunoileal bypass and laparoscopic GBP have similar complication rates D) GBP and vertical banded gastroplasty (VBG) result in similar amounts of weight loss E) The reoperation rate after LAGB is significantly lower than after laparoscopic GBP

To demonstrate knowledge of the advantages and disadvantages of bariatric surgery procedures Answer A is correct. LAGB achieves weight loss by being a restrictive, not a malabsorptive, procedure. Studies show that laparoscopic GBP patients lose weight more quickly than LAGB patients. For instance, in one study, patients who underwent laparoscopic GBP initially lost weight more rapidly: their mean percentage of excess body weight lost (%EBWL) was 65% during postoperative year 1, compared with 39% for LAGB patients. Thereafter, weight loss slowed, remaining nearly unchanged at 3 years (63%). Patients who underwent LAGB initially lost weight more slowly, but the ongoing weight loss was continuous, eventually approaching that of laparoscopic GBP. At 3 years, the %EBWL for LAGB patients was 55%. Complication rates and reoperation rates were similar between LAGB and laparoscopic GBP. In multiple studies, up to one third of LAGB patients need to have their bands removed, either because of complications or inadequate weight loss. Randomized, prospective trials have conclusively shown that GBP is as effective for weight control as the malabsorptive jejunoileal bypass is, although resulting in significantly fewer complications. Jejunoileal bypass is associated with a substantial incidence of both early complications (e.g., acute cirrhosis, electrolyte imbalance, and fulminant diarrhea) and late complications (e.g., cirrhosis, interstitial nephritis, arthritis, enteritis, nephrocalcinosis, and recurrent oxalate renal stones). Multiple randomized, prospective trials have found VBG to be significantly less effective in achieving long-term weight loss than standard GBP. The failure rate is thought to be high after VBG because patients experience no symptoms (such as dumping syndrome) when eating candy or drinking nondietetic sodas.

A 44-year-old male presents to your office with the complaint of a mass in his right thigh. On examination, you find a hard 8 cm mass in the medial compartment of the thigh that appears deep, nonmobile, and nontender. The most appropriate strategy for diagnosis is: A) Excisional biopsy B) Fine-needle aspiration C) Core-needle biopsy (CNB) D) Incisional biopsy E) Magnetic resonance imaging (MRI)

To demonstrate understanding for the diagnosis of soft tissue masses Answer C is correct Soft tissue sarcomas frequently present as large, painless masses. Approximately 38% are larger than 10 cm at presentation. The optimal imaging modality remains a matter of debate, but it is accepted that for extremity lesions, MRI provides more information than computed tomography (CT). Diagnosis is attained by pathology. A mass that is larger than 5 cm, is growing, or has persisted for longer than 4 weeks constitutes an indication for biopsy. In choosing the method, location, and orientation of the biopsy, it is critical to consider possible future surgical interventions. For masses larger than 5 cm, either an incisional biopsy or a CNB could be appropriate, although incisional biopsies increase the possibility of tumor spread through the tissue planes of the incision. CNB is therefore preferable for evaluation of soft tissue masses. Several studies have shown CNB to have very high sensitivity and specificity with respect to both malignant and benign soft tissue masses. For most masses smaller than 5 cm, an excisional biopsy with a clear surgical margin is indicated. Fine-needle aspiration is not a useful modality in the workup of soft tissue masses, because it does not provide a large enough sample to permit determination of histologic architecture and tumor grade.

An 88-year-old veteran is admitted to the emergency department with 2 days of abdominal pain and bloating. He has a history of an ulcer operation 50 years ago and an upper midline laparotomy scar. His abdomen is moderately distended and mildy tender, without any signs of peritonitis. He is hemodynamically normal, and his white blood cell count is 7,000. He undergoes a CT scan of the abdomen and pelvis that reveals evidence of a Billroth II reconstruction, enteric contrast within nondilated loops of small bowel and proximal colon, and an 8 cm dilated fluid-filled loop of intestine that appears proximal to the gastrojejunostomy. The next step in management should be: A) Discharge home with close follow-up B) Admission to the hospital, bowel rest, intravenous fluids, and serial abdominal examinations C) Admission to the hospital, bowel rest, nasogastric decompression, intravenous fluids, and serial abdominal examinations D) Admission to the hospital, bowel rest, nasogastric decompression, intravenous fluids, intravenous antibiotics, and serial abdominal examinations E) Laparotomy

To demonstrate understanding of afferent limb obstruction Answer E is correct The patient in this question has obstruction of the afferent limb of his Billroth II reconstruction. Afferent limb obstruction may occur as a consequence of adhesions, internal herniation, volvulus, or a kink at the angle formed with the gastric remnant. Obstruction to outflow of the afferent limb creates a closed-loop obstruction, with persistent secretion of bile and pancreatic fluids into the loop. Such obstruction often presents as recurrent pancreatitis. The diagnosis is facilitated by CT scanning; if it is confirmed, prompt exploration is mandated. Correction of the obstruction may necessitate conversion to a Roux-en-Y reconstruction, shortening of the afferent limb, or a side-to-side enteroenterostomy with the efferent limb.

An 88-year-old veteran is admitted to the emergency department with 2 days of abdominal pain and bloating. He has a history of an ulcer operation 50 years ago and an upper midline laparotomy scar. His abdomen is moderately distended and mildy tender without any signs of peritonitis. He is hemodynamically normal, and his white blood cell count is normal He undergoes CT of the abdomen and pelvis that reveals evidence of a Billroth II reconstruction, enteric contrast within nondilated loops of small bowel and proximal colon, and an 8 cm dilated fluid-filled loop of intestine that appears proximal to the gastrojejunostomy. The next step in management should be: A) Discharge home with close follow-up B) Admission to the hospital, bowel rest, intravenous fluids, and serial abdominal examinations C) Admission to the hospital, bowel rest, nasogastric decompression, intravenous fluids, and serial abdominal examinations D) Admission to the hospital, bowel rest, nasogastric decompression, intravenous fluids, intravenous antibiotics, and serial abdominal examinations E) Laparotomy

To demonstrate understanding of afferent limb obstruction Answer E is correct The patient in this question has obstruction of the afferent limb of his Billroth II reconstruction. Afferent limb obstruction may occur as a consequence of adhesions, internal herniation, volvulus, or a kink at the angle formed with the gastric remnant. Obstruction to outflow of the afferent limb creates a closed-loop obstruction, with persistent secretion of bile and pancreatic fluids into the loop. Such obstruction often presents as recurrent pancreatitis. The diagnosis is facilitated by CT; if it is confirmed, prompt exploration is mandated. Correction of the obstruction may necessitate conversion to a Roux-en-Y reconstruction, shortening of the afferent limb, or a side-to-side enteroenterostomy with the efferent limb.

A 33-year-old female is concerned about a small "growth" on her left forearm. On examination, you find a 6 mm bluish-black flat lesion with irregular borders. The appropriate next step in management is: A) Close observation B) Incisional biopsy with depth of excision reaching the subcutaneous fat layer C) Incisional biopsy with depth of excision reaching the dermal layer D) Excisional biopsy with depth of excision reaching the subcutaneous fat layer E) Excisional biopsy with depth of excision reaching the dermal layer

To demonstrate understanding of the appropriate technique for biopsy of skin lesions Answer D is correct Any suspicious skin lesion should be biopsied. This can be performed by excisional biopsy if the lesion is small or incisional biopsy if the lesion is large. Excisional biopsy should be performed by incorporating a 1 to 4 mm margin of normal skin surrounding the lesion depending on the clinical characteristics. This may eliminate the need for subsequent reexcision for some types of lesions (dysplastic nevi). However, no attempt should be made to perform a definitive radical excision until a diagnosis is established by biopsy. Full-thickness excision into the subcutaneous fat should be performed, and the margins should be marked for orientation.

A 56-year-old gentleman is referred to you with the diagnosis of gastric carcinoid tumors. In the distal stomach, he was found to have two 2 cm submucosal masses and one 1 cm mass that were biopsied but incompletely resected by endoscopy. The pathology was consistent with carcinoid, and his serum gastrin level is mildly elevated. Which of the following statements is true? A) He has type I gastric carcinoids and should be treated with endosopic polypectomy B) He has type I gastric carcinoids and should be treated with antrectomy C) He has type II gastric carcinoids and should be treated with endoscopic polypectomy D) He has type II gastric carcinoids and should be treated with antrectomy E) He has type III gastric carcinoids and should be treated with distal gastrectomy

To demonstrate understanding of the classification and treatment of gastric carcinoids Answer B is correct Gastric carcinoid tumors are rare, accounting for fewer than 11 to 30% of all GI carcinoids and fewer than 1% of all gastric tumors. The median age at diagnosis is 62, and tumors are equally distributed between men and women. Gastric carcinoid tumors are often discovered during endoscopic examination of patients experiencing chronic abdominal pain; patients may also complain of vomiting and diarrhea. These tumors are rarely associated with symptoms of the carcinoid syndrome. Diagnosis is usually confirmed by endoscopic biopsy, and endoscopic ultrasonography is helpful in determining the extent of gastric wall penetration and the degree of regional lymph node involvement. Gastric carcinoid tumors have been divided into three types, primarily on the basis of their association (or lack thereof) with hypergastrinemia. Type I tumors are associated with chronic atrophic gastritis, are generally small (< 1 cm), and are often multiple and polypoid. They grow slowly and only rarely metastasize to regional nodes or distant sites. Type II tumors are associated with the Zollinger-Ellison syndrome and multiple endocrine neoplasia type I (MEN type I) and, like type I tumors, are usually small and multiple. They also grow slowly, but they are more likely to metastasize than type I gastric carcinoids are. Type III (sporadic) gastric carcinoid tumors are the most biologically aggressive type. They are often large (> 1 cm) at the time of diagnosis and are not associated with hypergastrinemia. Type III lesions frequently metastasize to regional nodes (54%) or the liver (24%). For patients with small, solitary type I tumors, endoscopic polypectomy or open resection via gastrotomy (local excision) is the procedure of choice. For patients with multiple or recurrent tumors, antrectomy is indicated to remove the source of the hypergastrinemia. For patients with type II lesions, treatment is similar to that for patients with type I lesions, with the extent of gastric resection determined by the size and number of lesions. For patients with type III lesions, however, either distal or total gastrectomy with extended lymph node dissection is required. All patients undergoing a less than total gastrectomy should be followed with serial endoscopy at regular intervals.

A 65-year-old female has been suffering from intermittent abdominal pain for several weeks. On endoscopy, a 3 cm prepyloric ulcer is noted, and she is determined to be infected with H. pylori. She is treated successfully to eradicate the H. pylori and is maintained on a proton pump inhibitor. A surveillance endoscopy 3 months later reveals a 2 cm prepyloric ulcer. What is the next appropriate step in management? A) Increase the dose of the proton pump inhibitor B) Add an H2 blocker C) Acid-reducing procedure drainage D) Biopsy of the ulcer E) Repeat endoscopy in 3 months

To demonstrate understanding of the evaluation of gastric ulcers Answer D is correct Occasionally, patients present for surgery with refractory gastric ulcers. If the ulcer has not healed after 12 weeks of optimal medical therapy, resection is indicated to rule out an occult gastric malignancy. In such cases, the preoperative workup should include endoscopic biopsies of the ulcer base and the surrounding gastric mucosa so that a preoperative diagnosis of malignancy can be made if possible. In view of the concern about a possible gastric malignancy, it is reasonable to obtain a preoperative chest x-ray and a CT scan of the abdomen so as to detect possible nodal or distant metastases.

A 65-year-old female has been suffering from intermittent abdominal pain for several weeks. On endoscopy, a 3 cm prepyloric ulcer is noted, and she is determined to be infected with H. pylori. She is treated successfully to eradicate the H. pylori and is maintained on a proton pump inhibitor. A surveillance endoscopy 3 months later reveals a 2 cm prepyloric ulcer. What is the next appropriate step in management? A) Increase the dose of the proton pump inhibitors B) Add an H2 blocker C) Acid-reducing procedure D) Biopsy of the ulcer E) Repeat endoscopy in 3 months

To demonstrate understanding of the evaluation of gastric ulcers Answer D is correct Occasionally, patients present for surgery with refractory gastric ulcers. If the ulcer has not healed after 12 weeks of optimal medical therapy, resection is indicated to rule out an occult gastric malignancy. In such cases, the preoperative workup should include endoscopic biopsies of the ulcer base and the surrounding gastric mucosa so that a preoperative diagnosis of malignancy can be made if possible. In view of the concern about a possible gastric malignancy, it is reasonable to obtain a preoperative chest x-ray and a CT scan of the abdomen so as to detect possible nodal or distant metastases.

A 57-year-old man is noted to be anemic, and upper endoscopy reveals a gastric mass. Biopsy is consistent with adenocarcinoma, and resection is planned. Which of the following is true regarding surgical management of gastric adenocarcinoma? A) A D1 lymphadenectomy involves removal of perigastric, splenic, and celiac lymph nodes B) Gastric adenocarcinoma should be resected with a 2 cm gross margin C) European studies have shown a survival benefit for D2 lymphadenectomy compared with D1 lymphadenectomy in all patient groups D) Total gastrectomy may be necessary for proximal lesser curve adenocarcinomas E) The total number of lymph nodes removed has no effect on survival

To demonstrate understanding of the extent of gastric and lymph node resection for gastric adenocarcinoma Answer D is correct Gastric adenocarcinoma can spread microscopically beyond the gross extent of the tumor. As a result, a 5 cm gross margin is recommended for gastric resections for adenocarcinoma. Although lesions on the lesser curvature may appear to be distal, a 5 cm proximal margin can be near the gastroesophageal junction, necessitating a total gastrectomy. Gastric lymphadenectomy is classified as D0 (incomplete resection of perigastric nodes), D1 (perigasric node resection), and D2 (resection of splenic, left gastric, and celiac axis nodes). Whereas Japanese studies have found benefit from D2 resection, multiple Western studies have shown no survival benefit for D2 resections compared with D1 resections. The Dutch Gastric Cancer Group did find that D2 lymphadenectomy was beneficial 10 years after operation in the subgroup of patients who had positive second-order lymph nodes, but this has not been shown for all patient groups. Another study did find that the number of lymph nodes removed does impact survival; patients with more than 15 lymph nodes resected had significantly higher survival rates.

A 57-year-old man is noted to be anemic, and upper endoscopy reveals a gastric mass. Biopsy is consistent with adenocarcinoma, and resection is planned. Which of the following is true regarding surgical management of gastric adenocarcinoma? A) A D1 lymphadenectomy involves removal of perigastric, splenic, and celiac lymph nodes B) Gastric adenocarcinoma should be resected with a 2 cm gross margin C) European studies have shown a survival benefit for D2 lymphadenectomy compared with D1 lymphadenectomy in all patient groups D) Total gastrectomy may be necessary for proximal lesser curve adenocarcinomas E) The total number of lymph nodes removed has no effect on survival

To demonstrate understanding of the extent of gastric and lymph node resection for gastric adenocarcinoma Answer D is correct Gastric adenocarcinoma can spread microscopically beyond the gross extent of the tumor. As a result, a 5 cm gross margin is recommended for gastric resections for adenocarcinoma. Although lesions on the lesser curvature may appear to be distal, a 5 cm proximal margin can be near the gastroesophageal junction, necessitating a total gastrectomy. Gastric lymphadenectomy is classified as D0 (incomplete resection of perigastric nodes), D1 (perigasric node resection), and D2 (resection of splenic, left gastric, and celiac axis nodes). Whereas Japanese authors have found benefit from D2 resection, multiple Western studies have shown no survival benefit for D2 resections compared with D1 resections. The Dutch Gastric Cancer Group did find that D2 lymphadenectomy was beneficial 10 years after operation in the subgroup of patients who had positive second-order lymph nodes, but this has not been shown for all patient groups. Another study did find that the number of lymph nodes removed does impact survival; patients with greater than 15 lymph nodes resected had significanly higher survival rates.

A 61-year-old male complains of difficulty initiating swallowing and dysphagia and has a history of recurrent aspiration pneumonia. An EGD reveals a 3 cm pocket of undigested food material at approximately 10 cm from the incisors. Which of the following statements regarding this condition is not true? A) Cine contrast studies may reveal poor pharyngeal contractility B) In symptomatic patients, surgical therapy is indicated C) Surgical therapy involves a myotomy of the posterior pharyngeal constrictors D) This patient is at risk for aspiration on induction of anesthesia E) An esophageal motility study is not required in the preoperative workup

To demonstrate understanding of the management of a Zenker diverticulum . Answer C is correct A Zenker diverticulum is a pulsion diverticulum that arises adjacent to the inferior pharyngeal constrictor, between the oblique fibers of the posterior pharyngeal constrictors and the cricopharyngeus muscle. This mucosal outpouching results from a transient incomplete opening of the upper esophageal sphincter. The diverticulum ultimately enlarges, drapes over the cricopharyngeus, and dissects behind the esophagus into the prevertebral space. The pouch usually deviates to one side or the other; accordingly, the side on which the deviation occurs must be determined by means of a barium swallow so that the appropriate operative approach can be selected. Patients who are candidates for cricopharyngeal myotomy usually present with difficulty initiating swallowing, cervical dysphagia, and a history of pulmonary aspiration. These symptoms of cricopharyngeal dysfunction may or may not be associated with a Zenker diverticulum. Cine contrast studies may reveal poor pharyngeal contractility, pulmonary or nasal aspiration, abnormalities of the upper esophageal sphincter, pharyngeal pouches, or other structural abnormalities in the distal esophagus. Barium is the usual contrast agent as it is inert if aspirated. Esophageal motility studies (not usually performed) may show either incomplete upper esophageal relaxation on swallowing or poor coordination of the upper esophageal relaxation phase with pharyngeal contractions. Upper GI endoscopy is performed preoperatively to exclude the presence of a pharyngeal or esophageal carcinoma and to assess the upper GI anatomy. If there is evidence of GERD, proton pump inhibitors are given. In symptomatic patients (e.g., those with dysphagia, nocturnal cough, or recurrent pneumonia from aspiration), surgical therapy is indicated regardless of whether a pouch is present or how large it may be. Such treatment involves correcting the underlying cricopharyngeal muscle dysfunction with a cricopharyngeal myotomy. If there is a diverticulum larger than 2 cm, it should be excised in addition to the cricopharyngeal myotomy. Alternatively, the diverticulum may be managed via endoscopic obliteration of the common wall between the pharyngeal pouch and the esophagus with either a stapler or a laser. Cricopharyngeal incoordination may be temporarily relieved by injecting botulinum toxin into the cricopharyngeus.

A 61-year-old male complains of difficulty initiating swallowing and dysphagia and has a history of recurrent aspiration pneumonia. An EGD reveals a 3 cm pocket of undigested food material at approximately 10 cm from the incisors. Which of the following statements regarding this condition is not true? A) Cine contrast studies may reveal poor pharyngeal contractility B) In symptomatic patients, surgical therapy is indicated C) Surgical therapy involves a myotomy of the posterior pharyngeal constrictors D) This patient is at risk for aspiration on induction of anesthesia E) An esophageal motility study is not required in the preoperative workup

To demonstrate understanding of the management of a Zenker diverticulum Answer C is correct A Zenker diverticulum is a pulsion diverticulum that arises adjacent to the inferior pharyngeal constrictor, between the oblique fibers of the posterior pharyngeal constrictors and the cricopharyngeus muscle. This mucosal outpouching results from a transient incomplete opening of the upper esophageal sphincter. The diverticulum ultimately enlarges, drapes over the cricopharyngeus, and dissects behind the esophagus into the prevertebral space. The pouch usually deviates to one side or the other; accordingly, the side on which the deviation occurs must be determined by means of a barium swallow so that the appropriate operative approach can be selected. Patients who are candidates for cricopharyngeal myotomy usually present with difficulty initiating swallowing, cervical dysphagia, and a history of pulmonary aspiration. These symptoms of cricopharyngeal dysfunction may or may not be associated with a Zenker diverticulum. Cine contrast studies may reveal poor pharyngeal contractility, pulmonary or nasal aspiration, abnormalities of the upper esophageal sphincter, pharyngeal pouches, or other structural abnormalities in the distal esophagus. Barium is the usual contrast agent as it is inert if aspirated. Esophageal motility studies (not usually performed) may show either incomplete upper esophageal relaxation on swallowing or poor coordination of the upper esophageal relaxation phase with pharyngeal contractions. Upper GI endoscopy is performed preoperatively to exclude the presence of a pharyngeal or esophageal carcinoma and to assess the upper GI anatomy. If there is evidence of GERD, proton pump inhibitors are given. In symptomatic patients (e.g., those with dysphagia, nocturnal cough, or recurrent pneumonia from aspiration), surgical therapy is indicated regardless of whether a pouch is present or how large it may be. Such treatment involves correcting the underlying cricopharyngeal muscle dysfunction with a cricopharyngeal myotomy. If there is a diverticulum larger than 2 cm, it should be excised in addition to the cricopharyngeal myotomy. Alternatively, the diverticulum may be managed via endoscopic obliteration of the common wall between the pharyngeal pouch and the esophagus with either a stapler or a laser. Cricopharyngeal incoordination may be temporarily relieved by injecting botulinum toxin into the cricopharyngeus.

A 45-year-old man is undergoing excision of a melanoma lesion of the lower extremity, as well as an SLNB. Frozen section of the lymph node returns positive for melanoma. Which of the following treatment statements is true? A) SLNB has clearly been shown to demonstrate an overall survival benefit in patients with melanoma B) The evidence for completion lymphadenectomy in a patient with a positive SLNB clearly supports a therapeutic benefit C) There is a role for therapeutic lymph node dissection in patients with bulky lymphadenopathy D) Patients with in-transit lymph node metastases have a favorable prognosis E) Adjuvant therapy with interferon alfa is not associated with improvement in disease-free survival

To demonstrate understanding of the management of lymph node metastases in melanoma Answer C is correct Therapeutic lymph node dissection is currently recommended for management of the regional lymph node drainage basin in the presence of a positive sentinel node and in clinical stage III disease. Some challenge this recommendation as four randomized trials of elective lymph node dissection failed to demonstrate an overall survival benefit. Yet the majority of patients in these studies did not have nodal disease; thus, the trials were not sufficiently powered to detect a small survival benefit. Other trials have supported this practice, including the World Health Organization trial, which on subgroup analysis demonstrated a significantly improved 5-year survival rate in patients with occult nodal metastases detected at elective lymph node dissection compared with patients who had delayed lymphadenectomy at the time when they developed palpable nodal metastases (48% versus 27%, respectively; p = .04). Nonetheless, the impact of completion lymphadenectomy on overall survival is still a matter of debate. The results of the first Multicenter Selective Lymphadenectomy Trial (MSLT-I) suggest that SLNB with immediate completion lymphadenectomy if the sentinel node is positive improves disease-free survival but not overall survival for these patients. Although there were suggestions based on the data that a survival benefit might be gained by lymphatic mapping and SLNB followed by completion lymph node dissection in the event of a positive node, their statistical analysis has been criticized. There is, however, clearly a role for therapeutic lymph node dissection in patients with bulky nodal disease, and it can be performed with minimal morbidity and good palliation. Patients with in-transit metastases have an unfavorable prognosis. This reflects a disseminated stage of disease, with a 5-year survival rate of 25 to 30%. Surgical excision is the mainstay of therapy when the size and the number of lesions permit, although there is no formal recommendation regarding the extent of surgical margin. Adjuvant therapy with interferon alfa-2b is associated with improvement in disease-free survival but is also associated with significant toxicity.

A 36-year-old female underwent a laparoscopic "stomach wrap" performed for reflux 5 years ago. She cannot provide any more detail regarding the nature of her operation. Over the past several months, she has noticed a severe recurrence of her symptoms, which include abdominal pain and reflux. Which of the following statements is not true? A) Fundoplication is currently considered the procedure of choice for patients with GERD, regardless of the strength of their esophageal peristalsis B) Partial fundoplication is not as durable as total (360°) fundoplication C) Total fundoplication is associated with a higher incidence of postoperative dysphagia, especially in patients with weak peristalsis D) The majority of patients that undergo fundoplication will not require antireflux medication 10 years after the operation E) The Nissen fundoplication is a 360° wrap

To demonstrate understanding of the outcomes of laparoscopic fundoplication Answer C is correct The initial results of laparoscopic fundoplication obtained in the early 1990s indicated that the operation was effective in controlling reflux but that postoperative dysphagia occurred more often than had been anticipated. Many experts thought that this problem could be avoided by tailoring the fundoplication to the strength of esophageal peristalsis as measured by esophageal manometry. Accordingly, partial fundoplication (240°) was recommended for patients with impaired peristalsis, and total fundoplication (360°) was recommended for those with normal peristalsis. The short-term results of this tailored approach were promising. Gradually, however, it became evident that a partial fundoplication was not as durable as a total fundoplication and that a total fundoplication was not associated with a higher incidence of postoperative dysphagia even in patients with weak peristalsis. These findings suggest that the initial problems with postoperative dysphagia were primarily attributable to unknown technical factors that were largely eliminated from the procedure as surgeons garnered more experience with it. As a result, total fundoplication is currently considered the procedure of choice for patients with GERD, regardless of the strength of their esophageal peristalsis. A 2006 study aimed at critically assessing 10-year outcomes reported results from 100 consecutive patients after complete and partial fundoplication. In this series, the rate of symptomatic control of reflux symptoms at 5 and 10 years was 90%, with fewer than 10% of patients using antacid medications at 10 years; only one patient required reintervention for persistent dysphagia. Similar results were documented in subsequent studies, confirming laparoscopic Nissen fundoplication as an effective long-term treatment for GERD.

A 36-year-old female underwent a laparoscopic "stomach wrap" performed for reflux 5 years ago. She cannot provide any more details regarding the nature of her operation. Over the past several months, she has noticed a severe recurrence of her symptoms, which include abdominal pain and reflux. Which of the following statements is not true? A) Fundoplication is currently considered the procedure of choice for patients with GERD, regardless of the strength of their esophageal peristalsis B) Partial fundoplication is not as durable as total (360°) fundoplication C) Total fundoplication is associated with a higher incidence of postoperative dysphagia, even in patients with weak peristalsis D) The majority of patients who undergo fundoplication will not require antireflux medication 10 years after the operation E) The Nissen fundoplication is a 360° wrap

To demonstrate understanding of the outcomes of laparoscopic fundoplication Answer C is correct The initial results of laparoscopic fundoplication obtained in the early 1990s indicated that the operation was effective in controlling reflux but that postoperative dysphagia occurred more often than had been anticipated. Many experts thought that this problem could be avoided by tailoring the fundoplication to the strength of esophageal peristalsis as measured by esophageal manometry. Accordingly, partial fundoplication (240°) was recommended for patients with impaired peristalsis, and total fundoplication (360°) was recommended for those with normal peristalsis. The short-term results of this tailored approach were promising. Gradually, however, it became evident that a partial fundoplication was not as durable as a total fundoplication and that a total fundoplication was not associated with a higher incidence of postoperative dysphagia even in patients with weak peristalsis. These findings suggest that the initial problems with postoperative dysphagia were primarily attributable to unknown technical factors that were largely eliminated from the procedure as surgeons garnered more experience with it. As a result, total fundoplication is currently considered the procedure of choice for patients with GERD, regardless of the strength of their esophageal peristalsis. A 2006 study aimed at critically assessing 10-year outcomes reported results from 100 consecutive patients after complete and partial fundoplication. In this series, the rate of symptomatic control of reflux symptoms at 5 and 10 years was 90%, with fewer than 10% of patients using antiacid medications at 10 years; only one patient required reintervention for persistent dysphagia. Similar results were documented in subsequent studies, confirming laparoscopic Nissen fundoplication as an effective long-term treatment for GERD.

A 36-year-old female underwent a laparoscopic "stomach wrap" performed for reflux 5 years ago. She cannot provide any more details regarding the nature of her operation. Over the past several months, she has noticed a severe recurrence of her symptoms, which include abdominal pain and reflux. Which of the following statements is not true? A) Fundoplication is currently considered the procedure of choice for patients with GERD, regardless of the strength of their esophageal peristalsis B) Partial fundoplication is not as durable as total (360°) fundoplication C) Total fundoplication is associated with a higher incidence of postoperative dysphagia, even in patients with weak peristalsis D) The majority of patients who undergo fundoplication will not require antireflux medication 10 years after the operation E) The Nissen fundoplication is a 360° wrap

To demonstrate understanding of the outcomes of laparoscopic fundoplication Answer C is correct The initial results of laparoscopic fundoplication obtained in the early 1990s indicated that the operation was effective in controlling reflux but that postoperative dysphagia occurred more often than had been anticipated. Many experts thought that this problem could be avoided by tailoring the fundoplication to the strength of esophageal peristalsis as measured by esophageal manometry. Accordingly, partial fundoplication (240°) was recommended for patients with impaired peristalsis, and total fundoplication (360°) was recommended for those with normal peristalsis. The short-term results of this tailored approach were promising. Gradually, however, it became evident that a partial fundoplication was not as durable as a total fundoplication and that a total fundoplication was not associated with a higher incidence of postoperative dysphagia even in patients with weak peristalsis. These findings suggest that the initial problems with postoperative dysphagia were primarily attributable to unknown technical factors that were largely eliminated from the procedure as surgeons garnered more experience with it. As a result, total fundoplication is currently considered the procedure of choice for patients with GERD, regardless of the strength of their esophageal peristalsis. A 2006 study aimed at critically assessing 10-year outcomes reported results from 100 consecutive patients after complete and partial fundoplication. In this series, the rate of symptomatic control of reflux symptoms at 5 and 10 years was 90%, with fewer than 10% of patients using antiacid medications at 10 years; only one patient required reintervention for persistent dysphagia. Similar results were documented in subsequent studies, confirming laparoscopic Nissen fundoplication as an effective long-term treatment for GERD.

You determine that the patient in question 3 had undergone a laparoscopic Nissen procedure 5 years ago. You perform an esophagogastroduodenoscopy (EGD) that is normal and 24-hour pH monitoring that does not reveal significantly abnormal pH in the esophagus. What test will most accurately confirm the diagnosis of recurrent reflux? A) Upper GI series B) Multichannel intraluminal impedance and pH testing C) Symptomatic evaluation D) Laparoscopy E) CT

To demonstrate understanding of the physiology of reflux disease and impedance testing Answer B is correct Combined multichannel intraluminal impedance and pH testing has the ability to detect episodes of reflux, regardless of the pH of the refluxate, by identifying changes induced by the presence of a bolus in the esophagus; the episodes are then simply classified as acid or nonacid on the basis of concomitantly recorded pH values. Studies of healthy persons have demonstrated that multichannel intraluminal impedance and pH testing possesses increased sensitivity and specificity in detecting and characterizing gastroesophageal reflux, and the results have been shown to be highly reproducible. Although this technology is still not widely available, it has already been demonstrated to be useful in the workup of patients with GERD refractory to proton pump inhibitors and patients with respiratory symptoms of unknown origin.

A 45-year-old lawyer is referred to you with persistent reflux symptoms. She has failed medical management and is interested in discussing surgical options. In the preoperative workup for gastroesophageal reflux disease (GERD), what is the most accurate test in establishing the diagnosis? A) An upper gastrointestinal (GI) series B) Endoscopy C) Esophageal manometry D) Ambulatory pH monitoring E) Computed tomographic (CT) scan

To demonstrate understanding of the preoperative workup of GERD Answer D is correct All patients who are candidates for a laparoscopic fundoplication should undergo a preoperative evaluation that includes the following: (1) symptomatic evaluation, (2) an upper GI series, (3) endoscopy, (4) esophageal manometry, and (5) ambulatory pH monitoring. Ambulatory pH monitoring is the most reliable test for the diagnosis of GERD, with a sensitivity and specificity of about 92%. It is of key importance in the workup for the following reasons. It determines whether abnormal reflux is present. It establishes a temporal correlation between symptoms and episodes of reflux. Such a correlation is particularly important when atypical GERD symptoms (e.g., cough and chest pain) are present because 50% of these patients experience no heartburn and 50% do not have esophagitis on endoscopy. It allows staging on the basis of disease severity. Specifically, esophageal manometry and pH monitoring identify a subgroup of patients characterized by worse esophageal motor function (manifested by a defective lower esophageal sphincter or by abnormal esophageal peristalsis), more acid reflux in the distal and proximal esophagus, and slower acid clearance. These patients more frequently have Barrett metaplasia and experience respiratory symptoms; thus, they might benefit from early antireflux surgery. It provides baseline data that may prove useful postoperatively if symptoms do not respond to the procedure.

A 45-year-old lawyer is referred to you with persistent reflux symptoms. She has failed medical management and is interested in discussing surgical options. In the preoperative workup for gastroesophageal reflux disease (GERD), what is the most accurate test in establishing the diagnosis? A) An upper GI series B) Endoscopy C) Esophageal manometry D) Ambulatory pH monitoring E) CT scan

To demonstrate understanding of the preoperative workup of GERD Answer D is correct All patients who are candidates for a laparoscopic fundoplication should undergo a preoperative evaluation that includes the following: (1) symptomatic evaluation, (2) an upper GI series, (3) endoscopy, (4) esophageal manometry, and (5) ambulatory pH monitoring. Ambulatory pH monitoring is the most reliable test for the diagnosis of GERD, with a sensitivity and specificity of about 92%. It is of key importance in the workup for the following reasons. It determines whether abnormal reflux is present. It establishes a temporal correlation between symptoms and episodes of reflux. Such a correlation is particularly important when atypical GERD symptoms (e.g., cough and chest pain) are present because 50% of these patients experience no heartburn and 50% do not have esophagitis on endoscopy. It allows staging on the basis of disease severity. Specifically, esophageal manometry and pH monitoring identify a subgroup of patients characterized by worse esophageal motor function (manifested by a defective lower esophageal sphincter or by abnormal esophageal peristalsis), more acid reflux in the distal and proximal esophagus, and slower acid clearance. These patients more frequently have Barrett metaplasia and experience respiratory symptoms; thus, they might benefit from early antireflux surgery. It provides baseline data that may prove useful postoperatively if symptoms do not respond to the procedure.

A 45-year-old lawyer is referred to you with persistent reflux symptoms. She has failed medical management and is interested in discussing surgical options. In the preoperative workup for gastroesophageal reflux disease (GERD), what is the most accurate test in establishing the diagnosis? A) An upper gastrointestinal (GI) series B) Endoscopy C) Esophageal manometry D) Ambulatory pH monitoring E) Computed tomographic (CT) scan

To demonstrate understanding of the preoperative workup of GERD Answer D is correct. All patients who are candidates for a laparoscopic fundoplication should undergo a preoperative evaluation that includes the following: (1) symptomatic evaluation, (2) an upper GI series, (3) endoscopy, (4) esophageal manometry, and (5) ambulatory pH monitoring. Ambulatory pH monitoring is the most reliable test for the diagnosis of GERD, with a sensitivity and specificity of about 92%. It is of key importance in the workup for the following reasons. It determines whether abnormal reflux is present. It establishes a temporal correlation between symptoms and episodes of reflux. Such a correlation is particularly important when atypical GERD symptoms (e.g., cough and chest pain) are present because 50% of these patients experience no heartburn and 50% do not have esophagitis on endoscopy. It allows staging on the basis of disease severity. Specifically, esophageal manometry and pH monitoring identify a subgroup of patients characterized by worse esophageal motor function (manifested by a defective lower esophageal sphincter or by abnormal esophageal peristalsis), more acid reflux in the distal and proximal esophagus, and slower acid clearance. These patients more frequently have Barrett metaplasia and experience respiratory symptoms; thus, they might benefit from early antireflux surgery. It provides baseline data that may prove useful postoperatively if symptoms do not respond to the procedure.

Which of the following statements regarding gastric adenocarcinoma is not true? Please choose the single most appropriate answer to the question. A) In the United States, most cases of gastric cancer are identified at an advanced stage B) The 5-year survival of all cases of gastric cancer in the United States is less than 25% C) Chronic infection with Helicobacter pylori is the dominant risk factor for gastric adenocarcinoma D) An R0 resection for gastric adenocarcinoma is required for curative intent E) Given the propensity of submucosal spreading, a 3 cm margin is necessary during resection

To demonstrate understanding of the prognosis, risk factors, and surgical treatment of gastric adenocarcinoma Answer E is correct In the United States in particular, the incidence of stomach cancer has fallen substantially over the past 70 years. Whereas this disease was once a leading cause of cancer-related death in the United States, it now ranks 13th among major causes. Across all races, the 5-year relative survival was 23% for the period extending from 1992 to 1999. This result is probably related to the advanced stage at which most patients present. It has generally been assumed that adenocarcinoma of the distal stomach progresses from chronic gastritis to metaplasia through the teratogenic influence of environmental factors. The most commonly studied environmental factors are the nitrates and nitrose compounds present in high levels in salted, smoked, or pickled foods consumed in areas where gastric cancer is endemic. Finally, the emergence of chronic infection with H. pylori as the dominant risk factor for gastric adenocarcinoma has challenged the paradigm of the atrophic gastritis-intestinal metaplasia-gastric cancer sequence. R0 resection (i.e., resection of all gross disease with microscopically negative margins) has been shown to have a clear impact on overall survival after potentially curative surgery. In the German Gastric Cancer Study, a prospective multicenter observational trial, the calculated 10-year survival rate in the entire population was 26.3%, compared with 36.1% in patients who underwent an R0 resection. Given the propensity of tumor for submucosal spreading, many authors consider proximal margins of 5 to 6 cm, with routine frozen-section analysis, to be optimal.

Which of the following statements regarding gastric adenocarcinoma is not true? A) In the United States, most cases of gastric cancer are identified at an advanced stage B) The 5-year survival of all cases of gastric cancer in the United States is less than 25% C) Chronic infection with H. pylori is the dominant risk factor for gastric adenocarcinoma D) An R0 resection for gastric adenocarcinoma is required for curative intent E) Given the propensity of submucosal spreading, a 3 cm margin is necessary during resection

To demonstrate understanding of the prognosis, risk factors, and surgical treatment of gastric adenocarcinoma Answer E is correct. In the United States in particular, the incidence of stomach cancer has fallen substantially over the past 70 years. Whereas this disease was once a leading cause of cancer-related death in the United States, it now ranks 13th among major causes. Across all races, the 5-year relative survival was 23% for the period extending from 1992 to 1999. This result is probably related to the advanced stage at which most patients present. It has generally been assumed that adenocarcinoma of the distal stomach progresses from chronic gastritis to metaplasia through the teratogenic influence of environmental factors. The most commonly studied environmental factors are the nitrates and nitrose compounds present in high levels in salted, smoked, or pickled foods consumed in areas where gastric cancer is endemic. Finally, the emergence of chronic infection with H. pylori as the dominant risk factor for gastric adenocarcinoma has challenged the paradigm of the atrophic gastritis-intestinal metaplasia-gastric cancer sequence. R0 resection (i.e., resection of all gross disease with microscopically negative margins) has been shown to have a clear impact on overall survival after potentially curative surgery. In the German Gastric Cancer Study, a prospective multicenter observational trial, the calculated 10-year survival rate in the entire population was 26.3%, compared with 36.1% in patients who underwent an R0 resection. Given the propensity of tumor for submucosal spreading, many authors consider proximal margins of 5 to 6 cm, with routine frozen-section analysis, to be optimal

The patient from question 1 returns to the office to discuss treatment options. On detailed physical examination, no other suspicious lesions are found, and no palpable lymphadenopathy is present. An appropriate treatment plan includes: A) Surgical excision with 1 mm margins and no sentinel lymph node biopsy (SLNB) B) Surgical excision with 1 mm margins and SLNB C) Surgical excision with 2 mm margins and no SLNB D) Surgical excision with 2 mm margins and SLNB E) Surgical excision with 4 mm margins and SLNB

To demonstrate understanding of the surgical treatment of melanoma . Answer D is correct Surgical excision remains the mainstay of treatment for melanoma. The width of the recommended surgical margins depends on the thickness of the lesion and has been well defined by a series of prospective randomized clinical trials. The most recent recommendations suggest that a 0.5 cm margin is adequate for in situ melanoma, whereas margins of 1 cm are suggested for melanomas less than 1.0 mm in thickness. Two-centimeter margins should be obtained for lesions measuring 1 to 2 mm in depth. Although a reduction in surgical margin width to 1 cm for melanomas 1 to 2 mm in thickness may slightly increase the risk for local recurrence, it would not reduce overall survival statistics. Two-centimeter margins should be obtained for melanomas greater than 2 mm in thickness. Sentinel lymph node status is the single most important predictor of survival in patients with melanoma and is considered a standard approach in this country. SLNB is recommended for intermediate thickness (1 to 4 mm) and thick (> 4 mm) melanomas. It is generally not recommended for thin melanomas (< 1 mm) unless the lesion is of high risk (presence of ulceration, extensive regression, high mitotic rate, Clark level IV, or positive deep margin) or depending on the patient's age (SLNB may be performed in young patients).

A relatively fit 74-year-old man undergoes esophagogastroduodenoscopy (EGD) that reveals a 5 cm tumor in the fundus of the stomach. The biopsy result is gastric adenocarcinoma. An endoscopic sonogram shows a T3N1 tumor. Staging computed tomographic (CT) scans of the chest, abdomen, and pelvis are negative for metastases. Appropriate management includes: A) Definitive chemotherapy and radiotherapy B) Subtotal gastrectomy with D1 lymph node dissection and Billroth II reconstruction C) Subtotal gastrectomy with D2 lymph node dissection and Billroth II reconstruction D) Total gastrectomy with D1 lymph node dissection and Billroth II reconstruction E) Total gastrectomy with D2 lymph node dissection and Billroth II reconstruction

To demonstrate understanding of the treatment for gastric adenocarcinoma Answer E is correct Surgical resection remains the only potentially curative therapy for localized gastric cancer. Cure requires removal of all gross and microscopic disease. More specifically, a margin-negative (R0) resection entails wide local excision of the primary tumor with en bloc removal of all associated lymphatic vessels and any local or regional extension of disease. The evidence at present does not support routine performance of total gastrectomy for lesions of the distal fundus or antrum provided that histologically negative margins are achievable without compromise of the gastric inlet. A recommended practice is to perform a subtotal gastrectomy with Billroth II reconstruction for tumors of the distal stomach and total gastrectomy with Roux-en-Y esophagojejunostomy for most cancers of the fundus and the proximal stomach. D1 lymph node dissection involves resection of the perigastric lymph nodes along the greater and lesser curvature of the stomach. A D0 dissection is anything less than a D1 dissection. A D2 dissection entails resection of the D1 nodes along with nodes along the common hepatic artery, the left gastric artery, the celiac axis, and the splenic artery. A D3 lymph node dissection adds resection of nodes in the hepatoduodenal ligament and the root of the mesentery. Finally, a D4 resection calls for a D3 dissection plus resection of the retroperitoneal para-aortic and paracolic lymph nodes. Current American Joint Committee on Cancer guidelines state that pathologic examination of at least 15 lymph nodes is required for adequate staging. Recommended practice is to perform a D2 lymph node dissection, with resection of all perigastric lymph nodes along the greater and lesser curvatures of the stomach, as well as those along the common hepatic artery, the left gastric artery, the celiac axis, and the splenic artery.

A relatively fit 74-year-old gentleman undergoes esophagogastroduodenoscopy (EGD) that reveals a 5 cm tumor in the fundus of the stomach. The biopsy result is gastric adenocarcinoma. Endoscopic ultrasonography shows a T3N1 tumor. Staging CT scans of the chest, abdomen, and pelvis are negative for metastases. Appropriate management includes: A) Definitive chemotherapy and radiotherapy B) Subtotal gastrectomy with D1 lymph node dissection and Billroth II reconstruction C) Subtotal gastrectomy with D2 lymph node dissection and Billroth II reconstruction D) Total gastrectomy with D1 lymph node dissection and Roux-en-Y esophagojejunostomynt E) Total gastrectomy with D2 lymph node dissection and Roux-en-Y esophagojejunostomy

To demonstrate understanding of the treatment for gastric adenocardinoma Answer E is correct Surgical resection remains the only potentially curative therapy for localized gastric cancer. Cure requires removal of all gross and microscopic disease. More specifically, a margin-negative (R0) resection entails wide local excision of the primary tumor with en bloc removal of all associated lymphatic vessels and any local or regional extension of disease. The evidence at present does not support routine performance of total gastrectomy for lesions of the distal fundus or antrum, provided that histologically negative margins are achievable without compromise of the gastric inlet. A recommended practice is to perform a subtotal gastrectomy with Billroth II reconstruction for tumors of the distal stomach and total gastrectomy with Roux-en-Y esophagojejunostomy for most cancers of the fundus and the proximal stomach. D1 lymph node dissection involves resection of the perigastric lymph nodes along the greater and lesser curvature of the stomach. A D0 dissection is anything less than a D1 dissection. A D2 dissection entails resection of the D1 nodes along with nodes along the common hepatic artery, the left gastric artery, the celiac axis, and the splenic artery. A D3 lymph node dissection adds resection of nodes in the hepatoduodenal ligament and the root of the mesentery. Finally, a D4 resection calls for a D3 dissection plus resection of the retroperitoneal para-aortic and paracolic lymph nodes. Two prospective trials from Western Europe examined this issue further in an effort to evaluate the safety and efficacy of extended lymph node dissection. In the Dutch Gastric Cancer Group trial, 711 patients were randomly assigned to undergo either D1 or D2 lymphadenectomy as part of a potentially curative gastrectomy for biopsy-proven adenocarcinoma. Patients without evidence of disseminated metastases underwent either total gastrectomy or, if 5 cm proximal margins could be obtained, distal gastrectomy. In this study, a D2 lymph node dissection entailed distal pancreatectomy and splenectomy. Both morbidity and mortality were significantly higher in the D2 group than in the D1 group, and D2 dissection conferred no demonstrable survival advantage at a median follow-up of 72 months. Current AJCC guidelines state that pathologic examination of at least 15 lymph nodes is required for adequate staging.

A 51-year-old female was referred to you with a large mass of the stomach that is a biopsy-proven GIST. She has been told of a "targeted" therapy option using a drug called imatinib. What is the molecular target of this drug? A) c-myc B) Ret C) Fas ligand D) Epidermal growth factor receptor E) c-kit

To demonstrate understanding of the treatment of GISTs Answer E is correct. The characterization and targeting of the tyrosine kinase receptor c-kit have generated a new approach to treating GISTs. Phase III trials comparing standard- and high-dose imatinib mesylate in the treatment of these lesions have been completed in the United States and Europe. In these two trials, the progression-free response rate ranged from 43 to 53%, and the estimated 2-year survival ranged from 69 to 78%. The use of imatinib mesylate in adjuvant and neoadjuvant settings is still evolving, but recent data suggest a survival advantage of using imatinib in a neoadjuvant approach.

A 71-year old female is postoperative day 5 from a hip replacement when she suffers from hematemesis. Esophagogastroduodenoscopy reveals a bleeding posterior ulcer in the first portion of the duodenum. Endoscopic clipping and sclerotherapy were unsuccessful in stopping the bleeding. She has no history of peptic ulcer disease. She is receiving her sixth unit of blood and is hypotensive. Appropriate management includes: A) Repeat attempt at endoscopic clipping B) Interventional radiology consultation for embolization C) Proximal duodenotomy and omental (Graham) patch D) Proximal duodenotomy with oversewing of the ulcer E) Antrectomy and vagotomy

To demonstrate understanding of the treatment of a bleeding duodenal ulcer Answer D is correct The patient in this question is unstable and has failed nonoperative therapy. Surgical treatment involves exposing the posterior duodenal ulcer through a proximal duodenotomy. More specifically, a Kocher maneuver is performed and the pylorus is identified through palpation and identification of the pyloric vein as it courses anteriorly. Two traction sutures are placed in the anterior aspect of the pylorus, one superiorly and one inferiorly. A longitudinal incision is made that extends approximately 2 to 3 cm on either side of the pylorus. The bleeding vessel (gastroduodenal artery) is then ligated by directly oversewing the ulcer bed with at least three sutures. Figure-of-eight sutures are placed on the superior and inferior borders of the ulcer bed to ligate the gastroduodenal artery proximal and distal to the ulcer, and a third figure-of-eight suture is placed medially to control the transverse pancreatic branch.

A 67-year-old woman who initially presented with vague abdominal discomfort and was found to have a mass in the distal stomach along the greater curve is referred to you. Endoscopic biopsy revealed low-grade mucosa-associated lymphoid tissue (MALT) lymphoma, and staging CT scans showed that the tumor was localized to the stomach. The next step in management includes: A) Total gastrectomy B) Total gastrectomy with radical lymph node dissection C) Distal gastrectomy D) Distal gastrectomy with radical lymph node dissection E) Oral antibiotics

To demonstrate understanding of the treatment of gastric lymphoma Answer E is correct Gastric lymphoma is the second most common malignancy of the stomach, accounting for 2 to 9% of gastric tumors in the United States. Lymphomas of the stomach are of the non-Hodgkin type. The stomach is the most common site of extranodal involvement of non-Hodgkin lymphoma and accounts for nearly 50% of all such cases. Risk factors for gastric lymphoma include H. pylori infection, immunosuppression after solid-organ transplantation, celiac disease, inflammatory bowel disease, and HIV infection. Over the past decade, the management of patients with gastric lymphoma has undergone significant changes. Generally, there has been a shift away from surgical management, even in relatively localized cases (stages I and II). This shift is the result not only of the documented success of chemotherapy alone for more advanced cases (stages III and IV) but also of a better understanding of the etiology of gastric lymphoma. Approximately 45% of all gastric lymphomas are low-grade MALT lymphomas. The gastric mucosa is normally devoid of lymphoid tissue. It is hypothesized that MALT develops in the stomach in response to chronic H. pylori infection. Low-grade MALT lymphoma usually presents as stage I or II disease and has an indolent course. Since 1993, when regression of low-grade MALT lymphoma after eradication of H. pylori was first reported, numerous trials have documented the efficacy of anti-H. pylori therapy, with complete remission rates ranging from 50 to 100%. Low-grade lymphomas that are more advanced or do not regress with antibiotic therapy may be treated with H. pylori eradication and radiation (with or without chemotherapy). For localized persistent disease, modest doses of radiation, on the order of 30 Gy, may be employed. When chemotherapy is required, multiagent regimens, such as cyclophosphamide, vincristine (Oncovin), and prednisolone (COP), are often used. Surgical resection, once thought to be essential for the diagnosis, staging, and treatment of early-stage gastric lymphoma, now is used mainly in patients who experience bleeding or perforation.

An 88-year-old woman with Alzheimer disease is brought to the emergency department in a moribund condition by her daughter. She has hypotension that is refractory to intravenous fluids, and an upright chest radiograph reveals free air. Her daughter reports a history of gastric ulcers for which she takes a high-dose proton pump inhibitor. At laparotomy, the peritoneum is soiled with bile and a 1.5 cm anterior perforation is noted in the anterior second portion of the duodenum. The most appropriate surgical management includes: A) Primary repair of the ulcer B) Omental (Graham) patch C) Omental (Graham) patch and truncal vagotomy D) Highly selective vagotomy E) Antrectomy

To demonstrate understanding of the treatment of perforated duodenal ulcers Answer B is correct The first priority of an ulcer operation is to manage complications (e.g., bleeding and perforation); whether an accompanying acid-suppressing procedure is indicated and which one should be done depend on the clinical setting. Most commonly, duodenal perforation is treated with closure and omental patching, with or without an acid-reducing procedure. An important component of preoperative evaluation is determination of the severity and duration of disease. If symptoms are long-standing or recurrent—particularly when the patient has already received acid-reducing or anti-Helicobacter pylori therapy—a definitive acid-reducing procedure should be considered. As has been demonstrated, however, ulcer recurrence is significantly reduced even without an acid-reducing procedure when perforated ulcers are managed with anti-H. pylori agents in conjunction with a proton pump inhibitor. Therefore, a compliant patient who is presumed to be infected with H. pylori may not need to undergo an additional acid-reducing procedure. Perforated duodenal ulcers are typically treated by oversewing the perforation and then placing a portion of the greater omentum over the suture line. The duodenal ulcer wall is carefully débrided and closed with three or four interrupted silk sutures; the tails of the sutures may be used to hold the omentum in place. Alternatively, if the ulcer edges are edematous and not expected to close easily, the perforation may be closed with a true Graham patch, which involves plugging the defect with a well-vascularized omental pedicle. There is some controversy regarding whether an acid-reducing operation should be added to the omental patch procedure. If the patient is stable and has a history of peptic ulcer disease, a definitive ulcer operation is included; highly selective vagotomy may be preferable to truncal vagotomy and pyloroplasty in that it is less likely to give rise to dumping syndrome and postvagotomy diarrhea. If the patient has no history of peptic ulcer disease, has a severe medical illness, is hemodynamically unstable, has a long-standing perforation, or exhibits gross abdominal contamination, a definitive ulcer operation is omitted. In cases of gross contamination of the abdomen, it is probably inappropriate to divide the peritoneum over the esophagus during vagotomy and thereby expose the mediastinum to infection.

An 88-year-old woman with Alzheimer disease is brought to the emergency department in moribund condition by her daughter. She has hypotension that is refractory to intravenous fluids, and an upright chest x-ray reveals free air. Her daughter reports a history of gastric ulcers for which she takes a high-dose proton pump inhibitor. At laparotomy, the peritoneum is soiled with bile and a 1.5 cm anterior perforation is noted in the anterior second portion of the duodenum. The most appropriate surgical management includes: A) Primary repair of the ulcer B) Omental (Graham) patch C) Omental (Graham) patch and truncal vagotomy D) Highly selective vagotomy E) Antrectomy

To demonstrate understanding of the treatment of perforated duodenal ulcers Answer B is correct The first priority of an ulcer operation is to manage complications (e.g., bleeding and perforation); whether an accompanying acid-suppressing procedure is indicated and which one should be done depend on the clinical setting. Most commonly, duodenal perforation is treated with closure and omental patching, with or without an acid-reducing procedure. An important component of preoperative evaluation is determination of the severity and duration of disease. If symptoms are long-standing or recurrent—particularly when the patient has already received acid-reducing or anti-Helicobacter pylori therapy—a definitive acid-reducing procedure should be considered. As has been demonstrated, however, ulcer recurrence is significantly reduced even without an acid-reducing procedure when perforated ulcers are managed with anti-H. pylori agents in conjunction with a proton pump inhibitor. Therefore, a compliant patient who is presumed to be infected with H. pylori may not need to undergo an additional acid-reducing procedure. Perforated duodenal ulcers are typically treated by oversewing the perforation and then placing a portion of the greater omentum over the suture line. The duodenal ulcer wall is carefully débrided and closed with three or four interrupted silk sutures; the tails of the sutures may be used to hold the omentum in place. Alternatively, if the ulcer edges are edematous and not expected to close easily, the perforation may be closed with a true Graham patch, which involves plugging the defect with a well-vascularized omental pedicle. There is some controversy regarding whether an acid-reducing operation should be added to the omental patch procedure. If the patient is stable and has a history of peptic ulcer disease, a definitive ulcer operation is included; highly selective vagotomy may be preferable to truncal vagotomy and pyloroplasty in that it is less likely to give rise to dumping syndrome and postvagotomy diarrhea. If the patient has no history of peptic ulcer disease, has a severe medical illness, is hemodynamically unstable, has a long-standing perforation, or exhibits gross abdominal contamination, a definitive ulcer operation is omitted. In cases of gross contamination of the abdomen, it is probably inappropriate to divide the peritoneum over the esophagus during vagotomy and thereby expose the mediastinum to infection.

A patient whose father had colon cancer comes to your office concerned about whether he should undergo a screening colonoscopy. According to screening guidelines, when and how often should patients at high risk for developing colorectal cancer (e.g., those with a family history of colorectal cancer) undergo colonoscopy? A) At age 50 or 5 years younger than the age of the affected first-degree relative at 5-year intervals B) At age 50 or 15 years younger than the age of the affected first-degree relative at 10-year intervals C) At age 40 or 5 years younger than the age of the affected first-degree relative at 10-year intervals D) At age 40 or 10 years younger than the age of the affected first-degree relative at 5-year intervals E) At age 40 or 5 years younger than the age of the affected first-degree relative at 10-year intervals

To describe the screening guidelines for patients at high risk for developing colorectal cancer Answer D is correct Many groups have advocated CRC screening, and published guidelines are available from several organizations, including a collaborative guideline published by the American Cancer Society, the U.S. Preventive Services Task Force, and the American College of Radiology. Others include the American College of Gastroenterology and the National Comprehensive Cancer Network (NCCN). All of these guidelines recommend that screening begin at age 50 for average-risk patients with colonoscopy at 10-year intervals. The recommended screening options in patients unwilling or unable to undergo colonoscopy are fairly consistent among the various organizations and include (1) fecal occult blood testing (FOBT) yearly, (2) flexible sigmoidoscopy every 5 years, (3) yearly FOBT and flexible sigmoidoscopy every 5 years, (4) double-contrast barium enema every 5 years, and (5) CT colonography every 5 years. In high-risk patients (e.g., those with a family history of CRC), screening may begin at an earlier age—generally at age 40 or 10 years younger than the age of the affected first-degree relative and with shorter, 5-year intervals. There are also specific intensive screening and follow-up regimens for patients with known or suspected familial cancer syndromes.

A 57-year-old man with a history of diabetes mellitus and two previous attacks of diverticulitis requiring hospitalization presents to the emergency department with left lower quadrant pain, diarrhea, and a temperature of 37°C (100.3°F). On physical examination, he is tender to palpation in the left lower quadrant with mild rebound tenderness and guarding. Computed tomography (CT) is performed and reveals multiple sigmoid diverticula with bowel wall thickening and inflammation of the adjacent pericolic fat. What distinguishes a diagnosis of complicated diverticulitis from that of simple diverticulitis? A) The previous number of attacks of diverticulitis a patient has experienced B) The patient's comorbid conditions (i.e., degree of organ failure, immunosuppression, age) C) Diverticulitis involving the cecum D) The presence or absence of obstruction, abscess, fistula, or free perforation E) All of the above

To distinguish simple diverticulitis from complicated diverticulitis Answer E is correct. Some cases of diverticulitis are classified as complicated, meaning that the disease process has progressed to obstruction, abscess or fistula formation, or free perforation. Complicated diverticulitis may be particularly challenging to manage, especially because patients may have no known history of diverticular disease. Lower gastrointestinal bleeding is also a complication of diverticular disease in 30 to 50% of cases; in fact, diverticula are the most common colonic cause of lower gastrointestinal bleeding.

A 63-year-old man undergoes a low anterior resection for a rectal cancer 8 cm from the anal verge. In patients undergoing surgical treatment of rectal cancer, which of the following is not considered an adequate resection margin? A) Radial margin less than 2 mm B) Histologically negative margin in patients receiving adjuvant chemoradiation therapy C) Distal margin less than 5 cm D) Distal margin of 2 cm E) Radial margin of 5 mm

To identify adequate resection margins in rectal cancer and understand the importance of an adequate radial margin Answer A is correct There has been a great deal of debate about what constitutes an adequate margin of resection in surgical treatment of rectal cancer. With respect to distal margins, 2 to 5 cm has traditionally been considered to be the minimum necessary for curative resection. Growing interest in sphincter preservation has led investigators to consider smaller distal margins (i.e., < 2 cm). Studies have shown that clear margins smaller than 2 cm are not associated with higher local recurrence rates or reduced survival. Subsequent reports have suggested that even smaller histologically negative margins (i.e., < 1 cm) may be adequate in patients receiving adjuvant chemoradiation therapy. The importance of radial margin involvement after rectal cancer resection was not recognized until comparatively recently. Radial margins are assessed by means of serial slicing and evaluation of multiple coronal sections of the tumor and the mesorectum. Involvement of radial margins is a predictor of both local recurrence and survival after potentially curative rectal cancer surgery and may be associated with an increased risk of distant metastases. Radial margins smaller than 2 mm are associated with increased local recurrence rates. Adjuvant radiation therapy does not compensate for the adverse impact of positive margins on local recurrence rates.

A 48-year-old man presents with a single 3 cm metastatic lesion in the left lateral segment of the liver 4 years after right colectomy for colon cancer. In patients with isolated liver metastases from CRC, what is the approximate 5-year survival rate after resection? A) 5% B) 15% C) 20% D) 30% E) 50%

To identify approximate 5-year survival after hepatic metasectomy in patients with colorectal cancer Answer D is correct Numerous studies have addressed the treatment of patients with isolated liver metastases from CRC. Resection of isolated hepatic metastases has been reported to yield 5-year survival rates higher than 30%, with acceptable surgical morbidity and mortality. Investigators from the Memorial Sloan-Kettering Cancer Center developed a staging system known as the clinical score in an attempt to predict which patients are likely to benefit from aggressive surgical resection. This system used five factors that were found to be independent predictors of poor outcome: (1) node-positive primary disease, (2) a disease-free interval shorter than 12 months, (3) the presence of more than one hepatic tumor, (4) a maximum hepatic tumor size exceeding 5 cm, and (5) a CEA level higher than 200 ng/mL. Patients who met no more than two of these criteria generally had good outcomes, whereas those who met three or more were recommended for inclusion in adjuvant therapy trials.

Which of the following factors is not associated with an increased risk of developing diverticular disease and its complications? Please choose the single most appropriate answer to the question. A) Sedentary lifestyle B) Constipation C) High-fiber diet D) Frequent use of nonsteroidal antiinflammatory drugs (NSAIDs) E) Smoking

To identify factors that contribute to the development of diverticular disease and its associated complications Answer C is correct Several factors appear to promote the development of diverticular disease and its complications, including decreased physical activity, intake of NSAIDs, smoking, and constipation from any cause (e.g., diet or medications). The well-known Western afflictions cholelithiasis, diverticulosis, and hiatal hernia frequently occur together (Saint triad). Obesity has been associated with the intake of low-fiber diets, and growing numbers of young, obese patients with diverticulitis are being seen by physicians.

A patient presents to your office with a cancerous rectal polyp and is interested in transanal excision. Which of the following criteria does not preclude a transanal excision of a rectal cancer? A) Tumor greater than 3 cm in diameter B) Tumor encompassing greater than 30% circumference of the rectum C) Tumor greater than 8 cm from the anal verge D) Tumor invading the submucosa E) Regional lymph node involvement

To identify indications and contraindications to local excision of rectal cancer Answer D is correct T1 disease is, by definition, tumor invading the submucosa without evidence of regional lymph node involvement or metastatic disease. T1 disease is not a contraindication to transanal excision of a rectal cancer. Several criteria have been established to identify patients who may be candidates for transanal excision. According to guidelines published by the National Comprehensive Cancer Network (NCCN), the lesion must be no more than 3 cm in diameter, must encompass no more than 30% of the circumference of the rectum, and must be less than 8 cm from the anal verge. With the advent of transanal endoscopic microsurgery (TEMS), these criteria have been expanded to include patients with more proximal lesions. Poorly differentiated tumors and the presence of lymphovascular invasion may also be associated with increased nodal involvement and higher recurrence rates.

A 67-year-old woman is readmitted to the hospital with pneumonia 9 days after undergoing a right total knee replacement. She complains of crampy abdominal pain with distention as well as nausea and vomiting. She is afebrile, with normal vital signs. Her abdomen is distended on examination without evidence of peritoneal signs. An abdominal plain film is obtained and shows a massively dilated colon with a cecal diameter of 8 cm. What is the most appropriate next step in management? A) Observation B) Nihil per os (NPO), nasogastric tube decompression, correct electrolyte and metabolic abnormalities, minimize narcotic and anticholinergic medications, place a rectal tube, and give a tap-water enema C) Give neostigmine, 2.5 mg IV over 2 to 3 minutes D) Perform colonoscopic decompression E) Exploratory laparotomy

To identify the appropriate steps in the management of a patient with colonic pseudo-obstruction Answer B is correct Pseudo-obstruction can exist in the small bowel or colon and can be either acute or chronic. Acute colonic pseudo-obstruction, also known as Ogilvie syndrome, is the most common form. Colonic pseudo-obstruction occurs most commonly in hospitalized patients in the postoperative period or in response to a nonsurgical acute illness (e.g., pneumonia, myocardial infarction, hypoxia, shock, intestinal ischemia, or electrolyte imbalance). The pathophysiologic mechanisms underlying idiopathic pseudo-obstruction appear to be related to an imbalance in the parasympathetic and sympathetic influences on colonic motility. The presenting symptoms of acute colonic pseudo-obstruction are massive dilatation of the colon (with the cecum more dilated than the distal colon), crampy pain, nausea, and vomiting. If peritoneal irritation or systemic toxicity is present, immediate laparotomy is indicated; if not, treatment involves nasogastric decompression, placement of a rectal tube, tap-water enemas, correction of any underlying metabolic disturbances, and avoidance of narcotic and anticholinergic medications. With conservative management, acute colonic pseudo-obstruction resolves within 4 days in more than 80% of cases. Colonoscopy was previously the method of choice for decompression in this setting. It has been shown, however, that intravenous administration of neostigmine, 2.5 mg over 2 to 3 minutes, leads to prompt resolution of acute colonic pseudo-obstruction within minutes in nearly all cases. Now that this previously difficult and potentially lethal problem can readily be treated pharmacologically, colonoscopic decompression and surgical intervention should be reserved for cases in which pharmacologic measures fail.

A 32-year-old woman with ulcerative colitis has been treated with infliximab for a number of years. She becomes acutely ill with toxic megacolon, and an urgent subtotal colectomy is planned. In patients undergoing surgery for ulcerative colitis while concurrently taking infliximab, which of the following statements is not true? A) There is a higher incidence of postoperative infection B) Wound healing is not impaired C) Adverse effects of treatment include infusion and hypersensitivity reactions D) Patients undergoing an ileoanal procedure should first undergo a staged abdominal colectomy E) Data on the safety and efficacy of treatment during fulminant colitis are limited

To identify the effects of infliximab in the surgical planning and management of patients with ulcerative colitis Answer B is correct Infliximab is a chimeric monoclonal antibody directed against human tumor necrosis factor (TNF). Infliximab has been used to treat Crohn disease for over 10 years. It was not until 2005, however, that infliximab was approved for use in patients with ulcerative colitis. Potential adverse effects from infliximab include activation of tuberculosis, infusion reactions, hypersensitivity reactions, the development of lymphoma, and infectious complications. Additionally, recent accumulating data suggest that infliximab may significantly increase the risk of postoperative infection and healing complications following surgery for ulcerative colitis. The use of infliximab therapy in the setting of fulminant colitis is controversial. Whereas the data supporting infliximab in the treatment of moderate to moderately severe disease are convincing, the data demonstrating safety and efficacy in the setting of fulminant colitis are limited. Based on the current data, some experts have supported the initiation of infliximab treatment in patients with fulminant colitis who have failed IV steroid therapy. On the other hand, others have advocated the avoidance of infliximab therapy in patients with fulminant colitis. With wider use of anti-TNF therapy for ulcerative colitis, a greater percentage of patients admitted to the hospital with fulminant colitis are likely to already be on infliximab therapy; hence, the decision whether or not to initiate therapy is often moot. Because of the risk of severe complications, infliximab therapy should not be given in combination with cyclosporine therapy. Because of the increased risk for anastomotic leak and pelvic sepsis in patients undergoing the ileoanal procedure while on anti-TNF therapy, patients with fulminant or severe ulcerative colitis should have a staged abdominal colectomy prior to the ileoanal procedure if they are being treated with infliximab. The negative effect of infliximab on wound healing and infection is prolonged and appears to persist beyond 3 to 4 months after the last dose given. The prolongation of medical therapy in patients with severe disease who have already received high doses of corticosteroids has caused concerns that those patients who fail both steroid and subsequent cyclosporine or infliximab therapy may be at high risk for perioperative morbidity and mortality. Current experience, however, has not identified an increased risk for perioperative complications in patients who fail to respond to cyclosporine therapy and thus does not appear to compromise surgical results. Yet recent data on infliximab in ulcerative colitis do demonstrate a trend toward increased number and severity of septic complications.

A 44-year-old woman presents with a several-year history of constipation. She has no history of recent weight loss, nausea, vomiting, or bleeding per rectum. Previous efforts to modify her diet, including increased fiber and water intake, and increased physical activity have proven unsuccessful in alleviating her constipation. Physical examination demonstrates a normal body habitus and an unremarkable abdominal examination. On digital rectal examination, there are no masses. When asked to increase rectal tone, she is not able to do so. During a balloon expulsion test, she is not able to expel the balloon within 1 minute. The external anal sphincter contracts during manometry. Which of the following is the most appropriate diagnosis? A) Constipation-predominant irritable bowel syndrome B) Slow-transit constipation C) Normal-transit constipation D) Nonrelaxing puborectalis E) Constipation with suspected underlying cause

To identify the findings associated with a diagnosis of nonrelaxing puborectalis Answer D is correct All patients presenting with constipation should undergo a rectal examination. The anus should be examined for evidence of scarring or stricture. A digital rectal examination should be done to assess anal tone; high anal tone and inability to increase pressure when asked to squeeze are common findings in patients with obstructed defecation resulting from a nonrelaxing puborectalis. An effort should be made to look for any anterior defect in the rectovaginal septum, which would indicate the presence of a rectocele; such a defect, if present, may be made more prominent by having the patient strain. The finding of a rectal mass warrants further investigation. In general, diagnostic studies are conducted to rule out an underlying cause of constipation (e.g., partially obstructing colon cancer) and to diagnose specific disorders associated with severe constipation (e.g., a nonrelaxing puborectalis and slow-transit constipation). Therefore, the choice of investigative studies should be individualized according to the clinical situation. In patients with mild symptoms and poor dietary habits who have no indications of any secondary causes of constipation, no investigations need to be done on a routine basis. In patients with severe constipation, however, serum calcium concentrations, thyroid function tests, hemoglobin concentrations, glucose levels, serum electrolyte levels, and creatinine concentrations may be helpful. In patients who have very severe constipation or in whom medical management fails, further investigative tests are warranted. These tests are conducted to classify patients into three categories, each of which calls for a different treatment approach: (1) slow-transit constipation, (2) nonrelaxing puborectalis, and (3) normal-transit constipation. The initial investigations should include assessment of colonic transit time to determine if slow-transit constipation is present, as well as evaluation of pelvic floor function to determine if a nonrelaxing puborectalis is present. Pelvic floor studies are valuable for ruling out obstructed defecation as a cause of constipation. The balloon expulsion test can be performed in the office as an initial screening measure. A balloon filled with 50 mL of water is attached to tubing and placed in the rectum; patients with a nonrelaxing puborectalis generally cannot expel the balloon from the rectum in 1 minute while sitting on a commode. It should be kept in mind, however, that as many as 12% of patients with normal pelvic floor function will have difficulty with balloon expulsion in this setting. A thorough pelvic floor evaluation is best conducted in a pelvic floor laboratory with a specific interest in anorectal function. In addition to the balloon expulsion test, the evaluation generally involves manometry, including assessment of the reflexive relaxation of the internal sphincter after rectal distention. The presence of this reflexive relaxation rules out Hirschsprung disease as a cause of constipation. In patients with a nonrelaxing puborectalis, manometry during straining effort demonstrates abnormal function of the external sphincter — either failure to relax to enable expulsion or, on occasion, paradoxical contraction. Similar findings during straining can be documented by means of electromyography with a sponge electrode in the anal canal.

A 56-year-old male with no other significant past medical history returns to the office after undergoing a right colectomy without incident. His pathology report states that the specimen had negative margins, but the tumor was found to invade through the muscularis propria with evidence of regional lymph node metastases in two of 17 lymph nodes. He has no known distant metastases, which of the following is not true regarding the next most appropriate step in management? Please choose the single most appropriate answer to the question. A) Standard adjuvant chemotherapy includes 5-fluorouracil (5-FU) and leucovorin B) Standard adjuvant chemotherapy includes oxaliplatin C) Irinotecan improves survival without risk of systemic toxicity D) Postoperative adjuvant chemotherapy is considered standard of care E) In patients receiving 5-FU and leucovorin, there is no difference in outcome between patients receiving high-dose and low-dose leucovorin

To identify the standard of care and treatment regimen for stage III colon cancer Answer C is correct Postoperative systemic adjuvant therapy is the standard of care in patients with stage III disease. Intergroup Trial 0089 randomly assigned patients with high-risk stage II and III disease to receive either 5-FU plus high-dose leucovorin or 5-FU plus low-dose leucovorin. The investigators concluded that (1) the high-dose and low-dose regimens were equivalent, (2) a regimen consisting of four cycles of 5-FU with high-dose weekly leucovorin was equivalent to the low-dose leucovorin Mayo Clinic regimen, and (3) the addition of levamisole to the 5-FU plus leucovorin regimen did not improve survival. Clinical trials established 5-FU plus leucovorin as standard therapy for patients with high-risk stage II and stage III colon cancer. A new series of studies has now provided data regarding newer agents such as oxaliplatin, irinotecan, bevacizumab, and cetuximab. The MOSAIC trial, a large multinational trial, and the NSABP C-07 trial both demonstrated improved outcomes with the addition of oxaliplatin to 5-FU-based regimens. In the MOSAIC trial, the addition of oxaliplatin resulted in a 5.9% increase in disease-free survival at 5 years. There was also a statistically significant 4.6% improvement in overall survival seen in stage III patients with a 6-year median follow-up. Oxaliplatin is now considered a standard component of adjuvant therapy protocols for CRC. Irinotecan has not been shown to improve survival in the adjuvant setting and is associated with increased toxicity and is therefore not generally used.

You performed a sigmoid colectomy on a patient, and the pathology revealed stage II colon cancer. In patients with stage II colon cancer, which risk factor is not considered a possible indication for adjuvant chemotherapy? A) Bowel obstruction B) Colonic perforation C) High-grade or lymphovascular invasion D) The presence of fewer than 12 lymph nodes in the resected specimen E) Age over 80 years

To identify the subgroup of patients with stage II colon cancer who may benefit from adjuvant chemotherapy Answer E is correct. In stage II colon cancer patients who have undergone complete surgical resection, the relative risk of recurrence is small enough that adjuvant chemotherapy yields relatively little benefit in terms of survival. There is, however, a subgroup of patients who have recognized prognostic factors that significantly reduce survival and in whom adjuvant therapy is therefore more likely to be beneficial. These risk factors include (1) bowel obstruction, (2) colonic perforation, (3) high-grade or lymphovascular invasion, and (4) the presence of fewer than 12 lymph nodes in the resected specimen. Additional risk factors such as chromosome 18 deletions and microsatellite instability continue to be evaluated.

A 57-year-old man underwent an antrectomy with gastrojejunostomy for ulcer disease 30 years ago. He has persistent epigastric discomfort, and an upper endoscopy confirms alkaline reflux gastritis. Which of the following is true? A) If the patient has a Roux-en-Y gastrojejunostomy, there are no surgical options to correct the problem B) Alkaline reflux gastritis is rare after gastrojejunostomy C) This condition can usually be managed medically D) If the patient has a Billroth II reconstruction, it should be converted to a Roux-en-Y gastrojejunostomy E) The patient would not have developed this complication if he had a Billroth I reconstruction at the original operation

To learn about the development and management of postgastrectomy alkaline reflux gastritis Answer D is correct Alkaline reflux gastritis is common after antrectomy, occurring in 5 to 15% of patients. It is difficult to manage medically and often requires revisional surgery. Although most common after Billroth II gastrojejunostomy, it can develop after Billroth I gastroduodenostomy as well. If the patient has a Billroth II, it should be converted to a Roux-en-Y gastrojejunostomy to divert bile away from the stomach. In patients who already have a Roux-en-Y gastrojejunostomy, the jejunojejunostomy should be moved more distal to the stomach (45 to 60 cm) to better divert bile flow.

A 57-year-old man underwent an antrectomy with gastrojejunostomy for ulcer disease 30 years ago. He has persistent epigastric discomfort, and an upper endoscopy confirms alkaline reflux gastritis. Which of the following is true? A) If the patient has a Roux-en-Y gastrojejunostomy, there are no surgical options to correct the problem B) Alkaline reflux gastritis is rare after gastrojejunostomy C) This condition can usually be managed medically D) If the patient has a Billroth II reconstruction, it should be converted to a Roux-en-Y gastrojejunostomy E) The patient would not have developed this complication if he had a Billroth I reconstruction at the original operation

To learn about the development and management of postgastrectomy alkaline reflux gastritis Answer D is correct Alkaline reflux gastritis is common after antrectomy, occurring in 5 to 15% of patients. It is difficult to manage medically and often requires revisional surgery. Although most common after Billroth II gastrojejunostomy, it can develop after Billroth I gastroduodenostomy as well. If the patient has a Billroth II, it should be converted to a Roux-en-Y gastrojejunostomy to divert bile away from the stomach. In patients who already have a Roux-en-Y, the jejunojejunostomy should be moved more distal to the stomach (45 to 60 cm) to better divert bile flow.

You are asked to evaluate a 63-year-old man with progressive dysphagia to liquids and solids. An upper GI series shows a bird's beak appearance of the distal esophagus, and achalasia is suspected. Which of the following is true regarding achalasia? A) Upper endoscopy is not necessary in this patient if esophageal manometry confirms achalasia B) After performing a Heller myotomy, a Dor fundoplication is superior to a posterior partial fundoplication C) In a laparoscopic Heller myotomy, the myotomy should include 6 cm of distal esophagus and 2 to 2.5 cm of proximal stomach D) The classic finding on esophageal manometry in patients with achalasia is a hypertonic lower esophageal sphincter that relaxes normally E) Patients with achalasia and an esophagus diameter greater than 6 cm should undergo

To learn the diagnosis and management of achalasia Answer C is correct Esophageal manometry is the key test for establishing the diagnosis of esophageal achalasia. The classic manometric findings are (1) absence of esophageal peristalsis and (2) a lower esophageal sphincter that fails to relax appropriately in response to swallowing. In patients older than 60 years who have experienced the recent onset of dysphagia and excessive weight loss, secondary achalasia or pseudoachalasia from cancer of the esophagogastric junction should be ruled out. Either a Dor or a partial posterior (Guarner) fundoplication may be performed in conjunction with a Heller myotomy. Both have their advantages and disadvantages; neither has been shown to be superior. The Dor fundoplication is an anterior 180° wrap. Its advantages are that (1) it does not require posterior dissection and the creation of a window between the esophagus, the stomach, and the left pillar of the crus; (2) it covers the exposed esophageal mucosa after completion of the myotomy; and (3) it is effective even in patients with GERD. Its main disadvantage is that achieving the proper geometry can be difficult, and a wrong configuration can lead to dysphagia even after a properly performed myotomy. The advantages of the Guarner fundoplication are that (1) it is easier to perform; (2) it keeps the edges of the myotomy well separated; and (3) it might be more effective than a Dor procedure in preventing reflux. Its main disadvantages are that (1) it requires more dissection for the creation of a posterior window, and (2) it leaves the esophageal mucosa exposed. Once the mucosa has been exposed, the myotomy can safely be extended. Distally, it is extended for about 2 to 2.5 cm onto the gastric wall; proximally, it is extended for about 6 cm above the esophagogastric junction. Thus, the total length of the myotomy is typically about 8 to 8.5 cm. Historically, the presence of a sigmoid-shaped esophagus has been considered a contraindication to performing a myotomy to treat long-standing achalasia, making esophagectomy the only option. In a recent study, however, even when the esophageal diameter exceeded 6 cm and the esophagus was sigmoid, the outcomes of laparoscopic myotomy were not significantly different.

You are asked to evaluate a 63-year-old man with progressive dysphagia to liquids and solids. An upper GI series shows a bird's beak appearance of the distal esophagus, and achalasia is suspected. Which of the following is true regarding achalasia? A) Upper endoscopy is not necessary in this patient if esophageal manometry confirms achalasia B) After performing a Heller myotomy, a Dor fundoplication is superior to a posterior partial fundoplication C) In a laparoscopic Heller myotomy, the myotomy should include 6 cm of distal esophagus and 2 to 2.5 cm of proximal stomach D) The classic finding on esophageal manometry in patients with achalasia is a hypertonic lower esophageal sphincter that relaxes normally E) Patients with achalasia and an esophagus diameter greater than 6 cm should undergo esophagectomy

To learn the diagnosis and management of achalasia Answer C is correct Esophageal manometry is the key test for establishing the diagnosis of esophageal achalasia. The classic manometric findings are (1) absence of esophageal peristalsis and (2) a lower esophageal sphincter that fails to relax appropriately in response to swallowing. In patients older than 60 years who have experienced the recent onset of dysphagia and excessive weight loss, secondary achalasia or pseudoachalasia from cancer of the esophagogastric junction should be ruled out. Either a Dor or a partial posterior (Guarner) fundoplication may be performed in conjunction with a Heller myotomy. Both have their advantages and disadvantages; neither has been shown to be superior. The Dor fundoplication is an anterior 180° wrap. Its advantages are that (1) it does not require posterior dissection and the creation of a window between the esophagus, the stomach, and the left pillar of the crus; (2) it covers the exposed esophageal mucosa after completion of the myotomy; and (3) it is effective even in patients with GERD. Its main disadvantage is that achieving the proper geometry can be difficult, and a wrong configuration can lead to dysphagia even after a properly performed myotomy. The advantages of the Guarner fundoplication are that (1) it is easier to perform; (2) it keeps the edges of the myotomy well separated; and (3) it might be more effective than a Dor procedure in preventing reflux. Its main disadvantages are that (1) it requires more dissection for the creation of a posterior window, and (2) it leaves the esophageal mucosa exposed. Once the mucosa has been exposed, the myotomy can safely be extended. Distally, it is extended for about 2 to 2.5 cm onto the gastric wall; proximally, it is extended for about 6 cm above the esophagogastric junction. Thus, the total length of the myotomy is typically about 8 to 8.5 cm. Historically, the presence of a sigmoid-shaped esophagus has been considered a contraindication to performing a myotomy to treat long-standing achalasia, making esophagectomy the only option. In a recent study, however, even when the esophageal diameter exceeded 6 cm and the esophagus was sigmoid, the outcomes of laparoscopic myotomy were not significantly different.

You are asked to evaluate a 63-year-old man with progressive dysphagia to liquids and solids. An upper GI series shows a bird's beak appearance of the distal esophagus, and achalasia is suspected. Which of the following is true regarding achalasia? A) Upper endoscopy is not necessary in this patient if esophageal manometry confirms achalasia B) After performing a Heller myotomy, a Dor fundoplication is superior to a posterior partial fundoplication C) In a laparoscopic Heller myotomy, the myotomy should include 6 cm of distal esophagus and 2 to 2.5 cm of proximal stomach D) The classic finding on esophageal manometry in patients with achalasia is a hypertonic lower esophageal sphincter that relaxes normally E) Patients with achalasia and an esophagus diameter greater than 6 cm should undergo esophagectomy

To learn the diagnosis and management of achalasia Answer C is correct Esophageal manometry is the key test for establishing the diagnosis of esophageal achalasia. The classic manometric findings are (1) absence of esophageal peristalsis and (2) an LES that fails to relax appropriately in response to swallowing. In patients older than 60 years who have experienced the recent onset of dysphagia and excessive weight loss, secondary achalasia or pseudoachalasia from cancer of the esophagogastric junction should be ruled out. Either a Dor or a partial posterior (Guarner) fundoplication may be performed in conjunction with a Heller myotomy. Both have their advantages and disadvantages; neither has been shown to be superior. The Dor fundoplication is an anterior 180° wrap. Its advantages are that (1) it does not require posterior dissection and the creation of a window between the esophagus, the stomach, and the left pillar of the crus; (2) it covers the exposed esophageal mucosa after completion of the myotomy; and (3) it is effective even in patients with GERD. Its main disadvantage is that achieving the proper geometry can be difficult, and a wrong configuration can lead to dysphagia even after a properly performed myotomy. The advantages of the Guarner fundoplication are that (1) it is easier to perform; (2) it keeps the edges of the myotomy well separated; and (3) it might be more effective than a Dor procedure in preventing reflux. Its main disadvantages are that (1) it requires more dissection for the creation of a posterior window, and (2) it leaves the esophageal mucosa exposed. Once the mucosa has been exposed, the myotomy can be safely extended. Distally, it is extended for about 2 to 2.5 cm onto the gastric wall; proximally, it is extended for about 6 cm above the esophagogastric junction. Thus, the total length of the myotomy is typically about 8 to 8.5 cm. Historically, the presence of a sigmoid-shaped esophagus has been considered a contraindication to performing a myotomy to treat long-standing achalasia, making esophagectomy the only option. In a recent study, however, even when the esophageal diameter exceeded 6 cm and the esophagus was sigmoid, the outcomes of laparoscopic myotomy were not significantly different.

A 64-year-old man presents with weight loss and abdominal pain. Workup reveals an adenocarcinoma in the small intestine. Which of the following is the most common site of small bowel adenocarcinoma? A) Duodenum B) Proximal jejunum C) Distal jejunum D) Proximal ileum E) Distal ileum

To learn the most common location of small bowel adenocarcinomas Answer A is correct. Between 46 and 55% of small bowel adenocarcinomas occur in the duodenum. Patients frequently present with nausea, vomiting, abdominal pain, weight loss, and GI bleeding; occasionally, they present with iron deficiency anemia or a positive fecal occult blood test result. In rare cases, small bowel obstruction, often with the tumor serving as a lead point for intussusception, is the first manifestation of the disease.

A 64-year-old man presents with weight loss and abdominal pain. Workup reveals an adenocarcinoma in the small intestine. Which of the following is the most common site of small bowel adenocarcinoma? A) Duodenum B) Proximal jejunum C) Distal jejunum D) Proximal ileum E) Distal ileum

To learn the most common location of small bowel adenocarcinomas Answer A is correct. Between 46 and 55% of small bowel adenocarcinomas occur in the duodenum. Patients frequently present with nausea, vomiting, abdominal pain, weight loss, and gastrointestinal bleeding; occasionally, they present with iron deficiency anemia or a positive fecal occult blood test result. In rare cases, small bowel obstruction, often with the tumor serving as a lead point for intussusception, is the first manifestation of the disease.

The patient in question 1 is admitted to the hospital, made nihil per os (NPO), and started on intravenous fluids and antibiotics. Within several days, he has resolution of his symptoms and is discharged home. He returns 8 weeks later for an elective sigmoid resection with a colorectal anastomosis. After undergoing surgery, he asks you what his risk of recurrent diverticulitis is in the future. Which of the following is the most appropriate estimate of the recurrence rate of diverticulitis after sigmoid resection? A) 0% B) 1 to 10% C) 20 to 30% D) 30 to 40% E) More than 50%

To learn the recurrence rate of diverticulitis after sigmoid resection Answer B is correct Recurrent diverticulitis is rare after a colectomy for diverticulitis, occurring in 1 to 10% of patients. As many as 3% of patients who have undergone resection for diverticulitis will require repeat resection. The differential diagnosis includes Crohn disease, irritable bowel syndrome, carcinoma, and ischemic colitis. CT and colonoscopy should be carried out. Particular care should be taken to review pathologic specimens for evidence of Crohn disease. The only significant determinant of recurrent diverticulitis is the level of the anastomosis; the high pressure in the sigmoid colon distal to the anastomosis appears to be responsible. In one study, the risk of recurrence was four times greater in patients with a colosigmoid anastomosis than in those with a colorectal anastomosis. Reoperation requires a dissection that commences in noninflamed tissue. Dissection may be particularly difficult near the pelvic sidewall because of fibrosis; ureteral stenting may facilitate identification of the ureters.

A 21-year-old man suffered a subarachnoid hemorrhage after a motor vehicle collision. He is currently intubated and in the surgical intensive care unit. He subsequently develops bleeding from his nasogastric tube, and esophagogastroduodenoscopy (EGD) reveals diffuse gastritis. Which of the following is not true about acute hemorrhagic gastritis? A) Total gastrectomy is well tolerated in these patients B) Somatostatin may be beneficial C) A combination of acid-reducing medications may be necessary D) If Helicobacter pylori is present, antibiotics should be administered E) Infusion of vasopressin through the left gastric artery may reduce bleeding

To learn the treatment of acute hemorrhagic gastritis Answer A is correct Acute hemorrhagic gastritis typically occurs in critically ill patients. Stress ulcer prophylaxis can reduce its incidence. In patients who develop acute hemorrhagic gastritis, acid reduction should be undertaken to normalize the gastric pH; this may require H2 blockers, proton pump inhibitors, sucralfate, or antacids (either alone or in combination). If H. pylori is present, antibiotics should be administered. Other treatment options include intravenous somatostatin or a catheter-directed infusion of vasopressin directly into the left gastric artery. Total gastrectomy is a treatment of last resort as these patients are typically critically ill and are unlikely to tolerate such an extensive operation.

A 33-year-old man has been on long-standing medical therapy for his ileocolic Crohn disease but now appears to be failing medical management. Which of the following is true regarding ileocolic disease? A) Crohn disease is unlikely to recur after resection for ileocolic disease B) The ileum can fistulize to the ascending and sigmoid colon but not to the small intestine C) Ileocolic disease is rarely associated with intra-abdominal abscess D) Ileocolic resection is the most common operation for Crohn disease E) If an intra-abdominal abscess is detected, surgery should be performed immediately, consisting of drainage of the abscess, bowel resection, and primary anastomosis

To learn the treatment of ileocolic Crohn disease Answer D is correct Approximately half of those diagnosed with Crohn disease have ileocolic disease. Ileocolic resection is, in fact, the operation most frequently performed to treat Crohn disease. Ileosigmoid fistulas are among the most common fistulas associated with ileocolic Crohn disease, along with fistulas between the terminal ileum and the ascending colon and fistulas between the terminal ileum and adjacent loops of small bowel. Disease recurrence is common after ileocolic resection. The incidence of reoperation for recurrent disease after ileocolic resection is high and increases with the number of resections. Postoperative chemoprophylaxis with mesalamine can significantly reduce the recurrence rate. Ileocolic Crohn disease is often associated with intra-abdominal abscesses or fistulas. If an associated abscess is known to be present, CT-guided drainage should be done preoperatively so that a single-stage procedure can then be performed. If an unsuspected abscess is identified at the time of operation, the safest approach is to proceed with bowel resection, perform the posterior wall of the anastomosis, and exteriorize the anastomosis as a loop ileostomy. This loop ileostomy can then be safely closed, often without a formal laparotomy, 8 weeks after operation if there are no signs of ongoing sepsis. Performing a primary anastomosis in the setting of an abscess carries a high risk of anastomic leak.

In a patient with Crohn disease, which of the following is considered an absolute indication for surgery? Please choose the single most appropriate answer to the question A) Presence of enteroenteric fistula B) Abscess formation C) Small bowel obstruction D) Dysplasia E) Pregnancy

To recognize the indications for surgery in Crohn disease and identify conditions that may be amenable to attempts at medical management prior to undertaking surgical options Answer D is correct In many patients with Crohn disease, the behavior of the disease changes over time, from a more inflammatory and edematous process to one characterized more by fibrosis and scarring. Whereas antiinflammatory drugs are ideal for treating the former, surgery is frequently necessary for the latter. Failure to refer for surgical treatment of obstruction is, unfortunately, a common error among gastroenterologists. Severe abdominal pain is always a warning sign of obstruction and should be taken seriously. The importance of this point is illustrated by a case involving a patient who had obstructing ileocolic Crohn disease with gross proximal distention of the terminal ileum. This patient lost 20 lb, was experiencing severe abdominal pain, and was treated for more than a year with 6-mercaptopurine before being referred for operative management. Ileocolic resection led to rapid resolution of the symptoms. Enteroenteric fistulas, by themselves, are no longer considered an absolute indication for operation in the absence of other complicating factors. Symptomatic fistulas, such as those associated with obstruction or those associated with disabling symptoms (e.g., rectovaginal or enterocutaneous fistulas), may have to be treated surgically. Ileosigmoid fistulas, which effectively bypass the entire colon, may be associated with profound and refractory diarrhea (i.e., > 20 bowel movements/day) and may also have to be treated operatively. Abscesses are particularly common with ileocolic Crohn disease. If they cannot be controlled by means of computed tomography (CT)-guided drainage, surgical therapy may be indicated. The risk of colorectal cancer is approximately three times higher in patients with Crohn disease than in the general population. Screening recommendations for patients with long-standing Crohn colitis are similar to those for ulcerative colitis. In Crohn disease, cancer may also arise in long-standing fistula tracts. Persons who have Crohn disease may be less fertile than healthy, age-matched persons. One possible explanation for this difference is that feeling ill may result in reduced sexual desire or decreased sexual activity. Another is that pelvic inflammation caused by Crohn disease or by scarring and adhesion formation resulting from surgery may impair fertility. To reduce the chances of the latter, hyaluronic acid sheets may be placed around the tubes and ovaries; alternatively, the ovaries may be tacked to the undersurface of the anterior abdominal wall with absorbable sutures and thereby prevented from entering the pelvis and being trapped in scar tissue. There is no evidence that pregnancy exacerbates Crohn disease; however, there are some specific concerns that apply to pregnant patients with this condition. Because patients with Crohn disease often have bowel movements that are more liquid, they have a particular need for a well-functioning anal sphincter. If there is any chance of an obstetrics-related injury (e.g., from a large baby in a primagravida or from a breech presentation), a cesarean section is advisable to minimize the risk of sphincter trauma. The same is true in the presence of severe perianal Crohn disease. During pregnancy, prednisone and 5-aminosalicylic acid (5-ASA) medications are safe, whereas drugs such as metronidazole are not. If imaging studies are needed, magnetic resonance imaging and ultrasonography are the modalities of choice.

You are planning surgery in a 28-year-old man with ulcerative colitis. In patients with ulcerative colitis, absolute contraindications to immediate restorative proctectomy with ileal pouch-anal anastomosis include all of the following except: A) Perforation B) Peritonitis C) Sepsis D) Suspicion of Crohn colitis or indeterminate colitis E) Diagnosis of fulminant colitis

To recognize when a staged procedure is indicated and when immediate restorative proctectomy with ileal pouch-anal anastomosis is contraindicated Answer E is correct. The precise parameters under which it is best to stage the procedure with an initial abdominal colectomy have not been clearly defined. It is universally accepted that patients with perforation, peritonitis, or sepsis require a staged procedure. Any patient with suspicion of Crohn colitis or indeterminate colitis should undergo subtotal colectomy to allow for further diagnostic evaluation prior to creation of ileal pouch-anal anastomosis. Beyond this, there is no clear consensus. The available studies addressing this issue unfortunately involve a small number of patients, do not clearly define what is meant by "fulminant" colitis, or do not directly compare results between the two alternative strategies of staged colectomy versus immediate ileoanal anastomosis. Clearly, there is a small subset of patients with symptoms severe enough to require hospitalization who are healthy enough to safely undergo a primary ileoanal anastomosis. On the other end of the spectrum, severely ill patients, that is, most patients with fulminant colitis, should have a staged procedure. Because specific criteria to quantify the risk have not been defined, the decision to stage or not to stage ultimately rests with the clinical judgment of the experienced surgeon.

A 23-year-old woman presents to the hospital with low-grade fever, malaise, and bloody stools. Colonoscopy reveals pancolitis, and no other disease is detected on CT scan or physical examination. Which of the following is true regarding Crohn colitis? A) Total proctocolectomy with end-ileostomy is the traditional surgical procedure for Crohn colitis B) Colonic Crohn disease is easily distinguished from ulcerative colitis C) Granulomas are common in colonic Crohn disease D) If toxic megacolon develops, it should be treated with prednisone and antibiotics E) Subtotal colectomy with ileorectal anastomosis has a low recurrence rate

To understand Crohn colitis and its treatment Answer A is correct Colonic involvement is present in 29 to 44% of patients with Crohn disease. One of the challenges in treating colonic Crohn disease is obtaining the correct diagnosis. Whereas Crohn disease of the small bowel is fairly easy to diagnose, colonic disease often is not. Because granulomas are not present in most cases of colonic Crohn disease and because this condition can look very similar to ulcerative colitis both endoscopically and macroscopically, differentiation between Crohn colitis and ulcerative colitis can be difficult in the absence of small bowel or anal disease. Colonic Crohn disease appears to be more frequently associated with cutaneous manifestations (e.g., pyoderma gangrenosum). The main indications for operative management of colonic Crohn disease are stricture, malignancy, side effects of medical therapy, and failure of medical therapy. The traditional procedure for colonic Crohn disease is total proctocolectomy with end-ileostomy, which is associated with an 8 to 15% rate of recurrence in the small bowel proximal to the stoma. This operation remains the best choice in patients with severe rectal and anal Crohn disease and carries the lowest risk of disease recurrence. Because many patients with Crohn disease are young, surgeons have long been interested in operations that do not involve an ileostomy. In the absence of significant rectal and anal disease, subtotal colectomy with ileorectal or ileosigmoid anastomosis is an option. Unfortunately, this operation is associated with high recurrence rates (up to 70%). In common usage, the term toxic megacolon refers to any condition associated with colitis that is severe enough to result in sloughing of the colonic mucosa; such sloughing permits endotoxins to enter the circulatory system and evoke a septic response. The signs and symptoms of toxic megacolon include those characteristic of sepsis-leukocytosis, fever, tachycardia, and hypoalbuminemia. These patients are very ill and often manifest ileus, which is an ominous development that frequently signals impending perforation. Emergency surgical intervention is required.

A 83-year-old man has recurrent, unexplained episodes of GI bleeding. On the fourth episode, an actively bleeding Dieulafoy lesion is noted in the proximal stomach. Which of the following is true regarding Dieulafoy lesions and the management of this patient? A) Surrounding inflammation is common B) These lesions are typically found on the greater curvature C) Endoscopic therapy is rarely successful in controlling bleeding from these lesions D) If surgery is required, endoscopic tattooing of the lesion is recommended E) Surgical management often consists of an antrectomy

To understand Dieulafoy lesions and their management Answer D is correct Dieulafoy lesions are 1 to 3 mm bleeding vessels, which are typically found in the proximal stomach along the lesser curvature but can occur anywhere in the GI tract. They can be difficult to detect, and it is not uncommon for a patient to have multiple endoscopies before they are diagnosed. Surrounding inflammation is not present, and no pathologic abnormality is detected if a resection is performed. Surgery is rarely necessary as endoscopic therapy with clipping, cauterization, and sclerotherapy is successful in 95% of patients. Surgery, if required, can usually consist of an anterior gastrotomy and oversewing of the bleeding vessel. Sometimes resection of the vessel is employed, but antrectomy is not necessary. The lesions can be difficult to locate surgically, so preoperative endoscopic tattooing of the lesion is recommended.

A 54-year-old man underwent a left colectomy for colon cancer 2 weeks ago and now asks you about long-term follow-up. General recommendations for postoperative surveillance for patients with CRC include all of the following except A) Positron emission tomography (PET) every 4 months for 3 years followed by 8-month intervals for 3 years B) Measurement of carcinoembryonic antigen (CEA) levels every 2 to 3 months for 2 years, then every 3 to 6 months for 3 years, and then annually C) Clinical examination every 3 to 6 months fo 3 years and then annually D) Colonoscopy perioperatively and then every 3 to 5 years if the patient remains disease free E) Computed tomography (CT) of the chest and abdomen annually for 3 years

To understand and recognize the appropriate follow-up for patients how have undergone surgical resection of colorectal cancer Answer A is correct Several organizations, including the American Society of Clinical Oncology and the American Society of Colon and Rectal Surgeons, have developed algorithms for postoperative surveillance of CRC patients. Their recommendations generally apply to patients with stage II or III disease (and sometimes patients with T2 lesions) who are candidates for resection of recurrent disease. The recommendations vary somewhat among groups, but the following are generally agreed on: Measurement of CEA levels every 2 to 3 months for 2 years, then every 3 to 6 months for 3 years, and then annually Clinical examination every 3 to 6 months for 3 years and then annually Colonoscopy perioperatively and then every 3 to 5 years if the patient remains free of polyps and cancer (some guidelines also recommend colonoscopy 1 year after primary therapy) CT of the chest and abdomen annually for 3 years; pelvic CT should also be done in patients with rectal cance Other imaging studies (e.g., PET and chest x-ray) are not routinely recommended, nor are other blood tests (e.g., complete blood count and liver function tests).

A 68-year-old woman requires an antrectomy for gastric outlet obstruction secondary to benign prepyloric ulcers. Which of the following is true regarding antrectomy? A) A Billroth II reconstruction is another term for a gastrojejunostomy B) A Billroth I reconstruction maintains physiologic antegrade flow, which has been shown to be beneficial C) If a gastrojejunostomy is performed, the duodenal stump should always be closed with a lateral duodenostomy tube D) Omental patch reinforcement of the duodenal stump is contraindicated E) On the lesser curvature, the proximal extent of the antrum is just proximal to the incisura

To understand antrectomy and its reconstruction Answer E is correct Benign gastric outlet obstruction is a common indication for antrectomy. Proximally, the antrum extends just proximal to the incisura on the lesser curvature and to a point halfway between the pylorus and the fundus on the greater curvature. Options for reconstruction include a Billroth I (gastroduodenostomy) and a Billroth II (gastrojejunostomy). A Billroth I maintains antegrade flow. Although physiologic, this has not been proven to be beneficial. It does, however, eliminate the possibility of a duodenal stump leak and afferent and efferent loop syndrome, which can occur after Billroth II. If the duodenum appears uninflamed, a lateral duodenostomy tube is not necessary, but it can be helpful in the setting of an inflamed or difficult-to-close duodenal stump. Omental patch reinforcement of the duodenal stump can also be helpful in such an instance.

A 68-year-old woman requires an antrectomy for gastric outlet obstruction secondary to benign prepyloric ulcers. Which of the following is true regarding antrectomy? A) A Billroth II reconstruction is another term for a gastroduodenostomy B) A Billroth I reconstruction maintains physiologic antegrade flow, which has been shown to be beneficial C) If a gastrojejunostomy is performed, the duodenal stump should always be closed with a lateral duodenostomy tube D) Omental patch reinforcement of the duodenal stump is contraindicated E) On the lesser curvature, the proximal extent of the antrum is just proximal to the incisura

To understand antrectomy and its reconstruction Answer E is correct. Benign gastric outlet obstruction is a common indication for antrectomy. Proximally, the antrum extends just proximal to the incisura on the lesser curvature and to a point halfway between the pylorus and the fundus on the greater curvature. Options for reconstruction include a Billroth I (gastroduodenostomy) and a Billroth II (gastrojejunostomy). A Billroth I maintains antegrade flow. Although physiologic, this has not been proven to be beneficial. It does, however, eliminate the possibility of a duodenal stump leak and afferent and efferent loop syndrome, which can occur after Billroth II. If the duodenum appears uninflamed, a lateral duodenostomy tube is not necessary, but it can be helpful in the setting of an inflamed or difficult to close duodenal stump. Omental patch reinforcement of the duodenal stump can also be helpful in such an instance

A 53-year-old man is undergoing his third small bowel resection for Crohn disease. When operating on a patient with Crohn disease, if bowel is going to be resected, the extent of the resection margin is best determined by A) Previous response to medical therapy B) The distribution of lymph nodes throughout the mesentery C) Intraoperative frozen-section margins D) The number of strictureplasties that can be safely performed E) Identification of a palpably normal mesenteric border of bowel

To understand basic surgical principles in the management of Crohn disease Answer E is correct. The area to be resected should be as short as possible. There is no need to obtain frozen-section margins to determine the extent of resection; doing so leads to unnecessary loss of small bowel length. The resection should extend into palpably normal areas of small bowel. The easiest way of determining the area to be resected is to feel the mesenteric margin of the bowel until palpably normal tissue is reached. Because Crohn disease is generally more severe on the mesenteric side of the bowel, palpation in this area gives the most accurate impression of the intraluminal character of the bowel. Because it is not uncommon for patients to have multifocal Crohn disease, the entire small bowel should always be inspected at the time of operation. Operating on one area of disease while failing to treat a more proximal lesion is clearly not in the patient's interest.

A 62-year-old man is admitted to the medical intensive care unit with hypotension, melena, and a hematocrit of 23%. Placement of a nasogastric tube confirms upper gastrointestinal bleeding. After 3 units of packed red blood cells, he is hemodynamically stable. You perform an upper endoscopy, which reveals an ulcer in the first portion of the duodenum with a visible artery that is actively bleeding. Which of the following is not an option for endoscopic control of bleeding? A) Argon plasma coagulation B) Clip placement C) Injection sclerotherapy D) Coagulation E) Stent placement

To understand endoscopic techniques to control gastrointestinal bleeding Answer E is correct. Endoscopic techniques are typically successful in controlling gastrointestinal bleeding. Techniques can be classified as thermal and nonthermal. Thermal techniques can be further classified as contact (coagulation) and noncontact (argon plasma coagulation). Nonthermal techniques include injection sclerotherapy with epinephrine and endoscopic band, clip, or loop placement. Stent placement is used for obstructing neoplasms or strictures but is not an option for control of hemorrhage.

A 42-year-old woman underwent a laparoscopic GBP 3 years ago, and her weight has decreased from 260 to 140 pounds. She now presents with 2 weeks of intermittent periumbilical pain. Which of the following is true regarding internal hernias after GBP? A) Computed tomography (CT) always confirms the diagnosis of internal hernia B) Potential sites for internal hernia include the jejunojejunostomy anastomosis, the opening in the transverse mesocolon for the Roux limb, and between the transverse mesocolon and the Roux limb C) The chance of internal hernia is eliminated by suture closure of potential internal hernia sites during the original operation D) Internal hernias are less common after laparoscopic GBP than after the open procedure E) GBP patients are not at risk for bowel strangulation from internal hernia

To understand internal hernias after GBP Answer B is correct GBP places patients at risk for internal hernia with a closed-loop obstruction, leading to bowel strangulation. There are three typical locations for these internal hernias: the jejunojejunostomy anastomosis, the opening in the transverse mesocolon through which the retrocolic Roux limb is brought, and the Petersen hernia (located posterior to the Roux limb between the Roux mesentery and the transverse mesocolon). The primary symptom of an internal hernia is periumbilical pain, usually in the form of cramping consistent with visceral colic. These internal hernias may be very difficult to diagnose. Upper gastrointestinal radiographic series and abdominal CT scans are often normal, providing a false sense of security. The resulting assumption that no problem exists may be devastating for the patient should bowel infarction occur as a consequence of closed-loop obstruction. The plain abdominal radiograph should always be carefully inspected for abnormal placement or spreading of the staples in the Roux-en-Y anastomosis. The safest course of action in patients with recurrent attacks of cramping periumbilical pain is abdominal surgical exploration. The frequency of this complication seems to have increased with the advent of laparoscopic GBP, presumably because of the difficulty of closing the potential hernia spaces completely. Some attribute the problem to the decreased tendency toward adhesion formation after laparoscopic surgery or to reduction of the intra-abdominal fatty tissue in the mesentery after successful weight loss surgery. Even with suture closure of these defects at the time of operation, patients are still susceptible to internal herniation.

A 42-year-old woman underwent a laparoscopic gastric bypass (GBP) 3 years ago, and her weight has decreased from 260 to 140 pounds. She now presents with 2 weeks of intermittent periumbilical pain. Which of the following is true regarding internal hernias after gastric bypass? A) Computed tomographic (CT) scan always confirms the diagnosis of internal hernia B) Potential sites for internal hernia include the jejunojejunostomy anastomosis, the opening in the transverse mesocolon for the Roux limb, and between the transverse mesocolon and the Roux limb C) The chance of internal hernia is eliminated by suture closure of potential internal hernia sites during the original operation D) Internal hernias are less common after laparoscopic GBP than after the open procedure E) GBP patients are not at risk for bowel strangulation from internal hernia

To understand internal hernias after GBP Answer B is correct GBP places patients at risk for internal hernia with a closed-loop obstruction, leading to bowel strangulation. There are three typical locations for these internal hernias: the jejunojejunostomy anastomosis, the opening in the transverse mesocolon through which the retrocolic Roux limb is brought, and the Petersen hernia (located posterior to the Roux limb between the Roux mesentery and the transverse mesocolon). The primary symptom of an internal hernia is periumbilical pain, usually in the form of cramping consistent with visceral colic. These internal hernias may be very difficult to diagnose. Upper gastrointestinal radiographic series and abdominal CT scans are often normal, providing a false sense of security. The resulting assumption that no problem exists may be devastating for the patient should bowel infarction occur as a consequence of closed-loop obstruction. The plain abdominal radiograph should always be carefully inspected for abnormal placement or spreading of the staples in the Roux-en-Y anastomosis. The safest course of action in patients with recurrent attacks of cramping periumbilical pain is abdominal surgical exploration. The frequency of this complication seems to have increased with the advent of laparoscopic GBP, presumably because of the difficulty of closing the potential hernia spaces completely. Some attribute the problem to the decreased tendency toward adhesion formation after laparoscopic surgery or to reduction of the intra-abdominal fatty tissue in the mesentery after successful weight loss surgery. Even with suture closure of these defects at the time of operation, patients are still susceptible to internal herniation.

A 56-year-old woman is scheduled to have bariatric surgery, and she asks if her gallbladder will be removed at the time of the operation. Which of the following is true regarding the management of the gallbladder in patients undergoing bariatric surgery procedures? A) Cholecystectomy is easily performed at the time of laparoscopic GBP B) Ursodiol reduces the likelihood of development of gallstones when administered after GBP C) Cholecystectomy should never be performed during a GBP D) Gallstones are rare in obese patients E) Most bariatric surgeons remove the gallbladder at the time of laparoscopic GBP, even in the absence of gallstones

To understand management of the gallbladder in bariatric surgery Answer B is correct Gallstones are common in bariatric surgery patients. Approximately one third of morbidly obese patients either have had a cholecystectomy or may have had gallstones noted at the time of another intra-abdominal operative procedure (e.g., a gastric operation for morbid obesity). Preoperative evaluation of the gallbladder may be technically quite difficult in morbidly obese patients because ultrasonography may fail to visualize gallstones. Intraoperative ultrasonography is probably much more accurate. Should symptomatic gallstones be present in a patient undergoing a gastric procedure for obesity, the gallbladder should be removed if the surgeon judges it safe to perform this additional procedure. If placement of an adjustable gastric band is contemplated, the cholecystectomy should be undertaken first and the indwelling device placed only in the absence of intra-abdominal bile spillage during the procedure. In past studies, rapid weight loss led to the development of gallstones in 25 to 40% of patients who underwent GBP. The risk of cholelithiasis in this setting can be reduced to 2% by administering ursodeoxycholic acid, 300 mg orally twice daily. Laparoscopic cholecsytectomy at the time of laparoscopic GBP can be technically challenging; consequently, many surgeons prefer to take an expectant approach to the gallbladder rather than complicate the bariatric procedure with a simultaneous cholecystectomy (unless cholecystectomy is clearly indicated in a particular patient).

After suffering a stroke, a patient requires placement of a percutaneous endoscopic gastrostomy (PEG) tube for enteral feeding. Which of the following is true regarding PEG tubes? A) If a gastrocolic fistula develops, it will not close without surgery B) Excessive crossbar tension can cause tissue ischemia, resulting in separation of the gastric and abdominal walls and subsequent leakage of feedings into the abdominal cavity C) During the procedure, a needle is placed blindly through the abdominal wall and into the gastric lumen D) The tube should be placed 2 cm below the costal margin in the midclavicular line E) Local wound infections typically require removal of the PEG tube

To understand placement of PEG tubes and their potential complications Answer B is correct PEG tubes are commonly placed for enteral feeding. During the procedure, the endoscope is placed within the stomach, and the stomach is inflated. The endoscope is used to transilluminate the gastric and abdominal wall. The site of optimal transillumination is chosen as the site for tube placement, not necessarily 2 cm below the costal margin in the midclavicular line. The needle is passed into the gastric lumen under direct endoscopic visualization. Local wound infections are common and can typically be treated with antibiotics and/or simple drainage. Gastrocolic fistula can develop, usually presenting as diarrhea soon after feedings. If the PEG tract is mature, the PEG can be removed in patients with gastrocolic fistula and the hole will often seal without surgery. If the crossbar is positioned with excessive tension, it can lead to ischemia and separation of the stomach from the abdominal wall, potentially resulting in leaking of tube feedings into the peritoneal cavity.

A 72-year-old man is noted to have guaiac-positive stool. Which of the following is true regarding investigative studies to detect small bowel tumors? A) Capsule endoscopy is not useful for small bowel tumors because it does not permit biopsy of the tumor B) Push enteroscopy enables visualization of the entire small bowel C) Enteroclysis is better than small bowel follow-through for providing fine mucosal detail D) Tumors in the duodenum are typically diagnosed using upper gastrointestinal (GI) series E) Computed tomography (CT) is unable to identify small bowel tumors

To understand studies used to investigate for small bowel neoplasms Answer C is correct Although upper GI series can detect duodenal tumors, the diagnosis is usually made by means of esophagogastroduodenoscopy (EGD). Lesions within the first 100 cm of the small bowel may be evaluated with push enteroscopy. When the adenocarcinoma is situated elsewhere in the small bowel, it is localized with small bowel radiographs. Some authors consider enteroclysis to be superior to the more commonly used small bowel follow-through in this setting in that enteroclysis is better able to demonstrate fine mucosal detail. In experienced hands, enteroclysis may therefore be more sensitive. Some lesions are identified when CT or magnetic resonance imaging (MRI) is performed to evaluate complaints of abdominal pain. Furthermore, abdominal imaging may yield complementary staging information (e.g., the presence of regional adenopathy or metastatic disease). One promising new method for the identification of small bowel tumors is wireless capsule endoscopy. This minimally invasive technique may be particularly useful in identifying small lesions in the distal jejunum and ileum that cannot be identified radiographically.

A 72-year-old man is noted to have guaiac-positive stool. Which of the following is true regarding investigative studies to detect small bowel tumors? A) Capsule endoscopy is not useful for small bowel tumors because it does not permit biopsy of the tumor B) Push enteroscopy enables visualization of the entire small bowel C) Enteroclysis is better than SBFT for providing fine mucosal detail D) Tumors in the duodenum are typically diagnosed using upper gastrointestinal series E) CT is unable to identify small bowel tumors

To understand studies used to investigate for small bowel neoplasms Answer C is correct Although upper gastrointestinal series can detect duodenal tumors, the diagnosis is usually made by means of EGD. Lesions within the first 100 cm of the small bowel may be evaluated with push enteroscopy. When the adenocarcinoma is situated elsewhere in the small bowel, it is localized with small bowel radiographs. Some authors consider enteroclysis to be superior to the more commonly used SBFT in this setting, in that enteroclysis is better able to demonstrate fine mucosal detail. In experienced hands, enteroclysis may therefore be more sensitive. Some lesions are identified when CT or magnetic resonance imaging is performed to evaluate complaints of abdominal pain. Furthermore, abdominal imaging may yield complementary staging information (e.g., the presence of regional adenopathy or metastatic disease). One promising new method for the identification of small bowel tumors is wireless capsule endoscopy. This minimally invasive technique may be particularly useful in identifying small lesions in the distal jejunum and ileum that cannot be identified radiographically.

A 56-year-old man is postoperative day 7 from a sigmoid colectomy for a previous history of diverticulitis. There was no evidence of active diverticulitis at the time of operation, and the final pathology report ruled out any evidence of malignancy. He has not passed flatus or stool since his operation and complains of nausea as well as crampy abdominal pain. He has a nasograstric tube, which is having bilious output. On examination, he is afebrile, with normal vital signs, and has a distended abdomen without peritoneal signs. His wound is clean, dry, and intact, without evidence of infection. His white blood cell count is normal. A computed tomographic scan demonstrates distended, fluid-filled loops of small bowel with air-fluid levels, hyperemia, and bowel wall thickening. There is no evidence of abscess. There is air in the colon, which is decompressed, and the colorectal anastomosis appears patent, without evidence of leak. What is the next most appropriate step in management? A) Observation B) Replace the nasogastric tube with a long intestinal tube C) Exploratory laparotomy D) Colonoscopic decompression E) Diagnostic laparoscopy

To understand that the appropriate management for early postoperative bowel obstruction is conservative management/observation Answer A is correct Early postoperative obstruction is caused by adhesions in about 90% of patients. When there are no signs of toxicity and no acute abdominal signs, such obstruction can usually be managed safely with nasogastric decompression. As many as 87% of patients respond to nasogastric suction within 2 weeks. About 70% of the patients who respond to nonoperative treatment do so within 1 week, and an additional 25% respond during the following 7 days. If postoperative obstruction does not resolve in the first 2 weeks, it is unlikely to do so with continued nonoperative therapy, and reoperation is probably indicated; about 25% of patients whose postoperative obstruction was initially treated nonoperatively eventually require reoperation. An exception to this guideline arises in patients known to have severe dense adhesions (sometimes referred to as obliterative peritonitis) in response to multiple sequential laparotomies. These patients may have a combination of mechanical obstruction and diffuse small bowel and colonic ileus. The risk of closed-loop obstruction, volvulus, or strangulation in this group of patients is low. Repeat laparotomies and attempts to lyse adhesions may lead to complications, the development of enterocutaneous fistulae, or exacerbation of the adhesions. Often the best approach to managing these patients is observation for prolonged periods (i.e., months). TPN is indicated. The addition of octreotide to the TPN solution may be helpful and may make patients more comfortable. Because the risk of intestinal strangulation in patients with postoperative adhesive obstruction is extremely low (< 1%), one can generally treat these patients nonoperatively for longer periods. In fact, the conservative approach is often the wise one: reoperation may do more harm than good (e.g., by causing enterotomies and inducing denser adhesions). The traditional indications for operation in patients with early postoperative obstruction include (1) deteriorating clinical status, (2) worsening obstructive symptoms, and (3) failure to respond to nonoperative management within 2 weeks. With the rising cost of hospitalization, it might, in fact, be more cost-effective to reoperate on patients who have persistent obstruction after 7 days. This speculation would have to be tested by a well-organized cost-benefit study conducted in a prospective fashion.

You perform a small bowel resection and primary anastomosis for a small bowel tumor in a 44-year-old man. There is no evidence of metastasis on preoperative imaging or intra-abdominal exploration. Pathology evaluation of the tumor reveals a T3N1 adenocarcinoma of the ileum. What is the stage of this tumor, based on the American Joint Committee on Cancer (AJCC) staging system? A) Stage 0 B) Stage I C) Stage II D) Stage III E) Stage IV

To understand the AJCC staging system for small bowel carcinoma Answer D is correct The AJCC staging system for small bowel carcinoma depends on the tumor (T), the presence of lymph node spread (N), and the presence of distant metastases (M). Lesions that show evidence of carcinoma in situ are classified as stage 0. If there is no evidence of lymph node or distant metastases, T1 and T2 tumors are stage I. T3 and T4 lesions are considered stage II as long as there is no evidence of spread to lymph nodes or distant sites. When there are positive lymph nodes but no distant metastases, the tumor is considered stage III, regardless of the T stage. Evidence of distant metastases makes any small bowel tumor stage IV. The 5-year survival rate continues to be low (24 to 37%). Significant predictors of good overall survival include complete (R0) resection and low AJCC tumor stage.

A 56-year-old male with no other significant past medical history returns to the office after undergoing a right colectomy without incident. His pathology report states that the specimen had negative margins, but the tumor was found to invade through the muscularis propria with evidence of regional lymph node metastases in two of 17 lymph nodes. He has no known distant metastases. What is his staging according to the American Joint Committee on Cancer (AJCC) staging of CRC? A) Stage 0 B) Stage I C) Stage II D) Stage III E) Stage IV

To understand the TNM staging system for colorectal cancer Answer D is correct Accurate staging of CRC is extremely important for determining patient prognosis and assessing the need for adjuvant therapy. Traditionally, staging of CRC has been based on modifications of the Dukes classification, which was initially developed as a prognostic tool for rectal cancer in the 1930s. Since this classification was first implemented, it has undergone multiple modifications, of which the most widely used is the modified Astler-Coller system, initially introduced in the 1950s. Currently, the TNM classification, developed by the AJCC and the International Union against Cancer (UICC), is the preferred staging system. This system takes into account the depth of penetration into the bowel wall (T), the presence and number of involved mesenteric nodes (N), and the presence of distant metastases (M).

An elderly woman presents to the emergency department with hematemesis. After stabilization, an EGD is performed, which shows a gastric ulcer. Which of the following is not true regarding the classification of gastric ulcers? A) Type I gastric ulcers are associated with high acid secretion B) Type II gastric ulcers occur in synchrony with duodenal ulcers C) Type III gastric ulcers occur in a prepyloric location D) Type IV gastric ulcers are located near the gastroesophageal junction E) Type V gastric ulcers are diffuse and can be related to medications

To understand the classification of gastric ulcers Answer A is correct. Gastric ulcers are classified according to their location and to the role (if any) that gastric acid hypersecretion plays in their development. Type I ulcers are located on the lesser curvature and are not associated with acid secretion. Type II ulcers are associated with high acid secretion and are located on the lesser curvature, occurring in synchrony with duodenal ulcers. Type III ulcers are also associated with acid hypersecretion but occur in the prepyloric region. Type IV ulcers are not associated with acid secretion and are located in the cardia near the esophagogastric junction. Type V ulcers are diffuse and are related to the use of medications (e.g., nonsteroidal antiinflammatory drugs [NSAIDs]).

A 38-year-old man presents to you with a chief complaint of hemorrhoids. On examination, you find hemorrhoids that are proximal to the dentate line and prolapse but spontaneously reduce. What is the most appropriate classification for these hemorrhoids? A) Grade 1 B) Grade 2 C) Grade 3 D) Grade 4 E) External

To understand the classification of internal hemorrhoids . Answer B is correct Internal hemorrhoidal disease is demonstrated by two main symptoms: painless bleeding and protrusion. Pain is rarely associated with internal hemorrhoids because they originate above the dentate line in insensate rectal mucosa. The most popular etiologic theory states that hemorrhoids result from chronic straining at defecation (upright posture and heavy lifting may also contribute). This straining not only causes hemorrhoidal engorgement but also creates forces that decrease the fixation between the hemorrhoids and the anal muscular wall. Continued straining causes further engorgement and bleeding, as well as hemorrhoidal prolapse. Internal hemorrhoids are categorized into four grades based on clinical findings and symptoms: 1, bleeding without prolapse; 2, prolapse that spontaneously reduces; 3, prolapse requiring manual reduction; and 4, irreducible prolapse.

A 55-year-old man with chronic peptic ulcer disease develops a near-complete gastric outlet obstruction attributable to scarring. An antrectomy with Billroth II anastomosis is performed. Which of the following is not a potential complication of this procedure? A) Duodenal stump leak B) Delayed gastric emptying C) Afferent limb syndrome D) Alkaline reflux gastritis E) All of the above are potential complications

To understand the complications of antrectomy Answer E is correct Leakage from the duodenal stump is a potentially devastating complication that necessitates prompt reoperation, washout, and drainage. The diagnosis of duodenal stump leakage is confirmed by aspirating bilious fluid from a right upper quadrant fluid collection or by performing a technetium 99m-labeled hepatoiminodiacetic acid scan. Duodenal leaks are rarely closed primarily. Duodenostomy may be indicated to further decompress the afferent limb and prevent continuous leakage. The goals are to create a controlled fistula to the skin and to prevent the accumulation of biliary fluid in the abdomen. Delayed gastric emptying after gastrojejunostomy may occur and is generally managed conservatively. On rare occasions, reoperation is required for delayed anastomotic function. Afferent limb obstruction may occur as a consequence of adhesions, internal herniation, volvulus, or a kink at the angle formed with the gastric remnant. Obstruction to outflow of the afferent limb creates a closed-loop obstruction, with persistent secretion of bile and pancreatic fluids into the loop. Such obstruction often presents as recurrent pancreatitis. The diagnosis is facilitated by CT scanning; if it is confirmed, prompt exploration is mandated. Correction of the obstruction may necessitate conversion to a Roux-en-Y reconstruction, shortening of the afferent limb, or a side-to-side enteroenterostomy with the efferent limb. Alkaline reflux gastritis is one of the most common long-term complications of gastrectomy, developing in 5 and 15% of patients after gastric surgery. This complication is most frequently associated with Billroth II reconstructions. Although reflux is common, symptoms (e.g., epigastric pain, nausea, and bilious emesis) are relatively rare. Medical management is generally ineffective. If surgery is required, conversion of the Billroth II reconstruction to a Roux-en-Y reconstruction is indicated. In those patients who have a Roux-en-Y rather than a Billroth II reconstruction, the preferred treatment is to divert alkaline contents to a location 45 to 60 cm beyond the gastric remn

55-year-old man with chronic peptic ulcer disease develops a near-complete gastric outlet obstruction attributable to scarring. An antrectomy with Billroth II anastomosis is performed. Which of the following is not a potential complication of this procedure? A) Duodenal stump leak B) Delayed gastric emptying C) Afferent limb syndrome D) Alkaline reflux gastritis E) All of the above are potential complications

To understand the complications of antrectomy Answer E is correct Leakage from the duodenal stump is a potentially devastating complication that necessitates prompt reoperation, washout, and drainage. The diagnosis of duodenal stump leakage is confirmed by aspirating bilious fluid from a right upper quadrant fluid collection or by performing a technetium 99m—labeled hepatoiminodiacetic acid scan. Duodenal leaks can rarely be closed primarily. Duodenostomy may be indicated to further decompress the afferent limb and prevent continuous leakage. The goals are to create a controlled fistula to the skin and to prevent the accumulation of biliary fluid in the abdomen. Delayed gastric emptying after gastrojejunostomy may occur and is generally managed conservatively. On rare occasions, reoperation is required for delayed anastomotic function. Afferent limb obstruction may occur as a consequence of adhesions, internal herniation, volvulus, or a kink at the angle formed with the gastric remnant. Obstruction to outflow of the afferent limb creates a closed-loop obstruction, with persistent secretion of bile and pancreatic fluids into the loop. Such obstruction often presents as recurrent pancreatitis. The diagnosis is facilitated by computed tomograpy (CT); if it is confirmed, prompt exploration is mandated. Correction of the obstruction may necessitate conversion to a Roux-en-Y reconstruction, shortening of the afferent limb, or a side-to-side enteroenterostomy with the efferent limb. Alkaline reflux gastritis is one of the most common long-term complications of gastrectomy, developing in 5 and 15% of patients after gastric surgery. This complication is most frequently associated with Billroth II reconstructions. Although reflux is common, symptoms (e.g., epigastric pain, nausea, and bilious emesis) are relatively rare. Medical management is generally ineffective. If surgery is required, conversion of the Billroth II reconstruction to a Roux-en-Y reconstruction is indicated. In those patients who have a Roux-en-Y rather than a Billroth II reconstruction, the preferred treatment is to divert alkaline contents to a location 45 to 60 cm beyond the gastric remnant.

One year ago, a 62-year-old woman underwent a vagotomy and pyloroplasty for a bleeding duodenal ulcer. Which of the following is not a complication of this procedure? A) Dumping syndrome B) Afferent loop syndrome C) Diarrhea D) Bile reflux gastritis E) Poor gastric emptying

To understand the complications of vagotomy Answer B is correct Vagotomy with pyloroplasty is a surgical option for duodenal ulcers but does have a number of potential complications. These include dumping syndrome, diarrhea, bile reflux gastritis, and poor gastric emptying. Many are thought to be related to the vagotomy, but the exact etiology is unclear. Afferent loop syndrome does not occur after vagotomy and pyloroplasty as no afferent loop is created, but it can occur after antrectomy with gastrojejunostomy (Billroth II).

One year ago, a 62-year-old woman underwent a vagotomy and pyloroplasty for a bleeding duodenal ulcer. Which of the following is not a complication of this procedure? A) Dumping syndrome B) Afferent loop syndrome C) Diarrhea D) Bile reflux gastritis E) Poor gastric emptying

To understand the complications of vagotomy Answer B is correct Vagotomy with pyloroplasty is a surgical option for duodenal ulcers but does have a number of potential complications. These include dumping syndrome, diarrhea, bile reflux gastritis, and poor gastric emptying. Many are thought to be related to the vagotomy, but the exact etiology is unclear. Afferent loop syndrome does not occur after vagotomy and pyloroplasty as no afferent loop is created, but it can occur after antrectomy with gastrojejunostomy (Billroth II).

Which of the following factors do not affect the type of operation offered to a patient for rectal prolapse? Please choose the single most appropriate answer to the question A) A patient's age B) A patient's physical condition C) The amount of rectum that is prolapsed D) A previous history of hemorrhoids E) The degree of fecal incontinence

To understand the factors involved in the decision-making process for selecting the appropriate rectal prolapse procedure Answer D is correct The choice of operation is determined by the patient's age, sex, level of operative risk, associated pelvic floor defects, degree of incontinence, and history of constipation, as well as by the surgeon's experience. The goal is to correct the greatest number of anatomic problems (including the prolapse and any associated functional disorders) safely and efficiently while minimizing both perioperative morbidity and postoperative recurrence. The best way to think of prolapse operations is to divide them into broad categories of abdominal and perineal. The balance that must be discussed with the patient is the balance between immediate safety and long-term results. The abdominal approaches, including rectopexy, resection rectopexy, and other choices, whether by laparotomy or laparoscopy, tend to have lower recurrence rates and better functional outcomes than do the perineal methods. Conversely, the perineal approaches tend to have less intraoperative and postoperative morbidity and mortality but have higher rates of recurrence and inferior functional outcomes compared with the abdominal approaches. Therefore, major considerations to selection of the operation are the age and overall physical condition of the patient. In general, young, healthy patients are far better served by an abdominal procedure as they can better tolerate any potential intraoperative or postoperative morbidity and have a longer life expectancy. Therefore, the best possible function over the longest possible period of time is of paramount importance. The preferred operation then is a laparoscopically assisted resection rectopexy to optimize function, decrease recurrence potential, and minimize incisional length. Elderly, frail patients may best be served by a perineal approach to limit intra-abdominal trauma. The perineal procedure preferred is the perineal rectosigmoidectomy. If only a minor amount of prolapse can be delivered, then a Delorme procedure is performed. In our experience, the Delorme procedure has a higher recurrence rate and far less improvement in continence than does the perineal rectosigmoidectomy. Therefore, the Delorme procedure is infrequently undertaken.

A 23-year-old woman with Crohn disease since age 17 is now 3 months pregnant. In a pregnant patient with Crohn disease, all of the following statements are true except A) Metronidazole is the preferred antibiotic to treat disease flares B) Pregnancy does not exacerbate Crohn disease C) Preferred imaging modalities include ultrasonography and magnetic resonance imaging D) Cesarean section is advisable if there is a risk of obstetrically related sphincter trauma E) Prednisone is safe for use

To understand the impact pregnancy may have on the management of patients with Crohn disease Answer A is correct There is no evidence that pregnancy exacerbates Crohn disease; however, there are some specific concerns that apply to pregnant patients with this condition. Because patients with Crohn disease often have bowel movements that are more liquid, they have a particular need for a well-functioning anal sphincter. If there is any chance of an obstetrics-related injury (e.g., from a large baby in a primagravida or from a breech presentation), a cesarean section is advisable to minimize the risk of sphincter trauma. The same is true in the presence of severe perianal Crohn disease. During pregnancy, prednisone and 5-ASA medications are safe, whereas drugs such as metronidazole are not. If imaging studies are needed, magnetic resonance imaging and ultrasonography are the modalities of choice

A 23-year-old female is admitted to the hospital with fever and abdominal pain. Abdominopelvic CT and SBFT reveal thickening of the wall of the distal ileum with narrowing of the ileal lumen. A presumptive diagnosis of Crohn disease is made, and you are asked to see her in consultation. Which of the following is not an indication for surgery in a patient with Crohn disease? A) Abscess that cannot be treated by CT-guided drainage or antibiotics B) Symptomatic fistula C) To prevent disease recurrence later in life D) Bowel obstruction E) Lack of response to medical therapy

To understand the indications for surgery in patients with Crohn disease Answer C is correct In patients with Crohn disease, the entire gastrointestinal tract is susceptible, from the mouth to the anus. Treatment for Crohn disease requires a combination of medical and surgical therapies over a patient's lifetime. Surgery does not prevent disease recurrence or provide cure but is instead reserved for certain sequelae of the disease that cannot be treated medically. These include lack of response to medical therapy, obstruction, symptomatic fistulas, abscess formation that cannot be managed nonoperatively, cancer or dysplasia, and failure to grow in pediatric patients.

A 32-year-old man with a body mass index (BMI) of 45.7 kg/m2 has elected to undergo a laparoscopic GBP. Which of the following is true regarding intraoperative technique for a laparoscopic GBP? A) The Roux limb should always be placed in an antecolic position B) A 200 cm Roux limb is typically created C) When creating the gastric pouch, the entire lesser curvature should be dissected and the entire lesser omentum divided D) When creating the Roux limb, the jejunum should first be divided 5 cm distal to the ligament of Treitz E) A stapled gastrojejunostomy can be performed with either a linear or circular stapler

To understand the intraoperative technique for a laparoscopic GBP Answer E is correct. There has been some controversy regarding the relative merits and deficiencies of antecolic and retrocolic techniques for passage of the Roux limb. The original open GBP often made use of a retrocolic, antegastric approach. The subsequent evolution of gastric transection methods led to a retrocolic, retrogastric approach to limb passage, which decreased the distance the limb had to travel to reach the proximal stomach and reduced the tension placed on the gastrojejunal anastomosis. Eventually, the antecolic, antegastric technique was introduced; this approach became popular with laparoscopic surgeons because it usually made the procedure faster to perform. Both techniques continue to be employed today, and the current evidence is not sufficient to support a blanket recommendation favoring one over the other. If undue tension is evident when the Roux limb is passed via the antecolic pathway, however, it is advisable to switch to the shorter route available via the retrocolic, retrogastric pathway. In general, the Roux limb in a laparoscopic GBP is measured to a length of 45 to 60 cm, whereas in open GBP, the Roux limb should be significantly longer—up to 150 cm—if the long-limb modification is being performed. To begin creation of the Roux limb, the jejunum must first be divided. This is typically done 50 cm distal to the ligament of Treitz. Division 5 cm distal to the ligament of Treitz would make creation of the jejunojejunostomy unnecessarily difficult. To create the gastric pouch, the mesentery of the lesser curvature is transected with a cautery device or stapler to provide hemostasis. Additional dissection along the lesser curvature is not recommended, because it may increase the devascularization of the pouch. The gastrojejunostomy can be performed with either a linear or circular stapler.

A 48-year-old woman with long-standing jejunoileal Crohn disease has had three previous small bowel resections. She now presents with symptoms of partial small bowel obstruction. Small bowel follow-through (SBFT) reveals two discrete strictures in the midileum. Which of the following is true regarding strictures in patients with Crohn disease? A) A Heineke-Mikulicz strictureplasty is ideal for short strictures to 5 to 7 cm B) Multiple strictureplasties should not be performed during the same operation C) There is no risk for adenocarcinoma at the site of a small bowel stricture D) A Finney strictureplasty is performed in an end-to-side fashion E) In a Heineke-Mikulicz strictureplasty, the enterotomy is opened horizontally and closed longitudinally

To understand the management of strictures in jejunoileal Crohn disease Answer A is correct The major advance in the surgical treatment of Crohn disease over the past quarter-century has been the technique of small bowel strictureplasty. Currently, the two most prevalent strictureplasty techniques are Heineke-Mikulicz strictureplasty and Finney strictureplasty. The former is best suited for strictures up to 5 to 7 cm long and the latter for strictures up to 10 to 15 cm long. A Heineke-Mikulicz strictureplasty is performed by creating the enterotomy longitudinally and then closing the enterotomy horizontally. A Finney strictureplasty is performed by orienting the bowel in an inverted U formation and then performing a side-to-side anastomosis. Multiple enterotomies can be performed in the same operation, with the goal being to preserve intestinal length. There is a risk of adenocarcinoma developing at the site of a Crohn stricture. As a result, many surgeons advocate biopsying the active ulcer on the mesenteric side of the bowel at the time of strictureplasty

A 48-year-old woman with long-standing jejunoileal Crohn disease has had three prior small bowel resections. She now presents with symptoms of partial small bowel obstruction. Small bowel follow-through (SBFT) reveals two discrete strictures in the midileum. Which of the following is true regarding strictures in patients with Crohn disease? A) A Heineke-Mikulicz strictureplasty is ideal for short strictures to 5 to 7 cm B) Multiple strictureplasties should not be performed during the same operation C) There is no risk for adenocarcinoma at the site of a small bowel stricture D) A Finney strictureplasty is performed in an end-to-side fashion E) In a Heineke-Mikulicz strictureplasty, the enterotomy is opened horizontally and closed longitudinally

To understand the management of strictures in jejunoileal Crohn disease Answer A is correct. The major advance in the surgical treatment of Crohn disease over the past quarter-century has been the technique of small bowel strictureplasty. Currently, the two most prevalent strictureplasty techniques are Heineke-Mikulicz strictureplasty and Finney strictureplasty. The former is best suited for strictures up to 5 to 7 cm long and the latter for strictures up to 10 to 15 cm long. A Heineke-Mikulicz strictureplasty is performed by creating the enterotomy longitudinally and then closing the enterotomy horizontally. A Finney strictureplasty is performed by orienting the bowel in an inverted U formation and then performing a side-to-side anastomosis. Multiple enterotomies can be performed in the same operation, with the goal being to preserve intestinal length. There is a risk of adenocarcinoma developing at the site of a Crohn stricture. As a result, many surgeons advocate biopsying the active ulcer on the mesenteric side of the bowel at the time of strictureplasty.

A 52-year-old woman with anemia had a normal upper endoscopy and colonoscopy. Capsule endoscopy revealed a mass in the midjejunum. Which of the following tumors is not a primary malignancy of the small intestine? A) Lymphoma B) Leukemia C) Gastrointestinal stromal tumor (GIST) D) Adenocarcinoma E) Carcinoid

To understand the most common malignancies of the small bowel Answer B is correct Malignant tumors of the small intestine are rare, accounting for fewer than 5% of all GI tract malignancies. In the United States, only a few thousand new cases of small bowel cancer are reported each year. The majority of small bowel malignancies are adenocarcinomas, lymphomas, or carcinoid tumors, although GISTs are being noted with increasing frequency in the small intestine. Leukemias originate in the bone marrow, not in the small intestine.

A 54-year-old man was noted to have an 8 cm gastric mass on a CT scan performed after a motor vehicle accident. The mass is round and circumscribed and appears to arise from the muscle layer of the stomach. He is asymptomatic. EGD and biopsy are consistent with gastrointestinal stromal tumor (GIST). Which of the following is not true regarding GISTs of the stomach? A) The cell of origin is believed to be the interstitial cell of Cajal, an intestinal pacemaker cell B) The diagnosis of GIST is confirmed by immunohistochemical staining for CD34 C) The stomach is the most common site of GISTs D) Tumor size and mitotic rate are the strongest determinants of prognosis E) Extended lymphadenectomy is not necessary during resection of a gastric GIST

To understand the pathogenesis and treatment of gastric GISTs Answer B is correct GIST, although relatively rare in absolute terms, is the most common sarcoma of the GI tract, with approximately 6,000 cases reported each year in the United States alone. The stomach is the most common site of involvement, accounting for 60 to 70% of cases; the small intestine (25%), the rectum (5%), the esophagus (2%), and a variety of other locations account for the remainder. With the advent of immunohistochemistry and electron microscopy, it became clear that GIST has both smooth muscle and neural elements, and the cell of origin is now believed to be the interstitial cell of Cajal, an intestinal pacemaker cell. The diagnosis of GIST is secured by immunohistochemical staining for the tyrosine kinase receptor KIT (CD117), which highlights the presence of interstitial cells of Cajal. More than 95% of GISTs exhibit unequivocal staining for KIT. Approximately two thirds of GISTs also express CD34. Of the prognostic factors examined to date, tumor size and mitotic rate appear to be the most valuable. If the tumor is less than 2 cm in diameter and the mitotic count is lower than five per high-power field, the risk of an aggressive disease course is considered to be very low. The role of surgery in the treatment of a GIST is to resect the tumor with grossly negative margins and an intact pseudocapsule. Lymph node involvement is rare with GISTs; thus, no effort is made to perform extended lymph node dissection. The tumor must be handled with care to prevent intra-abdominal rupture. Formal gastric resection is rarely required: as a rule, it is indicated only for lesions in close proximity to the pylorus or the esophagogastric junction.

A 54-year-old man was noted to have an 8 cm gastric mass on CT scan performed after a motor vehicle accident. The mass is round and circumscribed and appears to arise from the muscle layer of the stomach. He is asymptomatic. EGD and biopsy are consistent with GIST. Which of the following is not true regarding GISTs of the stomach? A) The cell of origin is believed to be the interstitial cell of Cajal, an intestinal pacemaker cell B) The diagnosis of GIST is confirmed by immunohistochemical staining for CD34 C) The stomach is the most common site of GISTs D) Tumor size and mitotic rate are the strongest determinants of prognosis E) Extended lymphadenectomy is not necessary during resection of a gastric GIST

To understand the pathogenesis and treatment of gastric GISTs Answer B is correct GIST, although relatively rare in absolute terms, is the most common sarcoma of the GI tract, with approximately 6,000 cases reported each year in the United States alone. The stomach is the most common site of involvement, accounting for 60 to 70% of cases; the small intestine (25%), the rectum (5%), the esophagus (2%), and a variety of other locations account for the remainder. With the advent of immunohistochemistry and electron microscopy, it became clear that GIST has both smooth muscle and neural elements, and the cell of origin is now believed to be the interstitial cell of Cajal, an intestinal pacemaker cell. The diagnosis of GIST is secured by immunohistochemical staining for the tyrosine kinase receptor KIT (CD117), which highlights the presence of interstitial cells of Cajal. More than 95% of GISTs exhibit unequivocal staining for KIT. Approximately two thirds of GISTs also express CD34. Of the prognostic factors examined to date, tumor size and mitotic rate appear to be the most valuable. If the tumor is less than 2 cm in diameter and the mitotic count is lower than five per high-power field, the risk of an aggressive disease course is considered to be very low.The role of surgery in the treatment of a GIST is to resect the tumor with grossly negative margins and an intact pseudocapsule. Lymph node involvement is rare with GISTs; thus, no effort is made to perform extended lymph node dissection. The tumor must be handled with care to prevent intra-abdominal rupture. Formal gastric resection is rarely required: as a rule, it is indicated only for lesions in close proximity to the pylorus or the esophagogastric junction.

The pathology from the patient in question 1 returns high-grade soft tissue sarcoma, malignant fibrous histiocytoma subtype. Which of the following regarding soft tissue sarcomas is not true? A) Sarcomas may occur anywhere in the body and comprise more than 50 distinct histologic subtypes B) The majority of soft tissue sarcomas occur in the retroperitoneums C) Genetic factors, irradiation, chemical exposure, and lymphedema have all been shown to correlate with evolution of soft tissue sarcomas D) The best indicator of a tumor's biologic aggressiveness and metastatic potential is its grade, regardless of its histologic subtype E) The American Joint Committee on Cancer (AJCC) staging system for soft tissue sarcoma integrates tumor grade, size, depth of tissue invasion, degree of nodal involvement, and presence or absence of metastases

To understand the pathologic classification and staging of soft tissue sarcoma Answer B is correct Soft tissue sarcoma is a collective term for an unusual and diverse group of malignancies that arise from cells of the embryonic mesoderm. Although tissues derived from the mesoderm contain approximately 75% of the cells in the human body, sarcomas represent only 1% of adult tumors and 15% of pediatric tumors. Sarcomas may occur anywhere in the body and comprise more than 50 distinct histologic subtypes. Approximately 43% of soft tissue sarcomas occur in the extremities, 15% in the retroperitoneum, 10% in the trunk, 19% in the viscera, and 13% in other locations. In addition, some sarcomas occur in the GI tract. It has been estimated that in 2007, there will be approximately 9,220 cases of sarcoma in the United States, with 3,560 deaths. Soft tissue sarcoma-related mortality has been quite constant over the years, indicating that relatively little progress has been made in the treatment of most sarcomas. The etiology of soft tissue sarcoma is unclear and somewhat controversial. Genetic factors, irradiation, chemical exposure, and lymphedema have all been shown to have a strong correlation with the evolution of these lesions. Regarding its staging, answers 4 and 5 are true.

While giving an informational talk about bariatric surgery, you are asked by a member of the audience if certain patients are at higher risk for complications during bariatric surgery. Which of the following is not an independent risk factor for mortality after GBP? A) BMI greater than 50 kg/m2 B) Prior pulmonary embolus C) Age greater than 45 years D) Female gender E) Hypertension

To understand the risk factors for mortality after GBP Answer D is correct It is now possible to stratify the mortality risk for patients undergoing GBP by using a scoring system known as the Obesity Surgery Mortality Risk Score (OS-MRS), which includes five independent variables that can be identified preoperatively: (1) BMI greater than or equal to 50 kg/m2; (2) male gender; (3) hypertension; (4) pulmonary embolus risk (including previous thrombosis, pulmonary embolus, inferior vena cava filter, right-side heart failure, and obesity hypoventilation syndrome); and (5) patient age greater than or equal to 45 years. These factors were associated with a greater 90-day mortality in a prospective study of 2,075 patients who underwent GBP at a single institution, which was the basis for the initial proposal of this scoring system. The OS-MRS was subsequently validated in a multicenter study involving four institutions and 4,431 patients. With the presence of each variable equal to 1 point, each patient's potential score ranged from 0 to 5. Patients with a score of 0 or 1 had a low mortality risk (group A; mortality, 0.2%); those with a score of 2 or 3 had an intermediate mortality risk (group B; mortality, 1.1%); and those with a score of 4 or 5 had a high mortality risk (group C; mortality, 2.4%). These findings suggest that the OS-MRS is a valuable tool that can be effectively used to stratify risk and facilitate surgical decision making and patient discussion regarding bariatric surgery

While giving an informational talk about bariatric surgery, you are asked by a member of the audience if certain patients are at higher risk for complications during bariatric surgery. Which of the following is not an independent risk factor for mortality after GBP? A) BMI greater than 50 kg/m2 B) Prior pulmonary embolus C) Age greater than 45 years D) Female gender E) Hypertension

To understand the risk factors for mortality after GBP Answer D is correct It is now possible to stratify the mortality risk for patients undergoing GBP by using a scoring system known as the Obesity Surgery Mortality Risk Score (OS-MRS), which includes five independent variables that can be identified preoperatively: (1) BMI greater than or equal to 50 kg/m2, (2) male gender, (3) hypertension, (4) pulmonary embolus risk (including previous thrombosis, pulmonary embolus, inferior vena cava filter, right-side heart failure, and obesity hypoventilation syndrome), and (5) patient age greater than or equal to 45 years. These factors were associated with a greater than 90-day mortality in a prospective study of 2,075 patients who underwent GBP at a single institution, which was the basis for the initial proposal of this scoring system. The OS-MRS was subsequently validated in a multicenter study involving four institutions and 4,431 patients. With the presence of each variable equal to 1 point, each patient's potential score ranged from 0 to 5. Patients with a score of 0 or 1 had a low mortality risk (group A; mortality, 0.2%); those with a score of 2 or 3 had an intermediate mortality risk (group B; mortality, 1.1%); and those with a score of 4 or 5 had a high mortality risk (group C; mortality, 2.4%). These findings suggest that the OS-MRS is a valuable tool that can be effectively used to stratify risk and facilitate surgical decision making and patient discussion regarding bariatric surgery.

21-year-old man is diagnosed with Crohn disease of the terminal ileum. Which of the following is true regarding medical and surgical management of his disease? A) If surgical resection is needed, laparoscopy should not be attempted because of the high risk of complications B) When performing resection for Crohn disease, a 2 cm margin is recommended both proximally and distally C) Surgical therapy should always be performed first, followed by medical treatment to prevent recurrence D) Antibiotics, antiinflammatories, and immunosuppressives are potential medical therapies for Crohn disease E) Ninety percent of patients who undergo surgery for Crohn disease will require reoperation within 10 years

To understand the roles of medical and surgical management in the treatment of Crohn disease Answer D is correct The management of Crohn disease requires a combination of medical and surgical therapies. Generally speaking, except in the case of a free perforation, cancer, or dysplasia, one should not operate on a patient with Crohn disease without first attempting medical therapy. With the dramatically improved medical treatment options currently available, surgery can be avoided in many cases. This is often a desirable result, given the known risk of disease recurrence after surgical treatment of Crohn disease and the significant associated operative morbidity. In one single-center study, the reoperation rate for Crohn disease was 34% at 10 years. Medical therapies consist of probiotics, antibiotics, antiinflammatories, immunosuppressives, and biologics. If surgical therapy is needed, it can often be performed laparoscopically. Laparoscopic surgical techniques have gained acceptance in the treatment of Crohn disease. In performing a laparoscopic operation for Crohn disease, it is essential to adhere to the same technical standards that apply to corresponding open procedures. When resecting a segment of intestine for Crohn disease, conservative surgery is now the rule: only gross macroscopic disease is resected into palpably normal margins (in particular, a palpably normal mesenteric border of the bowel).

A 21-year-old man is diagnosed with Crohn disease of the terminal ileum. Which of the following is true regarding medical and surgical management of his disease? A) If surgical resection is needed, laparoscopy should not be attempted because of the high risk of complications B) When performing resection for Crohn disease, a 2 cm margin is recommended both proximally and distally C) Surgical therapy should always be performed first, followed by medical treatment to prevent recurrence D) Antibiotics, antiinflammatories, and immunosuppressives are potential medical therapies for Crohn disease E) Ninety percent of patients who undergo surgery for Crohn disease will require reoperation within 10 years

To understand the roles of medical and surgical management in the treatment of Crohn disease D is correct The management of Crohn disease requires a combination of medical and surgical therapies. Generally speaking, except in the case of a free perforation, cancer, or dysplasia, one should not operate on a patient with Crohn disease without first attempting medical therapy. With the dramatically improved medical treatment options currently available, surgery can be avoided in many cases. This is often a desirable result, given the known risk of disease recurrence after surgical treatment of Crohn disease and the significant associated operative morbidity. In one single-center study, the reoperation rate for Crohn disease was 34% at 10 years. Medical therapies consist of probiotics, antibiotics, antiinflammatories, immunosuppressives, and biologics. If surgical therapy is needed, it can often be performed laparoscopically. Laparoscopic surgical techniques have gained acceptance in the treatment of Crohn disease. In performing a laparoscopic operation for Crohn disease, it is essential to adhere to the same technical standards that apply to corresponding open procedures. When resecting a segment of intestine for Crohn disease, conservative surgery is now the rule: only gross macroscopic disease is resected into palpably normal margins (in particular, a palpably normal mesenteric border of the bowel).

A 71-year-old woman presents to you with a 5 cm adenocarcinoma of the descending colon and liver metastases. With respect to patients with metastatic colorectal cancer (CRC), which of the following is not true? A) In patients with unresectable disease, the best available evidence suggests that there is no difference in survival between patients initially managed surgically and patients initially managed nonoperatively B) In patients with hepatic metastases, there is no difference in survival between patients presenting with synchronous lesions and patients presenting with metachronous lesions C) Surgical resection or diversion is indicated in patients who present with significant bleeding, perforation, or obstruction D) Curative resection may be undertaken in patients with isolated liver or lung metastases E) Combined resection of the primary colon cancer and hepatic metastases in patients with synchronous hepatic metastases has been shown not to compromise cancer survival

To understand the surgical management of patients with stage IV colorectal cancer Answer B is correct As many as 20% of CRC patients have metastatic disease at the time of initial presentation. The need for surgical intervention in this group of patients is not well defined. Clearly, surgical resection or diversion is indicated in patients who present with significant bleeding, perforation, or obstruction. In asymptomatic patients with unresectable metastatic disease, the role of surgical resection of the primary lesion remains controversial. In patients with resectable metastatic disease (e.g., isolated liver or lung metastases), curative resection may be undertaken. In a retrospective review of patients presenting with unresectable stage IV CRC, there was no difference in survival between those who were initially managed surgically and those who were initially managed nonoperatively. In the surgical group, the morbidity was 30% and the mortality was 5%. Only 9% of the nonoperative patients subsequently required surgical intervention for bowel obstruction. In another retrospective series, patients managed surgically had significantly better overall survival than those managed nonoperatively but had a lesser tumor burden; 29% of the nonoperative patients eventually required surgery for bowel obstruction. When prognostic factors were evaluated in the surgical arm of this series, the only factor associated with improved outcomes was a less than 25% extent of liver involvement. On the basis of these and other studies, asymptomatic patients with unresectable metastatic CRC should be managed selectively: those with limited tumor burdens may benefit from surgical treatment, whereas those with more extensive disease (especially extensive liver involvement) may initially be managed nonoperatively. Management of patients with synchronous resectable isolated liver metastases continues to evolve. Many studies have documented improved survival after liver resection in patients with metastatic disease that is confined to the liver. Patients presenting with synchronous lesions have a worse prognosis than those presenting with metachronous lesions. Many of these patients have been managed with staged resections of the primary cancers and the liver metastases. Several groups have reported that such combined procedures do not substantially increase surgical morbidity and mortality or compromise cancer survival. These combined procedures should be done only in carefully selected patients at specialized centers with significant experience in resection of both CRC and liver tumors.

56-year-old woman has a biopsy-proven adenocarcinoma of the small bowel. Which of the following is not true regarding the treatment for small bowel adenocarcinoma? A) Jejunal and ileal adenocarcinomas are treated with segmental resection and wide mesenteric resection B) Periampullary adenocarcinomas can be resected by enucleation C) Sleeve duodenectomy with duodenojejunostomy is employed for distal duodenal tumors D) Surgical resection is the primary therapy for small bowel adnocarcinoma E) If the tumor invades a contiguous organ, an en bloc resection of the involved organ should be performed

To understand the surgical management of small bowel adenocarcinomas Answer B is correct Aggressive surgical resection remains the cornerstone of therapy for adenocarcinoma of the small intestine. For periampullary lesions, pancreaticoduodenectomy is typically required to achieve a margin-negative resection; enucleation is not sufficient. For lesions in the distal duodenum, a segmental sleeve resection with a duodenojejunostomy is appropriate. For lesions in the jejunum or the ileum, segmental resection may be performed with a wide mesenteric resection to encompass potentially involved regional lymph nodes. Contiguous organs are resected en bloc as necessary.

A 56-year-old woman has a biopsy-proven adenocarcinoma of the small bowel. Which of the following is not true regarding the treatment for small bowel adenocarcinoma? A) Jejunal and ileal adenocarcinomas are treated with segmental resection and wide mesenteric resection B) Periampullary adenocarcinomas can be resected by enucleation C) Sleeve duodenectomy with duodenojejunostomy is employed for distal duodenal tumors D) Surgical resection is the primary therapy for small bowel adnocarcinoma E) If the tumor invades a contiguous organ, an en bloc resection of the involved organ should be performed

To understand the surgical management of small bowel adenocarcinomas Answer B is correct Aggressive surgical resection remains the cornerstone of therapy for adenocarcinoma of the small intestine. For periampullary lesions, pancreaticoduodenectomy is typically required to achieve a margin-negative resection; enucleation is not sufficient. For lesions in the distal duodenum, a segmental sleeve resection with a duodenojejunostomy is appropriate. For lesions in the jejunum or the ileum, segmental resection may be performed with a wide mesenteric resection to encompass potentially involved regional lymph nodes. Contiguous organs are resected en bloc as necessary.

A 52-year-old man has a bleeding duodenal ulcer that cannot be controlled endoscopically, and you plan to oversew the vessel and perform a pyloroplasty with vagotomy. Which of the following is true? A) A Kocher maneuver should be performed to enable an incision on the lateral aspect of the duodenum B) Figure-of-eight sutures should be placed superior, inferior, and medial to the ulcer bed C) The pyloroplasty incision should be oriented transversely D) The left (anterior) vagus nerve is best identified by retracting the esophagus anteriorly and to the left E) The vagotomy is performed by cauterizing the anterior and posterior vagus nerves

To understand the technique of vagotomy and pyloroplasty Answer B is correct The steps of a vagotomy and pyloroplasty include exposure and division of the pylorus, ligation of the bleeding vessel, formation of the pyloroplasty, and vagotomy. A Kocher maneuver is needed to mobilize the pylorus and duodenum and to enable tension-free pyloroplasty. The pyloric incision is performed anteriorly through the pylorus, not laterally. The bleeding duodenal vessel is controlled with figure-of-eight sutures superiorly and inferiorly to control the gastroduodenal artery and medially to control the transverse pancreatic branch. The pyloroplasty incision should be oriented longitudinally, but it is then closed transversely to prevent narrowing of the duodenum. To expose the left (anterior) vagus nerve, the esophagus should be retracted posteriorly and to the right. Typically, the vagus nerves are isolated, clips are placed proximally and distally, and at least 2 cm of nerve is excised.

52-year-old man has a bleeding duodenal ulcer that cannot be controlled endoscopically, and you plan to oversew the vessel and perform a pyloroplasty with vagotomy. Which of the following is true? A) A Kocher maneuver should be performed to enable an incision on the lateral aspect of the duodenum B) Figure-of-eight sutures should be placed superior, inferior, and medial to the ulcer bed C) The pyloroplasty incision should be oriented transversely D) The left (anterior) vagus nerve is best identified by retracting the esophagus anteriorly and to the left E) The vagotomy is performed by cauterizing the anterior and posterior vagus nerves

To understand the technique of vagotomy and pyloroplasty Answer B is correct. The steps of a vagotomy and pyloroplasty include exposure and division of the pylorus, ligation of the bleeding vessel, formation of the pyloroplasty, and vagotomy. A Kocher maneuver is needed to mobilize the pylorus and duodenum and to enable tension-free pyloroplasty. The pyloric incision is performed anteriorly through the pylorus, not laterally. The bleeding duodenal vessel is controlled with figure-of-eight sutures superiorly and inferiorly to control the gastroduodenal artery and medially to control the transverse pancreatic branch. The pyloroplasty incision should be oriented longitudinally, but it is then closed transversely to prevent narrowing of the duodenum. To expose the left (anterior) vagus nerve, the esophagus should be retracted posteriorly and to the right. Typically, the vagus nerves are isolated, clips are placed proximally and distally, and at least 2 cm of nerve is excised.

At his follow-up visit, a 53-year-old man who underwent right colectomy for colon cancer asks you his risk of recurrence. In patients with CRC who develop recurrence, what percentage of all recurrences will occur within 4 years? A) 15% B) 30% C) 45% D) 60% E) 90%

To understand the timeframe for recurrent colorectal cancer Answer E is correct. Between 40 and 50% of patients experience relapses after potentially curative resection of CRC. Detection and treatment of recurrent disease before symptom development may improve survival. The time to recurrence is critical in that as many as 80% of recurrences occur within the first 2 years and as many as 90% within the first 4 years. Even when recurrent CRC is detected, only a small percentage of patients are candidates for reoperation, and resection in these patients may not improve overall survival. Systemic therapy may improve survival in some patients who have unresectable recurrent lesions.

A 66-year-old man is postoperative day 1 following repair of a ruptured abdominal aortic aneurysm for which he received 25 units of packed red blood cells, 20 units of FFP, an 8 liters of crystalloid. For the past 6 hours, he has made 0 cc of urine, his vasopressor requirement has increased, and he has increasing difficulty with ventilation with increased peak airway pressures. On examination, his abdomen is distended and taught and his extremities are cool. What is the next best step in the management strategy for this patient? A) Diuresis B) CT scan of the abdomen and pelvis C) Rigid sigmoidoscopy D) Laparotomy E) Further resuscitation with crystalloid

Key Concept/Objective: To demonstrate the understanding of abdominal compartment syndrome Answer D is correct The patient in this scenario has abdominal compartment syndrome (ACS) as a result of massive transfusion and the intravenous fluid load that he received during operation. ACS is defined as intra-abdominal hypertension greater than 25 mm Hg in conjunction with dysfunction of one or more systems that include pulmonary, renal, or cardiac. Intra-abdominal pressure can be determined indirectly by measuring bladder pressure. If criteria for ACS is met, the treatment is prompt abdominal decompression with laparotomy and temporary abdominal closure.

Midline laparotomy is one of the most common incisions used in abdominal and trauma surgery. Which of the following statements concerning this incision is not true? A) The advantages of a midline laparotomy are that it allows rapid and easy access to the abdominal cavity, with good exposure of the majority of the intra-abdominal organs and structures B) The disadvantage of a midline laparotomy is that it may not provide adequate exposure of injuries in the deep recesses of the upper quadrants C) When closing a midline laparotomy incision, fascial closure may be achieved with either continuous or interrupted suture, with higher rates of dehiscence in running closures D) In cases where there is specific concern of fascial dehiscence (e.g., malnourished or chronic steroid use), large monofilament sutures may be placed at intervals within the standard closure to serve as retention sutures E) If intraperitoneal contamination has occurred, either primary delayed closure should be performed or the wound should be packed and left to heal by secondary intention

Key Concept/Objective: To demonstrate the understanding of incision and closure of a midline abdominal wound Answer C is correct Midline laparotomy provides easy access to most of the abdominal organs, but the deep recesses of the upper quadrants can be difficult to expose through the midline. Fascial closure of the midline abdominal incision can be performed with continuous absorbable or nonabsorbable monofilament suture, though it may also be accomplished with interrupted sutures. The rate of fascial dehiscence is essentially the same for the two techniques; however, the extent of dehiscence is usually more limited when the closure is done with interrupted sutures. The continuous method has the advantage of speed. If intraperitoneal contamination has occurred, the rates of surgical site infection are high if the skin is closed primarily; therefore, either primary delayed closures should be performed or the wound should be packed and left to heal by secondary intention

Upon laparotomy for a gunshot wound to the abdomen a large, expanding, pulsatile zone I retroperitoneal hematoma is encountered at the level of the celiac axis and superior mesenteric artery. To gain control of the abdominal aorta for this patient, which of the following is not true? A) The supraceliac aorta can be exposed by incising the gastrohepatic ligament, and by mobilizing the esophagus to permit identification of the abdominal aorta at the diaphragmatic hiatus B) The supraceliac aorta and its branches can be exposed by means of a right medial visceral rotation C) The left medial visceral rotation will provide access to the left renal artery D) The infrarenal aorta can be approached by retracting the small intestine to the right, the transverse colon cephalad and the descending colon laterally E) If the left renal vein needs to be sacrificed during exposure of the abdominal aorta, it should be ligated more medial than the gonadal vein

Key Concept/Objective: To demonstrate the understanding of operative exposure of the abdominal aorta Answer B is correct The supraceliac aorta can be exposed at the diaphragmatic hiatus and via left medial visceral rotation. The former provides access to control the aorta but is usually inadequate to provide access for definitive repair. The left medial visceral rotation allows exploration of the entire length of the abdominal aorta. The infrarenal aorta can be approached by retracting the small intestine to the right, the transverse colon cephalad and the descending colon laterally. The left renal vein usually limits proximal exposure of the aorta though this approach and can be ligated without consequence so long as the gonadal vein is left intact to drain the kidney.

After suffering a gunshot wound to the right upper quadrant, a 20-year-old male undergoes exploratory laparotomy. A 50% laceration in the mid common bile duct is identified with surrounding spillage of bile. Which of the following is the most appropriate surgical management of this injury? A) Placement of a T-tube in the injury and placement of surgical drains B) Primary repair of the duct C) Primary repair of the duct over a T-tube D) Roux-en-y choledocojejunostomy E) Biologic patch ductoplasty

Key Concept/Objective: To demonstrate the understanding of the management of injury to the extra-hepatic biliary tree Answer D is correct The diagnosis of common bile duct injury is usually made at laparotomy for treatment of associated injuries. Primary repair of common bile duct injury is possible only when the laceration is small and there is no tissue loss. When there is tissue loss or the laceration is larger than 25% to 50% of the diameter of the duct, the best treatment option is a Roux-en-y cholecodojejunostomy

The patient in Question 2 had an unenventful recovery but on postoperative day 3, the output of the surgical drain that was placed in the splenectomy bed increased to 250cc over 24 hours and the quality changed to a viscous gray-brown fluid. He remains afebrile, tolerating a regular diet, and appears otherwise ready for discharge from the hospital. Which of the following is the best next step in management of this patient? A) Discharge home with the drain and close follow-up B) Observation for 24 hours and if no change in condition, discharge home with the drain with close follow-up C) Begin broad spectrum antibiotics and send a culture from the drain effluent D) Make NPO, begin intravenous fluids, and monitor the drain output E) Obtain a CT of the abdomen

Key Concept/Objective: To demonstrate the understanding of the management of postoperative complications following splenectomy Answer E is correct Common complication following splenectomy include subdiaphragmatic abscess in the splenectomy bed and pancreatic leak from the tail of the pancreas that may have been injured during splenectomy. An important step in management is CT imaging to rule out an undrained collection that would require percutaneous drain placement. If an abscess is diagnosed, drainage is performed and the patient is treated with broad spectrum antibiotics. A pancreatic leak can be diagnosed by sending the drain effluent for amylase. If otherwise asymptomatic, and there is no undrained collection, these leaks will most often seal after several days to weeks and require no further treatment. If they fail to do so, the main pancreatic duct may be injured and may require further treatment with ERCP and potentially distal pancreatectomy.

After blunt force trauma to the abdomen, a 24-year-old male is brought from the trauma bay to the operating room. Following splenectomy for a grade IV splenic injury, a retroperitoneal hematoma within Gerota's fascia on the left and extending down the left paracolic gutter is identified. It does not appear to change in size over a 10-minute observation period. What is the most appropriate treatment for this injury? A) No surgical treatment B) Exploration of the hematoma C) Exploration of the hematoma following proximal vascular control D) Placement of laparotomy pads and abdominal re-exploration in 24 to 48 hours E) Left nephrectomy

Key Concept/Objective: To demonstrate the understanding of the management of retroperitoneal hematomas Answer A is correct. Retroperitoneal hematomas are classified on an anatomic basis: zone 1 is the central area, bounded laterally by the kidneys and extending from the diaphragmatic hiatus to the bifurcation of the vena cava and aorta; zone 2 comprises the lateral area of the retroperitoneum from the kidneys laterally to the paracolic gutters; and zone 3 is the pelvic portion of the retroperitoneum. All zone 1 hematomas should be explored regardless of the mechanism of injury: they signal possible aortic, vena caval, duodenal, or pancreatic injury. Zone 2 and 3 hematomas should be explored in cases of penetrating trauma but not, as a rule, in cases of blunt trauma (except in cases of expanding zone 2 hematomas). Before a retroperitoneal hematoma is opened, proximal vascular control should be obtained so that hemorrhage will be minimized once the effect of the tamponade is lost.

Which of the following is not true regarding the operative treatment of liver injury? A) Clamping of the hepatic pedicle (the Pringle maneuver) can be helpful for evaluating grade IV and V hepatic injuries B) Hepatic vascular isolation is attained by executing a Pringle maneuver, clamping the aorta at the diaphragm, and clamping the suprarenal vena cava C) Periodic manual compression with the addition of laparotomy pads is useful in the treatment of complex hepatic injuries to provide time for hepatic resuscitation D) Suturing of the hepatic parenchyma can be useful to quell bleeding from grade III and grade IV liver lacerations E) Hepatotomy with selective ligation of bleeding vessels is useful for the operative treatment for deep penetrating hepatic wounds

Key Concept/Objective: To demonstrate the understanding of the operative treatment of liver injury Answer B is correct In addition to a Pringle maneuver and clamping of the aorta at the level of the diaphragm, hepatic vascular isolation entails clamping of the suprarenal as well as suprahepatic vena cava. The Pringle maneuver allows one to distinguish between bleeding from branches of the hepatic artery or portal vein and hemorrhage form the hepatic veins or retrohepatic vena cava. Period manual compression of the liver and perihepatic packing is capable of controlling hemorrhage from nearly all hepatic injuries. Suturing of the hepatic parenchyma is often employed to control persistently bleeding lacerations less than 3 cm in depth, although it is also an appropriate alternative for deeper lacerations if the patient cannot tolerate further hemorrhage associated with hepatotomy and selective ligation. The latter technique is performed for deep penetrating liver injury by using a finger-fracture technique to extend the length and depth of a laceration or missile tract until the bleeding vessels can be identified and adequately controlled.

A 19-year-old male suffers two gunshot wounds to the abdomen. Hemodyamically unstable in the trauma bay, he is taken to the operating room for laparotomy. Multiple injuries are found and include a shattered bleeding spleen, a large devitalizing enterotomy of the ileum and a large devitalizing injury to the transverse colon. The anesthetist informs you that he has received 12 units of blood, 10 units of plasma, and he remains hypotensive. His INR is 2.2 and his arterial pH is 7.19. What is the most appropriate management strategy for this patient? A) Placement of laparotomy pads, temporary closure of the abdomen with a Bogota bag, and transfer to the ICU B) Splenectomy, placement of laparotomy pads, temporary closure of the abdomen with a Bogota bag, and transfer to the ICU C) Splenectomy, jejunal resection without anastomosis, transverse colon resection without anastomosis, closure of the abdomen with a Bogota bag, and transfer to ICU D) Splenectomy, jejunal resection with anastomosis, transverse colon resection with anastomosis, temporary closure of abdomen with a Bogota bag, and transfer to ICU E) Splenectomy, jejunal resection with anastomosis, transverse colon resection with anastomosis, closure of abdomen, and transfer to ICU

Key Concept/Objective: To demonstrate the understanding of the principles of damage control surgery Answer C is correct If a patient remains unstable during a trauma laparotomy or becomes cold, acidotic, or coagulopathic, an abbreviated or damage-control procedure is indicated. The decision to perform a damage control procedure should be made at an early stage, before the so-called lethal triad (hypothermia, acidosis, and coagulopathy) has time to develop. Major bleeding and contamination should be controlled as best as possible. This may involve stapling of injured bowel without restoring intestinal continuity at the time of the initial operation. Restoring intestinal continuity is both time-consuming and has a high risk for anastomotic leak in the unstable patient. It may also be necessary to perform a rapid, temporary abdominal closure. The patient is then transported to the intensive care unit and is warmed and resuscitated. Return to the operating room in 24 to 48 hours is planned once hemodynamic and metabolic disturbances are reversed.

A 64-year-old woman was a restrained driver in a high-speed motor vehicle collision. After 2 liters of intravenous fluids and 1 unit of packed red blood cells, she is hemodynamically stable in the trauma bay. CT scans of her head, neck, chest, abdomen, and pelvis reveal only a 9 cm subcapsular hematoma of the right lobe of the liver that accounts for approximately 40% of the anterior surface of the liver. Which of the following management strategies should be employed for this patient? A) Admission to a trauma ICU for hemodynamic monitoring, bed rest, serial hematocrit checks, and serial abdominal exams B) Angiogram and embolization of hepatic arterial branch vessels C) Laparotomy and packing D) Laparotomy and exploration of the hematoma E) Laparotomy and anatomic hepatic resection

Key Concept/Objective: To demonstrate the understanding of the treatment of hepatic injury Answer A is correct Hepatic injuries are classified according to a grading system developed by the American Association for the Surgery of Trauma Committee with a grading system that ranges from I to VI with I representing superficial lacerations and small subcapsular hematomas and VI representing lethal avulsion of the liver from the vena cava. Isolated injuries that are not extensive (I to III) often require little or no treatment, however, extensive parenchymal injuries and those involving juxtahepatic veins may require complex maneuvers including hepatic vascular isolation for successful treatment. Grade I and II subscapsular hematomas, which involve less than 50% of the surface of the liver, can be managed nonoperatively if the patient is hemodynamically stable without significant transfusion requirement. If discovered at laparotomy, grade I or II subcapsular hematomas should be left alone if they are not expanding. Expanding and grade III or greater subscapsular hematomas require exploration.

A 36-year-old intoxicated male fell backwards off of a porch. In the trauma bay, his heart rate is 110 and blood pressure is 92/48. He has so far received 2 liters of intravenous fluid. His CT reveals a laceration of the spleen that involves the hilum, 40% devascularizaton of the splenic parenchyma, and a perisplenic hematoma. Which of the following is the most appropriate management strategy for this patient? A) Admission to a trauma ICU for hemodynamic monitoring, bed rest, serial hematocrit checks, and serial abdominal exams B) Angiogram and arterial embolization C) Laparotomy and splenectomy D) Laparotomy and splenorrhaphy E) Laparotomy and placement of lap pads for 24 to 48 hours

Key Concept/Objective: To demonstrate the understanding of the treatment of splenic injury Answer C is correct Splenic injuries are classified according to a grading system developed by the American Association for the Surgery of Trauma Committee with a grading system that ranges from I to V with I and II being small tears and subcapsular hematomas (< 50% surface area) with no active bleeding. Many splenic injuries can be managed nonoperatively. Splenectomy is the usual treatment for hilar injuries (grade IV) and pulverized splenic parenchyma (grade V).

A 59-year-old woman is brought to the ED after being the victim of a mugging. She is hemodynamically stable but confused. On examination, she has a large hematoma over her left frontal skull and linear ecchymoses over the left neck. Which of the following statements is true regarding blunt injuries to the vasculature of the neck? A) Most injuries are the result of direct trauma to the neck B) Traditional angiography should be used as a method of screening for vascular injury C) All high-grade injuries need surgical intervention D) Complications of such injuries include pseudoaneurysm, dissection, and emboli E) All patients with such injures should receive antiplatelet or anticoagulation agents

Key Concept/Objective: To demonstrate understanding of management of blunt injuries to the vasculature of the neck Answer D is correct Blunt injuries to cervical arteries, although relatively rare, can be difficult to diagnose and can lead to devastating complications (e.g., stroke and bleeding). Most such injuries result from stretching of the vessels as a consequence of hyperextension of the neck. Motor vehicle collision is a common cause of this sort of stretch injury. Injury may also be caused by direct trauma to the artery, such as with a seat belt or by fracture fragments, particularly cervical transverse foramen or basilar skull fractures. Arterial stretching leads to endothelial tearing, with subsequent intimal flaps, dissections, or emboli. Four-vessel angiography has historically been the gold standard for imaging the cervical vasculature but is invasive and expensive and has largely been replaced as a screening modality by CT angiography. CT angiography is noninvasive, can be performed relatively rapidly, and can be done in the course of CT scanning of the head and other parts of the body. Improvements in CT technology, particularly the use of multidetector CT scanners, have resulted in sensitivity and specificity rates similar to those of formal angiography. Biffl and colleagues developed a grading scale for blunt cervical arterial injury based on the angiographic appearance of the lesion. On this scale, grade I and II lesions show less than 25% and greater than 25% luminal narrowing, respectively; grade III lesions are pseudoaneurysms; grade IV lesions demonstrate thrombosis; and grade V lesions are transections with extravasation. Grade I to III lesions may progress to a higher grade over time. Treatment of blunt cervical arterial injury reduces morbidity and mortality. Groups in Denver and Memphis have shown a dramatic decrease in stroke rates (54% to 2.6% and 64% to 6.8%, respectively) for carotid artery injuries in treated versus untreated patients. Although treatment of some sort seems to be beneficial, the exact means of treatment is debated. The primary goal of treatment is to decrease embolic events, and the most common treatment is some means of interfering with clotting, which can be challenging in trauma patients with contraindications to anticoagulation, such as intracranial hemorrhage, solid-organ, or pelvic injuries. The risks and benefits of anticoagulation versus simple observation and expectant management need to be weighed. Asymptomatic patients with small intimal flaps or minimal dissection who have no contraindication to antiplatelet agents should be given aspirin. If a contraindication to aspirin exists, the patient should be treated expectantly. More than half of low-grade injuries heal within 2 weeks, and anticoagulation can be stopped after 14 days. If the patient is asymptomatic but has an acute dissection with no contraindications to anticoagulation, then either heparin or antiplatelet agents should be given. Patients with neurologic symptoms need to be anticoagulated.

A 45-year-old man sustained multiple stab wounds to his neck, chest, and abdomen in an altercation. On arrival at the trauma bay, he is hemodynamically unstable. Emergency release blood is called for, and he is being intubated. On examination, he has pulsatile blood spurting from a zone II neck wound when the paramedic removes his hand. In addition, he does not have left-sided breath sounds and his abdomen is tensely distended. Which is the proper management of a left common carotid laceration in this situation? A) Exploration through a supraclavicular incision and ligation of the common carotid artery at its takeoff from the brachiocephalic artery B) Exploration through an incision along the anterior border of the sternocleidomastoid (SCM) muscle and ligation of the common carotid artery C) Exploration through an incision along the anterior border of the SCM muscle and repair of the laceration D) Exploration through an incision along the anterior border of the SCM muscle and repair using a saphenous vein patch angioplasty E) None of the above

Key Concept/Objective: To demonstrate understanding of management of penetrating injuries to the common carotid artery Answer B is correct Management of common or external carotid artery injuries is governed by the extent of the injury and the overall status of the patient. Simple injuries to the external carotid artery should be repaired, whereas complex injuries should be ligated given that the external carotid artery normally has such good collateral flow. Even in cases with complex injuries to the common carotid artery, if the patient has no neurologic deficits, ligation can be done. The internal carotid will be reconstituted with flow from the external carotids. If an injury to the common carotid artery or the internal carotid artery has interrupted flow in the vessel, repair creates a theoretical disadvantage. Interruption of flow may lead to focal brain ischemia and partial disruption of the blood-brain barrier. Sudden restoration of blood flow may cause hemorrhage in the area of the ischemia and worsen the extent of brain injury; an anemic, or white, infarct of the brain may be converted to a hemorrhagic, or red, infarct. Whether this pathophysiology is important after traumatic injury is unclear and controversial. Deciding whether to repair or ligate when flow has been interrupted is often difficult. One approach is to base the decision on the patient's preoperative neurologic status. If there is no neurologic deficit, it is presumed that there are no areas of brain ischemia and that repair is safe. Conversely, a focal neurologic deficit is presumed to be related to ischemia, and in such cases, the risk of worsening the patient's neurologic status with restoration of blood flow is increased. Even though this approach is rational, it is not applicable in cases in which a detailed neurologic examination before surgery is not possible. Furthermore, this approach may be applicable only to patients in coma or with severe neurologic deficits. There are indications that milder neurologic deficits respond favorably to revascularization. Yet another approach is to gauge the appropriateness of repair according to the nature of the injury itself. In this approach, large, complicated injuries requiring involved and lengthy procedures for repair are ligated, whereas simple injuries requiring only simple and quick repairs are repaired. Similarly, repair is not indicated in patients with severe or multiple associated injuries. There is also a difference between the management of injuries of the common carotid artery and the management of injuries of the internal carotid artery. Common carotid injuries are more accessible and easier to repair, and repair is generally associated with a good outcome. Continued antegrade flow in the internal carotid artery is more likely after injury to the common carotid artery than after injury to the internal carotid artery because of the possibility of collateral flow via the external carotid artery. A reasonable way to deal with repair dilemmas is to make the decision on the basis of distal back-bleeding. Interruption of blood flow to the brain is tolerated only for a short time, and restoration of flow is unlikely to be accomplished quickly enough to improve outcome. It is therefore logical to base the decision about revascularization on the state of back-bleeding from the internal carotid artery. If back-bleeding is brisk, the patient is presumed to have good collateral flow, and the chances that there is an area of ischemia are low. Repair rather than ligation is safe in such circumstances. If internal carotid artery back-bleeding is minimal or absent, an ischemic infarct is more likely and restitution of arterial inflow is more dangerous. A corollary to this reasoning is that if back-bleeding is poor, a clot distal to the area of injury may be present, and return of flow with repair may dislodge the clot distally.

While exploring the neck through a left supraclavicular incision for bleeding from the vertebral artery, which of the following structures should not be sacrificed? A) Inferior thyroid artery B) Vertebral vein C) Thoracic duct D) Thyrocervical trunk E) Phrenic nerve

Key Concept/Objective: To demonstrate understanding of management of vertebral artery injury Answer E is correct A supraclavicular incision is made approximately one finger-breadth superior to the clavicle and is extended medially to the midpoint of the sternocleidomastoid insertion and laterally to the juncture of the middle and lateral thirds of the clavicle. The skin, the subcutaneous tissue, and the platysma are incised. The external jugular vein, if in the operating field, is suture-ligated. At a level just deep to the sternocleidomastoid and omohyoid muscles, the carotid sheath is encountered in the medial aspect of the wound. The lateral border of the internal jugular vein is dissected free of adjacent tissue and retracted medially. If the operation is on the left, the thoracic duct may be found in the medial portion of the wound. The thoracic duct is easily injured with retraction and should be divided and ligated if it is obviously in the way. Just lateral to the internal jugular vein at the same depth is the supraclavicular fat pad, which is dissected from the supraclavicular fossa in which it lies. Exposure is further enhanced by dissection and division of the anterior scalene muscle. The phrenic nerve, which is closely applied to the anterior surface of the anterior scalene muscle, is dissected free of the underlying muscle and retracted out of the field with a vessel loop. The anterior scalene muscle is then divided with the electrocautery. During dissection of the supraclavicular fat pad, it may be necessary to divide branches of the thyrocervical and costocervical trunks. The most prominent of these branches is the inferior thyroid artery, which stems from the thyrocervical trunk and courses medially toward the thyroid. After the supraclavicular fat pad has been dissected, the proximal portion of the vertebral artery is reached. The vertebral vein, which usually lies slightly superficial and medial to the vertebral artery, is divided and suture-ligated to provide better exposure. Care should be taken not to retract the vertebral vein too vigorously before suture ligation so that this vessel is not avulsed from the subclavian vein. At the depth of the vertebral artery, the white, cordlike elements of the brachial plexus are often visible in the superolateral aspect of the wound. If possible, traction should not be placed on the brachial plexus, and use of the electrocautery around the plexus should be minimized. After exposure, the proximal vertebral artery is ligated both proximal and distal to the site of injury. No attempts at repairing the injury should be made.

A 60-year-old man is brought to the ED by the emergency medical service after sustaining a stab wound to the left neck within the posterior triangle. At the scene, there was 500 cc of blood loss and a pressure dressing was applied. He is currently hemodynamically stable. On examination, he has a 2 cm laceration posterior to the sternocleidomastoid muscle with subcutaneous emphysema, blood, and air bubbles emanating from the wound. Which of the following is true regarding the management of penetrating esophageal trauma? A) Surgical exploration is sufficient to rule out an injury B) Contrast esophagography is as sensitive as esophagoscopy in diagnosing an injury C) Repair of acute injuries includes surgical débridement, drainage, and primary repair with muscle flap buttressing D) Anastomotic leak is the most common complication of esophageal repair E) All of the above

Key Concept/Objective: To demonstrate understanding of penetrating esophageal injury Answer E is correct Between 60 and 80% of esophageal injuries give rise to clinical signs or symptoms, the sensitivity and specificity of which depend on the location of the injury, the size of the perforation, the degree of contamination, the length of time elapsed after injury, and the presence of associated injury. Odynophagia, dysphagia, hematemesis, oropharyngeal blood, cervical crepitus, pain and tenderness in the neck or chest, resistance to passive motion, dyspnea, hoarseness, bleeding, cough, and stridor are commonly noted. Fever, subcutaneous emphysema, abdominal tenderness, and mediastinal crunching sounds (Hamman sign) may be observed. Pain is the most common presenting symptom (71% of patients), followed by fever (51%), dyspnea (24%), and crepitus (22%). Overall, the signs and symptoms associated with esophageal injury are fairly nonspecific, and a high index of suspicion must be maintained to make the diagnosis. Evaluation of potential penetrating trauma to the esophagus is based on the trajectory and path of the missile. For penetrating injuries near the organ, it is necessary to prove that the esophagus is uninjured. Generally, this is done in one of two ways: (1) surgical exploration demonstrates a missile path inconsistent with esophageal injury or (2) direct examination of the esophagus reveals no injury. If neither option is feasible, proof is obtained through diagnostic testing. Plain x-rays, CT scans, and contrast esophagograms have all been used for this purpose, with varying sensitivity and specificity. Endoscopy may add to the diagnosis, but the results depend on the operator's technique and experience, and there is a risk that it may exacerbate an esophageal tear or further injure an unstable cervical spine. In an otherwise asymptomatic patient who is awake, alert, and able to cooperate, a simple contrast swallow is usually sufficient to exclude injury. Injuries resulting from low-velocity projectiles and stab wounds do not cause large tissue defects; for these injuries, barium gives superior anatomic detail and is the agent of choice. Injuries from large-caliber or high-velocity projectiles usually cause more damage; thus, the contrast agent tends to spread more widely throughout the mediastinum during a diagnostic swallow. If contrast studies are indicated in this setting, a water-soluble agent may be used first. Such agents cause pulmonary damage when aspirated, however, and should not be used if tracheoesophageal fistula is suspected. For patients who are going to the OR for another reason (e.g., vascular repair or laparotomy), it may be most expedient to perform esophagoscopy simultaneously with the operation. If the entire esophagus is well visualized and no injury is identified, no further evaluation is required. If the study is suboptimal for any reason, or if the possibility of an injury persists, a contrast study should be done as soon as the patient's condition permits. Surgical repair entails local débridement, wide drainage, primary repair of the perforation, and buttressing of the repair with a viable muscle flap. Primary repair can usually be accomplished when the perforation is operated on within 24 hours of occurrence. Complications seen after esophageal repair include esophageal leakage and fistula, wound infection, mediastinitis, empyema, sepsis, and pneumonia. Esophageal anastomotic leakage is the most common complication associated with repair of perforations, occurring in 10 to 28% of repairs.

A 38-year-old woman was the victim of domestic abuse when she was stabbed in the right mid- and upper neck. She is transported by the emergency medical service to the ED and rapidly moved to the operating room after examination demonstrated findings consistent with a vascular injury. During an internal carotid artery exploration for bleeding, which of the following structures should not be sacrificed? A) Mastoid process B) Occipital artery C) Ascending ramus of the mandible D) Hypoglossal nerve E) Ansa cervicalis

Key Concept/Objective: To demonstrate understanding of the anatomy of carotid artery exploration Answer D is correct For wounds to the distal internal carotid artery, exposure can be difficult. A 3 to 5 French Fogarty balloon catheter can be placed through the area of injury or through a proximal arteriotomy to help with distal control. The catheter should be advanced distally and the balloon inflated to provide a dry field for arterial repair. Repair can be done around the catheter; the balloon is deflated near the conclusion of the repair, and the catheter is removed before the final several sutures are tied. If exposure of the distal internal carotid artery is needed, a number of important structures should be identified and protected. The hypoglossal nerve is usually encountered within several centimeters of the carotid bifurcation and should be dissected free of the internal carotid and retracted upward. This is facilitated by division of the occipital artery, which crosses superficial to the hypoglossal nerve on its course from the external carotid artery toward the occiput. It is also helpful to divide the ansa cervicalis branches that run inferiorly from the hypoglossal nerve to supply the muscles of the neck. Injury to the hypoglossal nerve results in impaired motor function of the tongue and can lead to dysarthria and dysphasia. Injury to or sectioning of the ansa cervicalis causes little or no morbidity. Further distal exposure of the internal carotid artery may require unilateral mandibular subluxation or division of the ascending ramus of the mandible. Such maneuvers are somewhat easier when the patient is nasotracheally intubated. They increase the size of the small area immediately behind the condyle and allow easier division of the stylohyoid ligament and the styloglossus and stylopharyngeus muscles. These three structures can be divided together adjacent to their common origin at the styloid process. If this is done, care should be taken to preserve the facial nerve, which lies superficial to these muscles and must be dissected free of the muscles before they are divided. The underlying glossopharyngeal nerve, which lies deep to these muscles and superficial to the internal carotid artery, should also be protected by dissecting it free of the muscles before their division. Injury to the facial nerve results in loss of function of the muscles of facial expression. If the glossopharyngeal nerve is injured, loss of motor and sensory supply to parts of the tongue and pharynx increases the risk of aspiration. Once the muscles originating from the styloid process have been divided, the styloid process itself can be resected to gain a further short distance of distal exposure. In very rare instances, it may prove useful to remove portions of the mastoid bone to provide even more distal exposure of the internal carotid artery as it enters the carotid canal. For more distal lesions, it is necessary to place a posterolateral scalp incision, reflect a medially based scalp flap, and divide the ipsilateral external auditory canal. This approach results in better exposure of the mastoid process and allows exposure of the intrapetrous portion of the internal carotid after removal of the overlying bone of the mastoid with a high-speed bone drill.

A 24-year-old man is transported to the emergency department (ED) after being stabbed in the chest two times. He was intoxicated and hemodynamically stable at the scene, but on arrival at the ED, a pulse cannot be found. Examination reveals two 3 cm lacerations over the left chest at the level of the nipple in the anterior and midaxillary line. After securing an airway and while resuscitation is being performed, an emergency department resuscitative thoracotomy (EDRT) is performed. Which of the following is not one of the six therapeutic goals of EDRT? A) Relief of cardiac tamponade B) Cross-clamping the pulmonary hilum to control hemorrhage C) Repairing an injury to the left pulmonary artery D) Cross-clamping the descending aorta for lower extremity hemorrhage E) Confirming endotracheal intubation

Key Concept/Objective: To demonstrate understanding of the goals of emergency department (ED) resuscitative thoracotomy Answer C is correct. The technique of EDRT ideally requires that an antiseptic solution be splashed on the chest, but skin preparation is not required. A left anterolateral thoracotomy incision is made from the sternal border to the midaxillary line in the fourth intercostal space. A chest retractor is inserted and opened widely. The costochondral junctions of the fifth, the fourth, and sometimes the third rib are divided quickly with the scalpel to provide exposure. The six main therapeutic goals are (1) confirming endotrachal intubation; (2) control of hemorrhage; (3) relief of cardiac tamponade if present; (4) effective cardiac compression; (5) cross-clamping the pulmonary hilum in the case of major lung hemorrhage, air embolism, or massive bronchopleural fistula; and (6) cross-clamping of the descending aorta for lower torso hemorrhage control. Attention is directed first to the injury. If there is exsanguination from a great vessel, the hemorrhage is controlled with pressure. If air embolism is the cause of the arrest, the hilum is clamped and air is evacuated from the aorta. Otherwise, the pericardium is opened anterior and parallel to the phrenic nerve. The hemopericardium is evacuated, the cardiac injury is controlled with digital pressure, and a temporary repair is performed. After the cause of the arrest has been addressed, the descending thoracic aorta is occluded with a vascular clamp or digital pressure and intrathoracic cardiac compression is initiated. The patient's intravascular volume is restored, and electrolyte imbalances are corrected. If the patient can be saved, he or she is transported to the operating room (OR) for definitive repair and closure.

An 18-year old male arrives in the emergency department (ED) with a penetrating wound to zone II of the neck overlying the sternocleidomastoid muscle just above the clavicle. He is tachycardic to 120 beats/min, and his blood pressure is 140/80 mm Hg. When the paramedic's hand is removed, examination reveals significant tissue destruction, with blood and air emanating from the wound. Which of the following statements is true regarding the management of penetrating neck trauma? A) Emergent tracheotomy is preferred to cricothyrotomy B) Pulsatile bleeding is not an indication for emergent operative intervention C) Zone I and III injuries should not be managed operatively D) Computed tomographic (CT) angiography of the head and neck is as sensitive as conventional angiography in identifying vascular injuries E) CT of the neck is as sensitive as endoscopic evaluation of the aerodigestive tract in identifying injuries

Key Concept/Objective: To demonstrate understanding of the initial management and workup of penetrating neck injuries Answer D is correct Recent studies in trauma patients show that there is a higher rate of complications in patients who had their cricothyrotomy converted rather than left as a cricothyrotomy. Some have even advocated that elective cricothyrotomies may be used preferentially in critically ill patients with difficult neck anatomy because of the low complication rates. In patients with median sternotomies, the higher placed cricothyrotomy is preferred to avoid secretions at the incision site. Although more data are becoming available showing similar or improved outcomes with cricothyrotomy, it is still typical to convert a cricothyrotomy to a tracheotomy in 1 to 2 days in a stable patient. Management of penetrating neck trauma starts by determining the depth of the wound. If the wound did not violate the platysma, injury to major underlying structures can be ruled out. If the wound clearly violated the platysma or its depth cannot be determined, further investigation is warranted. Patients with any obvious hard signs of vascular injury (e.g., pulsatile bleeding from the wound, an expanding hematoma, a bruit, or a neurologic deficit), shock, or air bubbling from the wound require immediate operative exploration. If the patient is stable and has no obvious signs of underlying vascular injury, a nonoperative workup should be obtained regardless of the location of the skin wound. Historically, management of penetrating neck injuries included categorizing them on the basis of their location in the neck. In this schema, the neck was divided into three distinct zones and management was determined by the zone in which the patient happened to have wounds in the skin. Zone I was from the clavicles to the inferior aspect of the cricothyroid cartilage and included the trachea, esophagus, aortic arch, great vessels (subclavian and carotid arteries as well as jugular veins), and cervical spine. Zone II, the largest of the three zones, extended from the inferior aspect of the cricothyroid cartilage to the angle of the mandible and included the aerodigestive tract, great vessels of the neck, and cervical spine. Zone III was from the angle of the mandible to the skull base and included the aerodigestive tract (the oral cavity, pharynx, and larynx), great vessels, cervical spine, and cranial vault. This concept of mandatory exploration of patients with zone II injuries and nonoperative workup of zones I and III has a number of problems. Division of the neck into three distinct areas results in extremely small zones I and III, and even zone II, in the middle of the neck, is not very long in most patients. Moreover, the location of the skin wound is not a reliable guide to the location of injuries to underlying structures. In penetrating trauma, the path of the missile or blade is clearly as important as the location of the skin wound. An object that enters the skin in zone II can easily injure structures in zone I or III, depending on the direction in which it entered the skin. A selective approach regardless of zone reduces the rate of negative neck explorations. Therefore, the zones of the neck are useful as descriptors but not as a guide for management. CT scanners are not as effective as screening tools for aerodigestive injuries. These structures, the larynx, pharynx, hypopharynx, cervical esophagus, and trachea, should be directly visualized. This evaluation may take many forms, including indirect laryngoscopy, flexible fiberoptic nasopharyngolaryngoscopy, flexible bronchoscopy, and rigid or flexible esophagoscopy. For vascular injuries, conventional angiography was for many years the gold standard for ruling out vascular injury, but it is invasive, can be time consuming, and is resource intensive. With the advent of multidetector CT scanners, CT angiography has sensitivities and specificities that approach those of conventional angiography.

Upon exploration of the abdomen for a gunshot wound, a 25-year-old trauma victim is found to have massive bleeding from the splenic hilum and a large full thickness injury of the splenic flexure of the colon that involved 60% circumference of the bowel wall and associated with mild spillage of stool. After splenectomy, he had received 8 units of blood and is requiring vasopressor support. Which of the following is involved in the appropriate surgical management of the colon for this patient? A) Repair with debridement and closure in one layer B) Repair with debridement and serosal patch C) Repair with resection and primary anastomosis D) Resection and end colostomy E) Repair with resection and primary anastomosis and end sigmoid colostomy

Key Concept/Objective: To demonstrate understanding of the management of colonic injury Answer D is correct Colonic injuries that involve less than 50% of the bowel wall without devascularization are amenable to primary repair with limited debridement. For colonic injuries that involve more tissue loss or devascularized segments of bowel, repair with resection and primary anastomosis is necessary. Patients with destructive colon injuries who have comorbid conditions, transfusion requirements greater than 6 units of blood, or are in shock, are at greater risk for staple line dehiscence and these patient should be managed with resection and colostomy

At laparotomy for a gunshot wound to the abdomen, a small enterotomy is found in the jejunum and a mesenteric injury with a bleeding mesenteric arterial branch vessel is found in the ileum. After hemostasis is achieved by ligation of the mesenteric vessel, the corresponding segment of bowel is aperistaltic and a Doppler cannot detect arterial flow on the antimesenteric side of the bowel wall. Which of the following is included in appropriate surgical management or this patient? A) Primary repair of the jejunal injury and no further treatment of the ileal injury B) Primary repair of the jejunal injury and resection of the involved ileal segment with anastomosis C) Resection of the involved jejunal segment with anastomosis and resection of the involved ileal segment with anastomosis D) Resection of the involved jejunal segment and no further treatment of the ileal injury E) Resection of the involved jejunal segment, no further immediate treatment of the ileal injury, and return to the operating room within 48 hours for second look laparotomy

Key Concept/Objective: To demonstrate understanding of the management of traumatic injuries to the small intestine Answer B is correct Once injuries to the small bowel are identified, the decision must be made whether to perform primary repair, resect injured segments, or some combination of the two. Primary repair of multiple injuries preserves bowel length and is preferred. Management of each wound is determined by its severity. Small partial-thickness injuries (grade I) are managed by reapproximating seromuscular layers with interrupted sutures. Small full-thickness wounds (grade II) are repaired with limited debridment and closure. Larger full-thickness wounds (grade III) may be repaired if luminal narrowing can be avoided. Otherwise resection and primary anastomosis is required. Extensive wounds and wounds associated with devascularization (grades IV and V) are treated with resection and anastomosis. When mesenteric injury is encountered in the absence of bowel injury, the associated bowel must be closely assessed for evidence of vascular compromise, and if found, bowel resection and anastomosis is required.

A 59-year-old gentleman suffered a stab wound to the left upper quadrant. At celiotomy, an intramural hematoma of the anterior stomach at the greater curve is found. Which of the following is included in the appropriate surgical management of this injury? A) Observation alone if it is a non-expanding lesion B) Unroofing and evacuation, followed by seromuscular closure C) Placement of seromuscular sutures in surrounding stomach to bury the hematoma D) Wedge resection of the injured stomach E) Distal gastrectomy and Billroth II reconstruction

Key Concept/Objective: To demonstrate understanding of the management of traumatic injuries to the stomach Answer B is correct Treatment of gastric injury is dictated by its severity which is classified according to a grading system developed by the American Association for the Surgery of Trauma (AAST). Intramural hematomas (grade I) are managed by means of unroofing and evacuation, followed by seromuscular closure with interrupted sutures. The great majority of gastric perforations (grades II and III) are amenable to primary repair, and a two-layer technique is recommended to achieve hemostasis of the well vascularized stomach. Injuries that include significant tissue loss (grade IV) or gastric devascularization (grade V) are rare but are generally not amenable to primary repair and require distall gastrectomy, and very rarely, total gastrectomy.

A 19-year-old male was stabbed in the buttock. A single deep wound is noted on the medial buttock and a rectal exam reveals gross blood. Rigid proctoscopy at the bedside uncovers a laceration of the lateral wall of the mid rectum that is no longer bleeding. The patient is hemodynamically stable. Which of the following is included in appropriate management for this patient? A) Admission to ward and conservative management with NPO, intravenous fluids, broad spectrum antibiotics, and serial abdominal examinations B) CT scan with rectal contrast C) Exploratory laparotomy and repair with primary closure or resection with anastomosis D) Exploratory laparotomy and proximal diversion E) Presacral drainage and proximal diversion

Key Concept/Objective: To demonstrate understanding of the management of upper and lower rectal injuries Answer C is correct. All patients with a penetrating wound to the pelvis, perineum, buttock, or upper thigh should be evaluated for rectal injury. Digital examination for the presence of rectal blood is mandatory, but its absence does not rule out rectal injury. Rigid proctoscopy should be performed whenever there is any suspicion of rectal injury. The anterior and lateral sidewalls of the upper two-thirds of the rectum are serosalized. Injuries to these regions of the rectum are considered intraperitoneal and are treated in the same manner as colonic injuries (see explanation for question 2). The upper two-thirds of the rectum posteriorly and the lower one-third of the rectum are not serosalized and injuries to these areas are classified as extraperitoneal. Injuries to the extraperitoneal rectum are explored and repaired and fecal diversion is recommended. Wounds of the lower rectum that are difficult to reach are not explored and instead are managed with fecal diversion and presacral drainage.

At laparotomy for a stab wound to the abdomen, an 18 year-old male is found to have a 1 cm colotomy of the transverse colon that was repaired primarily. There was minimal spillage of stool. Which of the following would be involved in the appropriate postoperative antibiotic therapy for this patient? A) No postoperative antibiotics B) A first generation cephalosporin for 24 hours C) Broad spectrum antibiotics for 24 hours D) Broad spectrum antibiotics for 10 to 14 days E) Broad spectrum antibiotics until the patient is afebrile with normalization of WBC

Key Concept/Objective: To demonstrate understanding of the management strategy of postoperative antibiotics for hollow viscus injury Answer C is correct When hollow viscus injury is suspected, antibiotics with broad spectrum aerobic and anaerobic coverage should be administered before skin incision. If injury is confirmed, the antibiotics should be continued for a 24 hour period. The EAST practice Management Guidelines Workgroup has reviewed the available evidence regarding perioperative antibiotic management in the trauma setting and data from prospective studies clearly indicate that prolonging antibiotic treatment beyond 24 hours provides no additional protection against surgical site infections.

A 23-year-old construction worker's legs were crushed between a truck and a building. He was brought to the emergency department for evaluation, and crush injury to both lower extremities is diagnosed. In patients with crush syndrome, also known as traumatic rhabdomyolysis, treatment includes all of the following measures except A) Limiting the return of myoglobin and other toxic materials to the circulation by isolated limb perfusion with tourniquet application proximal to the affected area B) High-volume IV crystalloid infusion C) Cardiac monitoring D) Forced diuresis E) Alkalinization of the urine

Key Concept/Objective: To identify the treatment of crush injury Answer A is correct Crush syndrome (also referred to as traumatic rhabdomyolysis) is a clinical syndrome consisting of rhabdomyolysis, myoglobinuria, and subsequent renal failure. It is caused by prolonged compression of muscle tissue (frequently in the thigh or the calf) and is usually seen in victims of motor vehicle accidents who require a long extrication procedure or in earthquake victims who are rescued from beneath rubble after being trapped for several hours or days. Once released from entrapment, crush syndrome patients are likely to exhibit agitation, severe pain, muscle malfunction, swelling, and other systemic symptoms. The pathophysiologic process underlying this syndrome begins with muscle breakdown from direct pressure, impaired muscle perfusion leading to ischemia and necrosis, and the release of myoglobin. As long as the patient is entrapped, the ischemic muscle is isolated from the circulation, and this isolation affords some protection against the systemic effects of the released myoglobin and other toxic materials. Extrication and the resulting reperfusion of necrotic and ischemic muscle lead to the second insult, the reperfusion injury. This injury is caused by the formation of toxic reactive oxygen metabolites, which leads to failure of ion pumps and increasing permeability of cell membranes and microvasculature. When large amounts of muscle are involved, the resulting fluid changes can rapidly induce shock. The large quantities of potassium, lactic acid, and myoglobin that are released into the circulation can lead to renal failure, disseminated intravascular coagulation, and circulatory arrest. Treatment should begin at the time of extrication so as to anticipate the onset of the syndrome. The first step is initiation of IV fluid therapy, starting with a 2 L crystalloid bolus and continuing with crystalloid infusion at a level of 500 mL/hr (the dosage must be adjusted in pediatric and cardiac patients). Cardiac monitoring is essential (T waves indicate hyperkalemia). Intravascular fluid expansion and osmotic diuresis, by maintaining high tubular volume and urine flow, may prevent renal failure. Forced diuresis can be achieved by giving mannitol or other diuretics. Alkalization of the urine with sodium bicarbonate (1 mEq/kg IV to a total of 100 mEq) is a controversial measure but is recommended by some on the grounds that it should, in theory, reduce intratubular precipitation of myoglobin. If compartment syndrome is suspected, compartment pressures should be measured.

A 27-year-old helmeted male fell from his motorcycle while traveling on a roadway partially covered with sand and gravel. He did not lose consciousness and was awake at the scene complaining of pain in his right hip. He was taken to the emergency department, where he underwent a trauma evaluation. He was speaking and had bilateral breath sounds and a strong carotid pulse. His heart rate was in the 90s, his blood pressure was 143/87 mm Hg, and he had a normal oxygen saturation. During the evaluation, gentle rocking of his pelvis evoked tenderness. A sheet wrap was placed. He then underwent chest and cervical films that did not demonstrate an injury. An anteroposterior pelvis film demonstrated an injury pattern consistent with an anteroposterior compression injury. A contrast-enhanced computed tomographic (CT) scan of the abdomen and pelvis showed the aforementioned pelvic fracture as well as extravasation of contrast from a branch of the right internal iliac artery. What is the most appropriate next step in the management of this patient? A) Immediate transfer to the intensive care unit for further resuscitation B) Immediate transfer to the operating room for an exploratory laparotomy C) Immediate transfer to the operating room for open reduction and internal fixation of the pelvic fracture D) Immediate transfer to the interventional radiology suite for angiographic embolization E) Perform the FAST exam (focused assessment for sonographic evaluation of the trauma patient)

Key Concept/Objective: To understand the management algorithm for patients with unstable pelvic fracture Answer D is correct Pelvic fractures are frequently associated with significant hemorrhage, not only because of the fracture itself but also because pelvic trauma is often accompanied by serious injuries to other parts of the body (e.g., the chest or the abdomen). Given that bleeding in pelvic fracture patients can occur in other body compartments besides the pelvis and can be arterial as well as venous, it is of the utmost importance to identify its source and, ideally, determine its nature as soon as possible. This information is crucial in determining what the next steps in management should be. The first step after diagnosing a pelvic fracture should be the immediate application of some type of external stabilization device (e.g., a sheet wrap or a device such as the Pelvic Binder [Pelvic Binder Inc., Dallas, TX]). The rationale behind this step is that approximating the fractured bones and thereby decreasing the volume of the pelvis may reduce blood loss, particularly from the fractured bones and the lacerated venous plexus. In addition, stabilization may minimize further damage to blood vessels and prevent dislodgment of recent clots. It is doubtful, however, whether this procedure actually reduces arterial hemorrhage to a significant degree. In hemodynamically unstable patients with clinical signs of a pelvic fracture, the next step (immediately after— or, preferably, while—x-rays of the chest, the pelvis, and the cervical spine are obtained according to advanced trauma life support [ATLS] protocols) should be the FAST (focused assessment for sonographic evaluation of the trauma patient) to rule out a significant intra-abdominal bleeding source. If the FAST is negative and no other obvious sources of hemorrhage (e.g., chest or extremities) are found, the pelvis is the most likely source of the bleeding. The question then arises as to whether the pelvic hemorrhage is predominantly arterial or venous. An arterial bleeding source in the pelvis is found in 73% of hypotensive patients who do not respond to initial fluid resuscitation. Contrast-enhanced CT is extremely helpful in determining the presence of arterial hemorrhage in cases of pelvic fracture, but it can be performed only if the patient is stable enough to undergo the time-consuming transfer to the imaging suite. Extravasation of contrast medium, a large retroperitoneal hematoma, or abrupt cutoff of an artery on CT indicates that angiographic embolization is necessary. Contrast extravasation (so-called contrast blush) is a particularly good predictor of arterial hemorrhage necessitating embolization, having a sensitivity and specificity of well over 80%. Thus, CT is an ideal means of identifying patients who should be treated with angiographic embolization and, ideally, should be performed in all pelvic fracture patients who are stable enough to undergo this procedure. Arterial hemorrhage should preferably be treated by angiographic embolization, which has shown excellent results in current studies. Success rates exceed 90%, and major complication rates are below 5%. This technique does, however, require a skilled, experienced, and permanently available interventional radiology service. In patients with unstable fractures, venous hemorrhage is treated with operative placement of an external fixation device. This measure requires specific expertise on the part of the trauma surgeon; in experienced hands, it should take no longer than 20 minutes to perform. Patients with more severe pelvic fractures probably benefit most from this procedure. Retroperitoneal packing may be employed as an adjunct to external fixator placement. It is unlikely that external fixation has a significant impact on arterial hemorrhage; consequently, it is vital to decide whether angiographic embolization should precede placement of an external fixation device in the operating room.

An 18-year-old man was in an altercation and suffered multiple gunshot wounds to the abdomen. He is taken emergently to the operating room for an exploratory laparotomy without preoperative imaging. At laparotomy, a large hematoma is seen within the Gerota fascia on the left, and exploration of the left kidney is planned. Which of the following is true regarding operative management of renal injuries? A) Intraoperative intravenous pyelography (IVP) is not necessary in this situation B) Isolation and control of the renal artery and vein are best performed after opening the Gerota fascia C) If the kidney appears ischemic, surface cooling is indicated to preserve function D) Principles of management include control of hemorrhage and repair of any injuries to the collecting system E) To preserve renal function, the renal injury should be addressed before other intra-abdominal sources of bleeding are controlled

Key Concept/Objective: To understand the operative management of renal injuries Answer D is correct Intraoperative IVP is indicated when exploration of a kidney is planned and no preoperative imaging is available. The main purpose of one-shot IVP in this setting is to confirm the presence of a contralateral functioning kidney; a potential benefit is the ability to rule out major injury. Although somewhat controversial, most evidence suggests that the renal artery and vein should be controlled prior to opening the Gerota fasica. Surface cooling is not recommended as it adds time to the operation and can contribute to hypothermia. The principles of operative management of renal injuries are control of bleeding followed by repair of injuries to the collecting system. The renal injury should not be addressed until other bleeding sources in the abdomen have been controlled.

A 52-year-old trauma patient is noted to have gross hematuria. Which of the following is true regarding hematuria in trauma patients? A) The degree of hematuria is directly related to the severity of renal injury B) In blunt trauma, hematuria necessitates evaluation of the entire genitourinary tract C) The absence of hematuria rules out a renal injury D) A computed tomographic (CT) scan should be the initial diagnostic test in hemodynamically unstable patients with hematuria E) Severe hematuria confirms a renal parenchymal injury

Key Concept/Objective: To demonstrate knowledge of the initial evaluation of trauma patients with hematuria Answer B is correct Hematuria is a common finding in trauma patients. The degree of hematuria is not directly related to the severity of injury to the urogenital tract. In some instances (renal artery thrombosis or pedicle avulsion), the patient with renal injury may have no hematuria, so the absence of hematuria does not rule out a renal injury. Severe hematuria does not help determine the location of the injury. Hematuria in blunt trauma patients requires evaluation of the entire urogenital tract, typically with a CT scan if the patient is hemodynamically stable. A CT scan is not indicated in hemodynamically unstable patients. Other tests, such as ultrasonography, diagnostic peritoneal lavage, or intraoperative intravenous pyelography (IVP), are better suited to those patients.

A 42-year-old man is the unrestrained driver in a motor vehicle collision. Reports from the accident scene include significant damage to the steering wheel, and cardiac injury is suspected. Which of the following statements is false regarding cardiac trauma? A) Patients who make it to the hospital with vital signs have greater than 50% chance of survival B) Cardiac tamponade is preferably treated by performing a pericardial window in the OR C) Cardiac tamponade is accurately diagnosed by pericardiocentesis D) Pump failure from cardiac contusion is usually attributable to right heart failure E) Right atrial perforation is the most common injury in patients with cardiac disruption who make it to the hospital alive

To demonstrate understanding of traumatic cardiac injury Answer C is correct The incidence of cardiac trauma continues to rise as a consequence of growing urban violence, improved detection of cardiac injuries, and an increase in the percentage of patients with such injuries who arrive at trauma centers alive. Improved prehospital transport, along with the continuing evolution of diagnostic, surgical, and anesthetic techniques, has contributed to an increase in overall survival in this population. Although the overall mortality associated with cardiac trauma remains high, survival rates of 50 to 95% are not uncommon in patients who arrive at the hospital with vital signs. Although cardiac ultrasonography has many advantages, the subxiphoid pericardial window remains the gold standard for diagnosis of cardiac injury. For otherwise stable patients with proximity wounds or suggestive signs and symptoms, a pericardial window should be considered when ultrasonographic findings are equivocal or when ultrasonography is unavailable. This procedure is usually performed in the OR with the patient under general anesthesia, often in combination with abdominal exploration. Some authorities advocate using pericardiocentesis to detect cardiac injuries, especially where rapid access to the OR, trauma surgeons, and anesthesiologists is not available. Drawbacks to this approach include the high rate of false positive and false negative results and the potential for iatrogenic cardiac injuries. Furthermore, pericardiocentesis is of limited use in treating tamponade because blood within the pericardial sac often is clotted and is not amenable to removal through a needle. Blunt cardiac injury typically involves a direct blow to the chest, usually sustained in a motor vehicle collision or a fall. Cardiac injuries generally are associated with sternal or rib fractures, although they may occur in the absence of any chest wall fracture; however, sternal fractures do not predict the presence of blunt cardiac injury. The most common location of blunt cardiac injury is the anterior heart, which consists primarily of the right ventricle. A blow that causes the sternum to exert a direct impact on the myocardium may result in direct injury to myocardial cells, sometimes leading to cell death, mechanical dysfunction, or dysrhythmias. The diagnosis of myocardial contusion is elusive. Many tests have been proposed, but none have proved definitive, except for direct visualization of the heart at surgery or autopsy. For practical purposes, the clinically significant sequelae of myocardial contusion are myocardial dysrhythmias and pump failure. Pump failure associated with cardiac contusion is usually the result of right heart failure in that most hemodynamically significant cardiac contusions are caused by injury to the anterior right ventricular free wall. Treatment of right heart failure from cardiac contusion consists of inotropic support and reduction of right ventricular afterload. Dysrhythmias secondary to cardiac contusion are treated in the same manner as dysrhythmias of any other etiology. In the rare patient with cardiorrhexia who presents to the hospital with signs of life, the most common injury is right atrial perforation.

A 42-year-old female unrestrained driver in a motor vehicle collision suffered blunt trauma to her abdomen and was taken to the operating room for peritonitis. Upon laparotomy, a 2 cm laceration of the second and third portion of the duodenum is noted with gross spillage of bile. The laceration encompasses approximately 50 percent of the circumference of the duodenum. Cannulation of the common bile duct through the cystic duct reveals that the ampulla is intact and is several centimeters away from the duodenal injury. What is the most appropriate surgical option for treatment of this injury? A) Pancreaticoduodenectomy B) Resection of the second and third portions of the duodenum with primary anastomosis C) Debridement of the duodenal injury and Roux-en-y duodenojejunostomy D) Primary repair of the duodenal injury E) Primary repair of the duodenal injury with pyloric exclusion and loop gastrojejunostomy

Key Concept/Objective: To demonstrate the understanding of the management of major duodenal injuries Answer E is correct For duodenal injuries that can be primarily closed, this is the first option. Concomitant pyloric exclusion is recommended to divert gastric contents in the period of healing. This is performed by either using a TA stapler to occlude the stomach at the level of the pylorus or by sewing the pylorus closed through a gastrotomy. Gastrojejunostomy is performed to establish gastrointestinal continuity. The pylorus will eventually open within 2 weeks to 2 months. If the injury is not amenable to primary repair, if the injury is proximal to the ampulla, antrectomy and gastrojejunostomy can be performed with closure of the duodenal stump. If the injury is distal to the ampulla, Roux-en-y duodenojejunostomy to the proximal end of the duodenal injury and oversewing the distal duodenum can be performed. For massive injuries of the proximal duodenum and head of the pandreas, destruction of the ampulla and proximal pancreatic duct or distal common bile duct may preclude reconstruction and a Whipple procedure may be necessary.

A 22-year-old man suffered a stab wound to the epigastrium. At laparotomy, gross hemoperitoneum is encountered and the bleeding source is a gastroepiploic artery that is ligated without consequence. A deep parenchymal laceration of the mid body of the pancreas with ductal transection is also found. The patient has thus far received 9 units of packed red blood cells and 9 units of plasma and is still hemodymically unstable. Which of the following is the correct surgical procedure for this patient's pancreatic injury? A) Pancreaticoduodenectomy B) Distal pancreatectomy and splenectomy C) Distal pancreatectomy alone D) Central pancreatectomy with pancreatico-jejunostomy E) Minor debridement and placement of closed-suction surgical drains

Key Concept/Objective: To demonstrate the understanding of the management of major pancreatic traumatic injury with ductal disruption Answer B is correct. Transection of the pancreas to the left of, or at the level of the superior mesenteric vessels is best treated by distal pancreatectomy. The parenchyma can be closed with a TA stapler or suture ligatures and a drain should be left near the transection line. Concern for the possibility of post-splenectomy sepsis and subphrenic abscess formation following splenectomy has prompted several authors to describe the technique of distal pancreatectomy with splenic salvage. Generous mobilization of the entire gland and spleen is a prerequisite and an average of 22 tributaries of the splenic vein and 7 branches of the splenic artery must be ligated, adding significant operative time. This approach can be considered in a hemodyamically stable patient with only minor associated injuries but is not appropriate for the hemodynamically compromised patient or patients with other major associated injuries.

A 17-year-old boy suffered a handle bar injury to the upper abdomen during collision with the sidewalk after attempting to jump his bike down 7 stairs. On admission to the trauma bay, he is hemodyamically stable and moderately tender to palpation in the epigastrium, without peritoneal signs. A CT scan suggests an intramural hematoma at the third portion of the duodenum. Which of the following includes the most appropriate management strategy for this patient? A) Admission to the surgical ward for observation, serial examination, and nasogastric decompression, and TPN if necessary B) Laparotomy and evacuation of the hematoma with seromuscular closure of the duodenum C) Laparotomy and segmental resection of the duodenum with primary anastomosis D) Laparotomy with evacuation of the hematoma and reconsruction with duodeno-jejunostomy E) Laparotomy with pancreatico-duodenotomy

Key Concept/Objective: To demonstrate the understanding of the management of minor duodenal injuries Answer A is correct History of a direct blow to the epigastrium may be suggestive of a duodenal injury. If a duodenal hematoma is detected by nonoperative means, it will most often resolve with nonoperative management that includes nasogastric suction and TPN. If a duodenal hematoma is discovered at laparotomy, it is treated with evacuation and does not require resection or reconstruction. Minor lacerations of the duodenum can be closed primarily.

Following two gunshot wounds to the abdomen, a 21-year-old male is explored in the operating room. Multiple injuries are noted that require primary repair of a gastrotomy, a splenectomy, and a segmental colectomy. A laceration of the edge of the body of the pancreas is noted and does not appear to transect the pancreatic duct. He is hemodyamically stable. Which of the following includes the appropriate surgical management of this patient's pancreatic injury? A) Primary repair of the pancreatic injury with capsular sutures. B) Distal pancreatectomy and splenectomy C) Placement of closed-suction surgical drains D) Pancreatico-jejunostomy E) No surgical treatment is necessary

Key Concept/Objective: To demonstrate the understanding of the management of minor pancreatic traumatic injury diagnosed intraoperatively Answer C is correct If identified at laparotomy, minor injuries to the pancreas that do not involve the pancreatic duct require only hemostasis and placement of closed suction drains. No attempt should be made to repair capsular lacerations because this may result in formation of a pancreatic pseudocyst. Drains are removed when amylase concentrations are less than half of that of serum, generally within 48 hours.

A 20-year-old man is brought to the ED with a single gunshot wound to his central abdomen between the xiphoid and the umbilicus. Examination demonstrates a distended and tender abdomen. He is taken to the OR for exploration while being resuscitated with blood and plasma. On exploration, hemoperitoneum is encountered and a zone I retroperitoneal hematoma is noted. Medial visceral rotation is performed, and an injury to the pancreas and superior mesenteric artery (SMA) is identified. Which of the following is the best management of the SMA injury in a relatively stable patient with this combination of injuries? A) Ligate the artery B) Perform primary repair of the artery C) Use a great saphenous vein (GSV) patch to repair the artery D) Ligate the artery at its origin and use a polytetrafluoroethylene (PTFE) graft or GSV patch to reconstruct the SMA takeoff from the distal abdominal aorta away from the site of injury E) Use a PTFE interposition graft to reconstruct the artery

Key Concept/Objective: To demonstrate understanding of the management of injury to the superior mesenteric artery (SMA) Answer D is correct Injuries to the SMA are managed according to the anatomic level of the perforation or thrombosis. On rare occasions, in patients with injuries beneath the neck of the pancreas, one may have to transect the pancreas to obtain proximal control. Another option is to perform left medial visceral rotation and apply a clamp directly to the origin of the SMA. Injuries to the SMA in this area or just beyond the base of the mesocolon are often associated with injuries to the pancreas. The potential for a postoperative leak from the injured pancreas near the arterial repair has led numerous authors to suggest that any extensive injury to the artery at this location should be ligated. Because of the intense vasoconstriction of the distal SMA in patients who have sustained exsanguinating hemorrhage from more proximal injuries treated with ligation, the collateral flow from the foregut and hindgut is often inadequate to maintain the viability of the organs in the distal midgut, especially the cecum and the ascending colon. Therefore, it is safest to place a saphenous vein or PTFE graft on the distal infrarenal aorta, away from the pancreatic injury and any other upper abdominal injuries. Such a graft can be tailored to reach the side or the anterior aspect of the SMA, or it can be attached to the transected distal SMA in an end-to-end fashion without significant tension. Soft tissue must be approximated over the aortic suture line of the graft to prevent the development of an aortoenteric fistula in the postoperative period. In patients with severe shock from exsanguination caused by a complex injury to the SMA, damage-control laparotomy is indicated: the injured area should be resected and a temporary intraluminal Argyle, Javid, or Pruitt-Inahara shunt inserted to maintain flow to the midgut during resuscitation in the surgical intensive care unit (ICU). When ligation is indicated for more distal injuries to the SMA, segments of the ileum or even the right colon may have to be resected because of ischemia. The survival rate in patients with penetrating injuries to the SMA is approximately 55 to 60% overall but only 20 to 25% when any form of repair more complex than lateral arteriorrhaphy is necessary.

A 27-year-old unrestrained male driver is transported to your emergency room in stable condition after motor vehicle collision. The report includes a crushed steering wheel and he complains of epigastric pain. In the trauma bay he remains hemodynamically stable and moderate tenderness without rebound or guarding is noted on his physical examination. A CT scan reveals a small hematoma on the anterior tail of the pancreas abutting a superficial laceration of the gland in the same area. The main pancreatic duct does not appear to be involved. No other injuries are detected. Which of the following should be included in the next appropriate step in management of this patient? A) Admission to the surgical ward for observation, bowel rest, and serial abdominal examinations B) Admission to the intensive care unit for invasive monitoring, bowel rest, and serial abdominal examinations C) Surgical exploration with debridement and placement of surgical drains D) Surgical exploration with distal pancreatectomy incorporating the site of injury within the specimen E) ERCP to evaluate whether the duct is disrupted

Key Concept/Objective: To demonstrate the understanding of the management of minor pancreatic traumatic injury diagnosed preoperatively Answer A is correct. Traumatic pancreatic injuries can be cataloged by the American Association for the Surgery of Trauma's grading scale. Grade I injuries involve small hematomas and superifical injuries without ductal injury. Grade II injuries involve large hematomas or major lacerations without duct injury or tissue loss. Grade I injuries can be managed conservatively. Grade II injuries can also be managed conservatively but can sometimes require operative management. The most important determinant of outcome in pancreatic injury is the status of the duct which often can be gained from a CT scan. The results of ERCP early in pancreatic injury are mixed and this modality is generally not recommended in the acute injury phase.

An 18-year-old ejected driver of a high-speed rollover motor vehicle accident presents with stridor and subcutaneous emphysema of the neck. What is the preferred method of securing an airway in this patient? A) Intubation over a flexible bronchoscope B) Blind intubation C) ED cricothyrotomy D) ED tracheostomy E) OR tracheostomy

Key Concept/Objective: To demonstrate understanding of airway management with tracheal injury Answer E is correct. With tracheobronchial injuries, as with all injuries, airway management is the first priority in treatment. If the patient is maintaining his or her own airway and is adequately ventilated, a cautious, noninterventional approach is probably the best initial choice until further diagnostic workup is performed or other life-threatening injuries are stabilized. Careless handling or mishandling of the airway (e.g., inadvertently placing an endotracheal tube through a transected or ruptured airway and into soft tissue) can be disastrous and may compound the injury. ED tracheotomies are difficult and may be dangerous, to say the least. How best to secure an airway in a patient with neck trauma and possible tracheal injury is a matter of debate. With blind endotracheal intubation, the path of the tube distal to the larynx is unknown, and it is possible to lose the lumen or create a false passage. With intubation over a flexible bronchoscope, the tube can be visualized as it passes beyond the site of injury, and some of the dangers of blind intubation are thereby mitigated; however, some degree of sedation is usually required, and if the patient is oversedated, the airway that was being spontaneously protected may be lost. Paralytic medications should generally be avoided in this setting for the same reason. Urgent tracheotomy or cricothyrotomy, performed in the OR, is advocated by many as the safest and securest way of obtaining airway control. If the trachea is completely transected, the distal trachea can usually be found in the superior mediastinum and grasped for insertion of a cuffed tube. The approach taken to airway control must vary with the resources and expertise available at each institution. One must also keep in mind that even after the airway is secured, it may still be possible to exacerbate the injury by means of aggressive ventilation with high airway pressures. Tube thoracostomies should be appropriately placed at this time and connected to suction, even though dyspnea may worsen.

The patient in question 1 was taken to the OR for exploration and repair of an injured superficial femoral artery. After revascularization of the left leg, the lower leg is noted to become hyperemic and tense. A decision to perform lower extremity fasciotomy is made. Which of the following statements regarding compartment syndrome is true? A) Fasciotomy should not be performed unless an elevated pressure (> 30 mm Hg) is measured B) Loss of pulse is an early sign of compartment syndrome C) The lower leg has three compartments: anterior, lateral, and posterior D) The forearm has four compartments: superficial flexor, deep flexor, superficial extensor, and deep extensor E) Delayed primary closure and skin grafting are appropriate methods of closing fasciotomy sites

Key Concept/Objective: To demonstrate understanding of compartment syndrome and fasciotomy Answer E is correct Compartment syndrome is a condition characterized by abnormally high pressure within a closed space. The elevated tissue pressure leads to venous obstruction within the space. When the pressure continues to increase, the intramuscular arteriolar pressure is eventually exceeded. At that point, blood can no longer enter the capillary space, and the result is shunting within the compartment. If the pressure is not released, muscle and nerve ischemia occurs, leading to irreversible damage to the contents of the compartment. The key to the diagnosis of compartment syndrome is continuous assessment of any extremity injury in which elevated pressures may develop. It must always be remembered that the diagnosis is a clinical one; although compartment pressures may be measured, clinical suspicion and findings suggestive of compartment syndrome on physical examination should suffice to mandate therapy. There are a number of clinical situations in which fasciotomy should be considered: when there is a 4- to 6-hour delay before revascularization, when arterial injuries are present in conjunction with venous injuries, when crush injuries or high-kinetic energy injuries have been sustained, when vascular repair has already been performed (reperfusion), when an artery or vein has been ligated, when a patient is comatose or has a head injury and physical examination is impossible, and when a patient has tense compartments or elevated compartment pressures. In these scenarios, fasciotomy should be considered as a prophylactic maneuver. The lower leg contains four osseofascial compartments: the anterior compartment, the lateral compartment, the superficial posterior compartment, and the deep posterior compartment. The thigh contains three osseofascial compartments: the quadriceps, the hamstrings, and the adductors. For fasciotomies of the lower leg, there are two techniques: perifibular fasciotomy and the double-incision technique. Perifibular fasciotomy affords access to all four compartments of the leg via a single lateral incision that extends from the head of the fibula to the ankle, following the general line of the fibula. Although decompression in the lower extremity may be achieved via either of the two techniques, the double-incision technique is preferred in the setting of trauma because it readily ensures that all four compartments have been decompressed. The forearm consists of three osseofascial compartments: the superficial flexor compartment, the deep flexor compartment, and the extensor compartment.

A 17-year-old male suffered a single gunshot to the lower abdomen. At laparotomy, he is found to have a through-and-through injury to the sigmoid colon and a pulsatile hematoma in the left pelvis. Which of the following is not true regarding iliac artery injuries? A) In young trauma patients, both internal iliac arteries can be ligated with little risk of pelvic ischemia B) Partial transection of the external iliac artery can be repaired by completing the transection and performing an end-to-end anastomosis C) In damage control laparotomy, a complex iliac artery injury can be resected and vascular flow maintained with a Javid shunt D) Ligation of the external iliac artery has a 40% risk of need for leg amputation E) A prosthetic graft in a contaminated field has a low risk of infection because the pelvis is a well-vascularized area

Key Concept/Objective: To demonstrate understanding of control of iliac artery injuries Answer E is correct Injuries to the internal iliac arteries are usually ligated even if they occur bilaterally because young trauma patients typically have extensive collateral flow through the pelvis. Options for repair of an injured common or external iliac artery include lateral arteriorrhaphy, completion of a partial transection with an end-to-end anastomosis, and resection of the injured area with insertion of a conduit. If damage control laparotomy is planned because the patient is in hemorrhagic shock, the injured area can be excised and a temporary prosthetic Javid shunt inserted to maintain flow to the ipsilateral lower extremity during resuscitation in the surgical intensive care unit. Ligation of the common or external iliac artery in a hypotensive patient carries a 40 to 50% risk of leg amputation. In a contaminated field such as this patient, any repair of the iliac arteries carries a high risk of infection and possibility of rupture of the repair. In such a situation, ligation of the iliac artery and extra-anatomic bypass is a viable alternative.

A 54-year-old woman was the restrained passenger of a motor vehicle hit on the passenger's side. She is hemodynamically stable in the trauma bay but complaining of right-sided chest pain and dyspnea. Examination reveals paradoxical movement of a segment of chest wall. Computed tomography (CT) of the chest confirms that multiple ribs are fractured with an underlying hemothorax. Which of the following statements is false regarding flail chest? A) Intubated patients will have paradoxical chest wall movement B) Intracostal nerve blocks and thoracic epidural anesthesia are effective in controlling pain C) Significant pathophysiology can be attributed to underlying pulmonary contusion D) Complications include hypoventilation, atelectasis, and pneumonia E) Internal fixation of ribs may be indicated in certain cases

Key Concept/Objective: To demonstrate understanding of flail chest physiology and management Answer A is correct Flail chest is the most serious of the blunt chest wall injuries. It is common after any form of blunt thoracic trauma, and although it may occur as an isolated finding, it is usually associated with other significant injuries. Flail chest represents a disruption of the stability and normal respiratory mechanics of the rib cage. It involves fractures of adjacent ribs, each of which is fractured in two or more places, so a panel of chest wall moves independently of, and in the opposite direction to, the remainder of the chest. When it occurs in conjunction with separation of the costochondral or costosternal joints, the sternum can also be part of the flail segment, and the condition is termed a sternal flail chest. The following are the three components of the pathophysiology of flail chest: 1. Alteration of chest wall mechanics. The paradoxical motion of a large flail segment occasionally impairs the patient's ability to achieve an adequate tidal volume or an effective cough. 2. Underlying pulmonary contusion. In the vast majority of serious flail chest injuries, this is the most significant physiologic aberration. In the contused portion of the lung, there is extravasation and accumulation of blood and fluid in the alveolar air space, which results in sufficient shunting to produce hypoxemia. 3. Pain. The extreme pain of multiple rib fractures leads to profound splinting and diminution of tidal volume and prevents adequate coughing and pulmonary toilet in most alert patients. The combination of depressed tidal volume and inadequate coughing leads to hypoventilation, atelectasis, and often pneumonia. The diagnosis is typically suspected on the basis of the presence of numerous adjacent rib fractures on a chest radiograph but can be conclusively confirmed only by the presence of a paradoxical motion observed in the involved segment in a spontaneously breathing patient. A flail segment may be overlooked in a patient on positive pressure ventilation because there may be no paradoxical motion without inspiratory effort. Therefore, in an intubated patient, the diagnosis must be sought through careful examination and palpation of the rib cage for stability. Depending on the presence and severity of an underlying pulmonary contusion and adequate pulmonary toilet, many flail chest victims can avoid intubation. However, any patient with flail chest who demonstrates further deterioration of pulmonary function and who becomes hypoxic or hypercarbic should undergo mechanical ventilation aimed at (1) ensuring a tidal volume adequate for establishing normal chest wall excursion and (2) maintaining a respiratory rate adequate for achieving normocarbia. Hypoxia is managed by increasing the fraction of inspired oxygen (FIo2) and applying sufficient positive end-expiratory pressure to achieve adequate oxygenation (usually defined as arterial oxygen saturation greater than 90%) with nontoxic levels of Fio2. A few patients with severe disruption of chest wall mechanics as a result of flail chest continue to require positive pressure ventilation even though adequate pain control has been achieved and the pulmonary contusions are beginning to resolve. Some of them may benefit from internal fixation of the multiple rib fractures, which restores chest wall stability and eliminates much of the fracture-related pain. A variety of methods have been described to stabilize the ribs and obtain compression osteosynthesis of each fracture site. An isolated, single-institution series described the efficacy of this procedure in patients with flail chest. Further clinical trials are needed.

A construction worker suffers a crushing injury to his pelvis. On evaluation in the trauma bay, he is hemodynamically stable but has an obvious pelvic fracture. There is blood at the penile meatus. Which of the following is true regarding traumatic urethral injuries? A) Urethral injuries are more common in women than in men B) Placement of a urethral catheter should be performed before imaging studies are obtained C) Pelvic fractures in men are typically associated with anterior urethral injuries D) Retrograde urethrography should be performed to rule out urethral injury E) All of the above are true

Key Concept/Objective: To demonstrate understanding of the evaluation of urethral injuries Answer D is correct Most injuries to the male urethra are caused by blunt trauma. Prostatomembranous urethral distraction injuries in males occur in 5% of pelvic fractures, which are the most common cause of posterior urethral injury. Anterior urethral (penile and bulbar urethral) injuries are commonly caused by straddle injury but may be the result of penile fracture or penetrating injuries to the genitalia. The female urethra is rarely injured, but when such injury occurs, it is usually associated with bladder injury and pelvic fracture. Blood at the urethral meatus—the classic sign of injury to the male urethra—an indication for immediate urethrography. Attempts at catheter placement prior to imaging studies risk converting an incomplete injury to a complete disruption and are to be discouraged if urethral injury is suspected.

A 56-year-old woman involved in a rollover motor vehicle crash is transported to the emergency department (ED) by ambulance. She is making loud, incomprehensive sounds, opens her eyes, and withdraws her arms only after painful stimuli. Her blood pressure is 75/30 mm Hg, and her pulse is 110 beats/min. Which of the following statements about traumatic brain injury (TBI) in this patient is false? A) Glascow Coma Scale (GCS) score is greater than 8, so she does not need to be intubated B) Hypotension and hypoxemia are independent risk factors for mortality with TBI C) Hyperglycemia is not a risk factor for poor outcome in TBI D) Hypertonic saline has been demonstrated to lower intracranial pressure (ICP) in patients with TBI E) Use of mannitol is appropriate in patients with lateralizing symptoms and TBI in the absence of direct ICP monitoring

Key Concept/Objective: To demonstrate understanding of the initial management of traumatic brain injury (TBI) Answer C is correct. Hypoxia and hypotension are two of the worst secondary insults following TBI and should be avoided at all costs during initial patient management. Severe oxygen desaturation (< 60%) during transport to the hospital is associated with a 3.5-fold increase in mortality, and the duration of in-hospital oxygen desaturation (< 90%) is an independent predictor of mortality. One episode of post-TBI hypotension doubles mortality risk, whereas two or more episodes increase the relative risk of mortality to 8.1, and the total duration of hypotensive episodes is a significant predictor of morbidity and mortality. Patients with SCI above T5 are particularly prone to severe hypotension as a result of vasogenic spinal shock and should be aggressively treated with volume resuscitation and administration of alpha-adrenergic vasopressors. Therefore, aggressive and complete physiologic resuscitation is the highest priority for patients with head and spinal cord injuries. Early orotracheal intubation and ventilation are recommended for patients with a GCS score of 8 or lower or a motor score of 4 or lower. Other indications for immediate intubation are loss of protective laryngeal reflexes and ventilatory insufficiency, as manifested by hypoxemia (arterial oxygen tension [PaO2] < 60 mm Hg), hypercarbia (arterial carbon dioxide tension [PaCO2] > 45 mm Hg), spontaneous hyperventilation (causing PaCO2 < 26 mm Hg), and respiratory arrhythmia. Indications for intubation before transport are deteriorating consciousness (even if the patient is not in a coma), bilateral fractured mandible, copious bleeding into the mouth (as occurs with fracture of the base of the skull), and seizures. Patients with TBI should be normoventilated (PaCO2 ≈ 40 mm Hg) unless they clearly demonstrate signs of raised ICP. Fluid replacement should be performed with isotonic solutions such as normal saline or packed red blood cells when appropriate. Although appropriate in certain clinical scenarios, lactated Ringer solution should generally be avoided in patients with TBI as it has been shown to increase ICP and decrease intracranial compliance. Hypertonic saline may be more appropriate in patients with TBI as it has been shown to improve intracranial compliance and lower ICP. Glucose-based solutions should be avoided in adult patients with TBI. Elevated serum glucose in the first 24 hours after TBI or following surgical intervention is associated with a worse neurologic outcome. In a recent review from the IMPACT study, hyperglycemia on admission or during hospital course was an independent prognostic indicator of outcome 6 months following TBI. A recent review of 77 patients with severe TBI identified early hyperglycemia as a strong predictor of poor outcome and death. Intracranial hypertension should be treated aggressively as it is a primary predictor of outcome in the acute period following TBI. Hyperventilation reduces ICP by decreasing cerebral blood volume (CBV) and does not interfere with volume resuscitation. Because it can result in iatrogenic cerebral ischemia, however, it should be used only in patients with acute neurologic deterioration with lateralizing signs (e.g., anisocoria, unilateral motor posturing). Hyperosmolar therapies are also effective agents for reduction of ICP but have differing effects on volume status. Mannitol is an osmotic diuretic that has long been used in the setting of TBI and is thought to reduce ICP by drawing free water out of the brain as well as improving blood rheology, resulting in improved cerebral blood flow and reduced CBV.

A 19-year-old male is in a high-speed motorcycle crash. On arrival at the ED, he is hypotensive, with flaccid paralysis of all four extremities and incontinence of urine and stool. Noncontrast cervical spine CT demonstrates a displaced fracture of C4 with retropulsion into the spinal canal. Which of the following statements is true regarding the management of SCI? A) Craniocervical traction can be performed in highly unstable fractures with the use of fluoroscopy B) Specialists agree that steroids should be given in call cases of SCI C) Complete SCI is a contraindication to surgical intervention D) Surgical intervention does not have a role in the acute management of SCI E) Surgical intervention is limited to patients with otherwise nonreducible spine fractures

Key Concept/Objective: To demonstrate understanding of the management of acute spinal cord injury (SCI) Answer A is correct Traction should always be applied under strict neurologic monitoring. If the patient's condition deteriorates when the weight is increased, additional weight should be removed and the patient should immediately undergo imaging (e.g., with plain films, magnetic resonance imaging, or both). In the case of a highly unstable fracture, traction should be guided by fluoroscopy rather than serial x-rays. The challenge is to identify the most effective treatment or combination of treatments with the fewest severe side effects—a task requiring many experiments for each treatment tested. Methylprednisolone, which is thought to act by scavenging free radicals, has been reported to be neuroprotective in patients with SCIs. Considerable controversy remains, however, regarding the clinical benefit of methylprednisolone administration after acute SCI. There is general agreement among physicians that immobilization of the patient to prevent further injury and early stabilization of fractures and dislocations of the spine are necessary. The single widely accepted indication for early urgent surgical treatment is ongoing neurologic deterioration in the presence of spinal canal compromise from bone and disk fragments, hematoma, or unreduced subluxation. Surgical indications still under debate include incomplete SCI (with persistent spinal cord compression) and complete SCI with the possibility of some neurologic recovery.

On exploration of the abdomen for a gunshot wound, a 25-year-old trauma victim is found to have massive bleeding from the splenic hilum and a large full-thickness injury of the splenic flexure of the colon that involves 60% circumference of the bowel wall associated with mild spillage of stool. After splenectomy, he has received 8 units of blood and requires vasopressor support. Appropriate surgical management of the colon injury involves? A) Repair with débridement and closure in one layer B) Repair with débridement and a serosal patch C) Repair with resection and primary anastomosis D) Resection and end colostomy E) Repair with resection and primary anastomosis and end sigmoid colostomy

Key Concept/Objective: To demonstrate understanding of the management of colonic injury Answer D is correct Colonic injuries that involve less than 50% of the bowel wall without devascularization are amenable to primary repair with limited débridement. For colonic injuries that involve more tissue loss or devascularized segments of bowel, repair with resection and primary anastomosis is necessary. Patients with destructive colon injuries who have comorbid conditions, have transfusion requirements greater than 6 units of blood, or are in shock are at greater risk for staple line dehiscence and should be managed with resection and colostomy.

A 29-year-old professional pianist is brought to the emergency department after suffering a rollover motor vehicle accident while driving at high speed. He is alert, oriented, and hemodynamically stable. On examination, his only injury is a 4 cm longitudinal laceration to his distal left forearm over the radial artery. Blood spurts from the wound when direct pressure is not held, and the distal radial pulse at the wrist is not palpable. Motor and sensory examinations of the left hand are normal. What is the appropriate management of this patient? A) Apply a pressure dressing and perform serial examinations B) Explore the wound in the OR and ligate the injured radial artery C) Explore the wound in the OR and perform a primary repair of the radial artery D) Explore the wound in the OR, perform a primary repair of the radial artery, and place the patient on a heparin drip E) Explore the wound in the OR and perform a repair of the artery using a vein patch angioplasty

Key Concept/Objective: To demonstrate understanding of the management of distal extremity arterial injuries Answer B is correct On occasion, arteries in the distal extremities (e.g., the radial, ulnar, or tibial arteries) may have to be repaired or ligated after trauma. In most patients, there is little need for repair of these arteries, which can typically be ligated without deleterious effects. The safety of ligation is predicated on the presence of adequate arterial flow from the nonaffected arteries, as well as retrograde blood flow from an intact palmar or plantar arch. Repair of injuries to these arteries is associated with the possibility of embolization or other surgical problems. In addition, the patency rate for grafts in the distal extremities tends to be low.

A 24-year-old male is brought to the ED after a single gunshot wound to his left lower abdomen. He is severely hypotensive with a distended abdomen and a pulseless left lower extremity. On exploration in the OR, an injury to the left common iliac artery and vein is identified. During the case, the patient receives one round of advanced cardiac life support after developing pulseless electrical activity; he regains a carotid pulse, and with massive resuscitation, his blood pressure has stabilized. What is the most appropriate management of this patient? A) Ligate both the left common iliac artery and vein and resuscitate in the ICU B) Ligate the left common iliac vein, place a Pruitt shunt in the artery, resuscitate in the ICU, and perform definitive operation at a later date C) Perform primary repair of the vein and then subsequent primary repair of the artery D) Perform primary repair of the vein and then subsequent interposition PTFE graft repair of the artery E) Perform primary repair of the vein and then subsequent repair of the artery using the ipsilateral internal iliac artery

Key Concept/Objective: To demonstrate understanding of the management of injury to the iliac artery and vein Answer B is correct Ligation of either the common or the external iliac artery in a hypotensive trauma patient leads to a greater than 50% amputation rate in the postoperative period; consequently, injuries to these vessels should be repaired if at all possible. The standard options for repair—lateral arteriorrhaphy, completion of a partial transection with an end-to-end anastomosis, and resection of the injured area with insertion of a conduit—are feasible in most situations. On rare occasions, it may be preferable either to mobilize the ipsilateral internal iliac artery to serve as a replacement for the external iliac artery or to transpose one iliac artery to the side of the contralateral iliac artery. When a patient is in severe shock from exsanguination caused by a complex injury to the common or the external iliac artery, damage-control laparotomy is indicated. The injured area should be resected and a temporary intraluminal Argyle, Javid, or Pruitt-Inahara shunt inserted to maintain flow to the ipsilateral lower extremity during resuscitation in the surgical ICU. One unique problem associated with repair of the common or the external iliac artery is the choice of technique when significant enteric or fecal contamination is present in the pelvis. In such cases, there is a substantial risk of postoperative pelvic cellulitis, a pelvic abscess, or both, which may lead to blowout of any type of repair. When extensive contamination is present, it is appropriate to divide the common or external iliac artery above the level of injury, close the injury with a double row of continuous 4-0 or 5-0 polypropylene sutures, and bury the stump underneath uninjured retroperitoneum. If a stable patient has obvious ischemia of the ipsilateral lower extremity at the completion of this proximal ligation, one may perform an extra-anatomic femorofemoral crossover bypass with an 8 mm externally supported PTFE graft to restore arterial flow to the extremity. If the patient is unstable, one should take several minutes to perform an ipsilateral four-compartment below-the-knee fasciotomy; this step will counteract the ischemic edema that inevitably leads to a compartment syndrome and compromises the early survival of the leg. After adequate resuscitation in the surgical ICU, the patient should be returned to the OR for the femorofemoral graft within 4 to 6 hours. Injuries to the internal iliac arteries are usually ligated even if they occur bilaterally because young trauma patients typically have extensive collateral flow through the pelvis. The survival rate in patients with isolated injuries to the external iliac artery in older series exceeded 80% when tamponade was present. If the injury was large and free bleeding had occurred preoperatively, however, the survival rate was only 45%. Current studies report overall survival rates of approximately 45 to 55% for injuries to the common iliac artery and 62 to 65% for injuries to the external iliac artery. Injuries to the common or the external iliac vein are best treated by a lateral venorrhaphy with continuous 4-0 or 5-0 polypropylene sutures. Significant narrowing often results, and a number of reports have demonstrated occlusion on postoperative venography. For patients with narrowing or occlusion, as well as for those in whom ligation was necessary to control exsanguinating hemorrhage, the use of elastic compression wraps and elevation for the first 5 to 7 days after operation is mandatory.

A 42-year-old female nonrestrained driver in a motor vehicle collision suffered blunt trauma to her abdomen and was taken to the operating room for peritonitis. On laparotomy, a 2 cm laceration of the second and third portions of the duodenum is noted, with gross spillage of bile. Cannulation of the common bile duct through the cystic duct reveals that the ampulla is intact and is several centimeters away from the duodenal injury. The most appropriate surgical option for treatment of this injury is: A) Pancreaticoduodenectomy B) Resection of the second and third portions of the duodenum with primary anastomosis C) Débridement of the duodenal injury and Roux-en-Y duodenojejunostomy D) Primary repair of the duodenal injury E) Primary repair of the duodenal injury with pyloric exclusion and loop gastrojejunostomy

Key Concept/Objective: To demonstrate understanding of the management of major duodenal injuries Answer E is correct For duodenal injuries that can be primarily closed, this is the first option. Concomitant pyloric exclusion is recommended to divert gastric contents in the period of healing. This is performed by either using a TA stapler to occlude the stomach at the level of the pylorus or by sewing the pylorus closed through a gastrotomy. Gastrojejunostomy is performed to establish gastrointestinal continuity. The pylorus will eventually open within 2 weeks to 2 months. If the injury is not amenable to primary repair, if the injury is proximal to the ampulla, antrectomy and gastrojejunostomy can be performed with closure of the duodenal stump. If the injury is distal to the ampulla, Roux-en-Y duodenojejunostomy to the proximal end of the duodenal injury and oversewing the distal duodenum can be performed. For massive injuries of the proximal duodenum and head of the pandreas, destruction of the ampulla and proximal pancreatic duct or distal common bile duct may preclude reconstruction and a Whipple procedure may be necessary.

A 22-year-old man suffered a stab wound to the epigastrium. At laparotomy, gross hemoperitoneum is encountered, and the bleeding source is a gastroepiploic artery that is ligated without consequence. A deep parenchymal laceration of the midbody of the pancreas with ductal transection is also found. The patient has thus far received 9 units of packed red blood cells and 9 units of plasma and is still hemodymically unstable. The correct surgical procedure is? A) Pancreaticoduodenectomy B) Distal pancreatectomy and splenectomy C) Distal pancreatectomy alone D) Central pancreatectomy with pancreaticojejunostomy E) Minor débridement and placement of closed-suction surgical drains

Key Concept/Objective: To demonstrate understanding of the management of major pancreatic traumatic injury with ductal disruption Answer B is correct Transection of the pancreas to the left of or at the level of the superior mesenteric vessels is best treated by distal pancreatectomy. The parenchyma can be closed with a TA stapler or suture ligatures, and a drain should be left near the transection line. Concern for the possibility of postsplenectomy sepsis and subphrenic abscess formation following splenectomy has prompted several authors to describe the technique of distal pancreatectomy with splenic salvage. Generous mobilization of the entire gland and spleen is a prerequisite, and an average of 22 tributaries of the splenic vein and seven branches of the splenic artery must be ligated, adding significant operative time. This approach can be considered in a hemodyamically stable patient with only minor associated injuries but is not appropriate for the hemodynamically compromised patient or patients with other major associated injuries.

After blunt force trauma to the abdomen, a 24-year-old male is brought from the trauma bay to the operating room. Following splenectomy for a grade IV splenic injury, a retroperitoneal hematoma within Gerota fascia on the left and extending down the left paracolic gutter is identified. It does not appear to change in size over a 10-minute observation period. The most appropriate treatment of this injury is? A) No surgical treatment B) Exploration of the hematoma C) Exploration of the hematoma following proximal vascular control D) Placement of laparotomy pads and abdominal re-exploration in 24 to 48 hours E) Left nephrectomy

Key Concept/Objective: To demonstrate understanding of the management of retroperitoneal hematomas Answer A is correct Retroperitoneal hematomas are classified on an anatomic basis: zone 1 is the central area, bounded laterally by the kidneys and extending from the diaphragmatic hiatus to the bifurcation of the vena cava and aorta; zone 2 comprises the lateral area of the retroperitoneum from the kidneys laterally to the paracolic gutters; and zone 3 is the pelvic portion of the retroperitoneum. All zone 1 hematomas should be explored regardless of the mechanism of injury: they signal possible aortic, vena caval, duodenal, or pancreatic injury. Zone 2 and 3 hematomas should be explored in cases of penetrating trauma but not, as a rule, in cases of blunt trauma (except in cases of expanding zone 2 hematomas). Before a retroperitoneal hematoma is opened, proximal vascular control should be obtained so that hemorrhage will be minimized once the effect of the tamponade is lost.

A 50-year-old man was working on his second-story roof when he slipped, fell off, and landed on his back. He sustained multiple fractures, including a type III odontoid fracture, displaced pubic rami fractures, and bilateral distal radioulnar fractures. Which of the following spinal fractures does not require operative reduction and fixation? A) Type I Jefferson fracture B) Type III odontoid fracture of C2 with 4 mm displacement C) Type II odontoid fracture of C2 with 6 mm displacement D) Burst fracture of T12 with 50% loss of vertebral body height E) Chance fracture of L1

Key Concept/Objective: To demonstrate understanding of the management of spinal fractures Answer B is correct Atlas (Jefferson, or C1) fractures, which represent 5 to 10% of all cervical spine fractures, result from axial loading. Because of the large diameter of the spinal canal and the tendency of fragments to move outward, these fractures usually are not accompanied by significant neurologic injury. However, 40% of patients with an atlas fracture have another cervical fracture as well (usually involving C2). The integrity of the transverse ligament largely determines whether the fracture is stable. Injuries that involve the midportion of the transverse ligament or the periosteal insertion (e.g., types Ia and Ib) will not heal spontaneously and must be treated with surgical fixation of the C1-C2 complex. In contrast, type II injuries (e.g., avulsion injuries or comminuted lateral mass fractures) will usually heal in a rigid external orthosis (e.g., a halo vest). Three different types of odontoid fractures are recognized. Fractures of the tip of the odontoid process (avulsion fracture, type I) are uncommon but are thought to be stable in most cases. They can be treated by using a hard cervical collar with or without preceding cervical traction or by immobilizing the patient in a halo vest. Fractures of the neck (type II) and fractures at the junction of the odontoid process and the axis body (type III) are more common (accounting for 65 to 80% and 20 to 35% of odontoid fractures, respectively). Management of type II fractures depends on the degree to which the dens is displaced: fractures with more than 5 mm of displacement are typically managed surgically, whereas fractures with less than 5 mm of displacement can be treated nonsurgically with a halo vest or a semirigid orthosis (e.g., a sterno-occipitomandibular immobilizer [SOMI] brace). Most type III odontoid fractures will fuse with rigid external mobilization (i.e., either cervical traction followed by placement in a rigid cervical collar or placement in a halo vest). Again, more than 5 mm of displacement is an indication for surgical stabilization. Burst fractures are considered unstable even if there is no initial neurologic deficit. Early ambulation should be avoided because the axial loading may result in progressive collapse or angulation, with concomitant neurologic damage. Indications for the surgical treatment of burst fractures include the following: 1. Loss of more than 50% of body height 2. Retropulsed bony fragments narrowing the canal by more than 50% 3. Kyphotic angulation of 25° or more. A Chance fracture is a horizontal fracture through all three columns. It occurs most commonly in the lower lumbar spine and is a highly unstable injury. Chance fractures are now less frequent than they once were because of the widespread use of shoulder belts in addition to lap belts, which prevents upper torso flexion during deceleration. Chance fractures are treated with surgical stabilization, and decompression and correction of spinal alignment may be required as well. Fracture dislocations, also known as fracture subluxations, are three-column injuries that usually involve disruption of the ligamentous structures or the disk space. Dural lacerations and neurologic injury are common with such injuries. Fracture dislocations are considered unstable and are treated with surgical decompression and stabilization.

A 19-year-old male was stabbed in the buttock. A single deep wound is noted on the medial buttock, and a rectal examination reveals gross blood. Rigid proctoscopy at the bedside uncovers a laceration of the lateral wall of the midrectum that is no longer bleeding. The patient is hemodynamically stable. Appropriate management of this patient includes? A) Admission to the ward and conservative management with nothing by mouth, intravenous fluids, broad-spectrum antibiotics, and serial abdominal examinations B) A computed tomographic (CT) scan with rectal contrast C) Exploratory laparotomy and repair with primary closure or resection with anastomosis D) Exploratory laparotomy and proximal diversion E) Presacral drainage and proximal diversion

Key Concept/Objective: To demonstrate understanding of the management of upper and lower rectal injuries Answer C is correct. All patients with a penetrating wound to the pelvis, perineum, buttock, or upper thigh should be evaluated for rectal injury. Digital examination for the presence of rectal blood is mandatory, but its absence does not rule out rectal injury. Rigid proctoscopy should be performed whenever there is any suspicion of rectal injury. The anterior and lateral sidewalls of the upper two-thirds of the rectum are serosalized. Injuries to these regions of the rectum are considered intraperitoneal and are treated in the same manner as colonic injuries (see explanation for question 2). The upper two-thirds of the rectum posteriorly and the lower one-third of the rectum are not serosalized, and injuries to these areas are classified as extraperitoneal. Injuries to the extraperitoneal rectum are explored and repaired, and fecal diversion is recommended. Wounds of the lower rectum that are difficult to reach are not explored and instead are managed with fecal diversion and presacral drainage.

A 19-year-old male suffers two gunshot wounds to the abdomen. Hemodyamically unstable in the trauma bay, he is taken to the operating room for laparotomy. Multiple injuries are found and include a shattered bleeding spleen, a large devitalizing enterotomy of the ileum, and a large devitalizing injury to the transverse colon. The anesthetist informs you that the patient has received 12 units of blood and 10 units of plasma, and he remains hypotensive. His international normalized ratio is 2.2, and his arterial pH is 7.19. The appropriate management strategy includes? A) Placement of laparotomy pads, temporary closure of the abdomen with a Bogota bag, and transfer to the intensive care unit (ICU) B) Splenectomy, placement of laparotomy pads, temporary closure of the abdomen with a Bogota bag, and transfer to the ICU C) Splenectomy, jejunal resection without anastomosis, transverse colon resection without anastomosis, closure of the abdomen with a Bogota bag, and transfer to the ICU D) Splenectomy, jejunal resection with anastomosis, transverse colon resection with anastomosis, temporary closure of the abdomen with a Bogota bag, and transfer to the ICU E) Splenectomy, jejunal resection with anastomosis, transverse colon resection with anastomosis, closure of the abdomen, and transfer to the ICU

Key Concept/Objective: To demonstrate understanding of the principles of damage control surgery Answer C is correct. If a patient remains unstable during a trauma laparotomy or becomes cold, acidotic, or coagulopathic, an abbreviated or damage control procedure is indicated. The decision to perform a damage control procedure should be made at an early stage, before the so-called lethal triad (hypothermia, acidosis, and coagulopathy) has time to develop. Major bleeding and contamination should be controlled as best as possible. This may involve stapling of injured bowel without restoring intestinal continuity at the time of the initial operation. Restoring intestinal continuity is both time-consuming and has a high risk of anastomotic leak in the unstable patient. It may also be necessary to perform a rapid, temporary abdominal closure. The patient is then transported to the ICU and is warmed and resuscitated. Return to the operating room in 24 to 48 hours is planned once hemodynamic and metabolic disturbances are reversed

On final pathology, the biopsy of the lesion in the patient from question 1 returns positive for superficial spreading melanoma with a tumor thickness of 2.1 mm. Which of the following is not true regarding melanoma? A) Risk factors for melanoma include a family history of melanoma, the presence of blond or red hair, the presence of marked freckling of the upper part of the back, and a history of a squamous cell carcinoma of the skin B) The pattern of growth in melanoma, that is, radial versus vertical, is associated with overall prognosis C) Superficial spreading melanoma accounts for over 70% of melanoma cases D) Nodular melanoma lesions are often dome shaped and may occur anywhere on the body E) Acral lentiginous melanomas occur on the hands and feet, often under the nail bed

Key Concept/Objective: To demonstrate understanding of the risk factors and histologic subtypes of melanoma Answer A is correct A history of actinic keratosis and not squamous cell carcinoma is a risk factor for the development of melanoma. In one study, six risk factors were identified by multivariate analysis to be important in the development of malignant melanoma and include the following: a family history of melanoma, a history of three or more blistering sunburns before age 20 years, the presence of blonde or red hair, the presence of actinic keratosis, a history of 3 or more years of an outdoor summer job as a teenager, and the presence of marked freckling on the upper part of the back. Individuals with one or two of these factors have a 3.5-fold increased risk of developing melanoma, whereas if they have three or more, they have a 20-fold increase in risk. Melanoma may be classified into histologic subtypes based on patterns of growth and anatomic location. The significance of these subtypes is generally less important than the pattern of growth (radial versus vertical growth) and depth of penetration. The most common subtypes include lentigo maligna melanoma, superficial spreading melanoma, acral lentiginous melanoma, and nodular melanoma. Superficial spreading accounts for over 70% of melanomas. These lesions are most common in white adults and typically occur on the back or legs. Nodular melanomas are the second most common, accounting for between 15 and 30% of all melanomas. These lesions often appear dome shaped and may occur anywhere on the body. They typically invade the dermis early in their natural history owing to an early vertical growth phase. Lentigo maligna melanoma accounts for approximately 5% of all melanomas and is thought to arise in a focus of lentigo maligna, or Hutchinson freckle. These lesions demonstrate a prolonged radial growth phase before developing an invasive component. Acral lentiginous melanomas occur on the hands or feet often under the nail bed, where the dermis is thinner.

A 17-year-old was the unhelmeted driver of an all-terrain vehicle that was involved in a rollover accident at high speed. Evaluation revealed an epidural hematoma on the left with midline shift. An intraventricular drain was placed to monitor ICP. Which of the following statements regarding the tiered approach to the management of elevated ICP is false? A) Decompressive craniectomy should be instituted prior to barbiturate coma B) Repeat noncontrast head CT scan should be obtained prior to elevating therapy from one tier to the next C) Mannitol and hypertonic saline infusion can be repeated as needed independent of the specific tier D) Neuromuscular paralysis compromises the entire neurologic examination E) A neurosurgeon or neurointensivist should see the patient prior to elevating treatement

Key Concept/Objective: To demonstrate understanding of the tiered approach to managing elevated intracranial pressure (ICP) Answer D is correct Moderate hypothermia and barbiturate metabolic coma are highly effective means of reducing ICP but are associated with significant morbidity. As a result, many centers are considering surgical intervention prior to inducing barbiturate coma. Therapies within a tier should be maximized before moving to a higher tier, and careful consideration should be given to repeat head CT scans when a patient requires escalation of therapy (to evaluate for intracranial lesion progression). Mannitol and hypertonic saline can be delivered in boluses for acute control of ICP, but in adults, they are not typically used as a regular, intermittent therapy. They can be used across all tiers of therapy but are limited by their side effects of induced serum hyperosmolarity and induced hypernatremia. Although muscular paralysis compromises most of the neurologic examination, it is important to remember that neuromuscular blockade does not interfere with pupillary response, which is mediated by smooth muscle contraction.

A 21-year-old man is the nonhelmeted driver of a motorcycle who swerved off the road in an attempt to avoid hitting another vehicle. The GCS score at the scene is 7, so he is intubated and brought to the ED, where a head computed tomographic (CT) scan demonstrates a 1 cm left-sided subdural hematoma and 3 mm of midline shift. Neurosurgery evaluation is performed, and nonoperative intervention is recommended; an intraventricular catheter is placed to monitor ICP. Despite addressing all basic measures, the ICP is elevated at 26 mm Hg. Repeat noncontrast head CT is stable from prior images. Which management is most appropriate? A) Continue mannitol and hypertonic saline despite a serum osmolarity of 332 mOsm/L B) Moderate hyperventilation to PaCO2 of 30 mm Hg C) Decompressive craniotomy D) Fentanyl and propofol sedation to reduce cerebral metabolic demand E) Induce phenobarbital coma

Key Concept/Objective: To demonstrate understanding of the tiered approach to managing elevated intracranial pressure (ICP) Answer D is correct Patients who experience elevated ICP despite the above prophylactic measures should be managed by a protocol-directed, tiered approach. Therapies within a tier should be maximized before moving to a higher tier, and careful consideration should be given to repeat head CT when a patient requires escalation of therapy (to evaluate for intracranial lesion progression). It is generally not advisable for escalation of tiered therapy to occur without careful assessment by the treating neurosurgeon or neurointensivist as some etiologies of elevated ICP may result from reversible events (e.g., hypoventilation) that require more specific treatment strategies. Mannitol and hypertonic saline can be delivered in boluses for acute control of ICP, but in adults, it is not typically used as a regular, intermittent therapy. They can be used across all tiers of therapy but are limited by their side effects of induced serum hyperosmolarity and induced hypernatremia. Mannitol is usually administered in intravenous boluses of 0.25 to 1 g/kg over 10 to 15 minutes, until either ICP is controlled or serum osmolarity reaches 320 mOsm/L. Many neuroscience intensive care units prefer the use of fentanyl and propofol for ICP control as they are relatively shorter-acting agents that have demonstrated success in the management of severe TBI. Propofol is a sedative hypnotic with a rapid onset and a short duration of action. In patients requiring more than 48 hours of sedation, propofol infusion results in significantly fewer ventilator days than treatment with intermittent lorazepam and has a more favorable economic profile and shorter weaning time than midazolam. In moderate and severe TBI patients, a prospective trial found significantly improved ICP control with propofol infusion (versus morphine). After maximizing sedation and supporting CPP, many clinicians will institute muscular paralysis for ICP control. Although its efficacy may vary among patients, pharmacologic paralysis can be used in conjunction with maximal sedation to further reduce ICP. Paralysis is also achieved using continuous intravenous infusion and should be titrated to effect (using percutaneous peripheral nerve stimulation). Although muscular paralysis compromises most of the neurologic examination, it is important to remember that neuromuscular blockade does not interfere with pupillary response, which is mediated by smooth muscle contraction. Hyperventilation can be used as a strategy for ICP lowering despite maximal sedation and chemical paralysis but carries the risk of iatrogenic cerebral ischemia. Therefore, mild to moderate hyperventilation should be used only in patients with a continuous brain oxygenation monitor, such as a brain tissue oxygen probe or jugular bulb oximetry monitor. Decompressive craniectomy is an important component of protocol-driven therapy for patients with severe TBI and should be performed in patients who have exhausted tier I and II therapies and do not have adequate ICP control with intermittent hyperosmolar therapies. Moderate hypothermia and barbiturate metabolic coma are highly effective means of reducing ICP but are associated with significant morbidity. As a result, many centers are considering surgical intervention prior to inducing barbiturate coma.

A 64-year-old woman was a restrained driver in a high-speed motor vehicle collision. After 2 L of intravenous fluids and 1 unit of packed red blood cells, she is hemodynamically stable in the trauma bay. CT scans of her head, neck, chest, abdomen, and pelvis reveal only a 9 cm subcapsular hematoma of the right lobe of the liver that accounts for approximately 40% of the anterior surface of the liver. The following management strategy should be employed: A) Admission to a trauma ICU for hemodynamic monitoring, bed rest, serial hematocrit checks, and serial abdominal examinations B) Angiography and embolization of the hepatic arterial branch vessels C) Laparotomy and packing D) Laparotomy and exploration of the hematoma E) Laparotomy and anatomic hepatic resection

Key Concept/Objective: To demonstrate understanding of the treatment of hepatic injury Answer A is correct Hepatic injuries are classified according to a grading system developed by the American Association for the Surgery of Trauma Committee with a grading system that ranges from I to VI, with I representing superficial lacerations and small subcapsular hematoms and VI representing lethal avulsion of the liver from the vena cava. Isolated injuries that are not extensive (I to III) often require little or no treatment; however, extensive parenchymal injuries and those involving juxtahepatic veins may require complex maneuvers, including hepatic vascular isolation for successful treatment. Grade I and II subscapsular hematomas, which involve less than 50% of the surface of the liver, can be managed nonoperatively if the patient is hemodyamically stable without significant transfusion requirement. If discovered at laparotomy, grade I or II subcapsular hematomas should be left alone if they are not expanding. Expanding and grade III or greater subscapsular hematomas require exploration.

A 36-year-old intoxicated man fell backwards off a porch. In the trauma bay, his heart rate is 110 beats/min and blood pressure is 92/48 mm Hg. He has so far received 2 L of intravenous fluid. His CT scan reveals a laceration of the spleen that involves the hilum, 40% devascularizaton of the splenic parenchyma, and a perisplenic hematoma. The most appropriate management includes: A) Admission to a trauma ICU for hemodynamic monitoring, bed rest, serial hematocrit checks, and serial abdominal examinations B) Angiography and arterial embolization C) Laparotomy and splenectomy D) Laparotomy and splenorrhaphy E) Laparotomy and placement of lap pads for 24 to 48 hours

Key Concept/Objective: To demonstrate understanding of the treatment of splenic injury Answer C is correct. Splenic injuries are classified according to a grading system developed by the American Association for the Surgery of Trauma Committee with a grading system that ranges from I to V, with I and II being small tears and subcapsular hematomas (< 50% surface area) with no active bleeding. Many splenic injuries can be managed nonoperatively. Splenectomy is the usual treatment for hilar injuries (grade IV) and pulverized splenic parenchyma (grade V).

A 57-year-old pedestrian was struck by a motor vehicle traveling at 35 mph. He is brought to the hospital by ambulance and is hemodynamically stable with a Glascow Coma Scale score of 14. On physical examination, he has an obvious deformity to his left upper leg with exposed femur. Pulses on the left leg are not palpable and demonstrate a diminished signal at the popliteal, dorsalis pedis, and posterior tibial positions. Which of the following is the most appropriate initial management of this patient? A) Perform ankle-brachial indices (ABIs) of both legs in the emergency department (ED) to screen for vascular injury B) Perform computed tomographic angiography (CTA) of both legs to screen for vascular injury C) Urgently take the patient to the operating room (OR) for exploration and primary repair of the injured artery D) Urgently take the patient to the OR for on-table angiography, followed by repair of the injured artery E) Urgently take the patient to the OR for fixation of femur and recheck pulse examination

Key Concept/Objective: To demonstrate understanding of the workup and management of injuries to peripheral blood vessels Answer A is correct In patients with soft signs of arterial injury, the ABI—also known as the arterial pressure index (API), the Doppler pressure index (DPI), or the ankle-arm index (AAI)—is a highly useful adjunct to the physical examination. The ABI is obtained by placing a blood pressure cuff on the supine patient proximal to the ankle or wrist of the injured limb. The systolic pressure is determined with a Doppler probe at the respective posterior tibial and dorsalis pedis arteries or at the ulnar and radial arteries. The ratio of the highest systolic pressure obtained in the affected extremity to the systolic pressure in an unaffected extremity (most often a brachial artery) is the ABI. A 1991 study assessed the sensitivity and specificity of Doppler-derived arterial pressure measurements in trauma patients undergoing evaluation for possible extremity vascular injury. An ABI was obtained in 100 consecutive injured limbs, and all patients then underwent contrast arteriography. An ABI lower than 0.90 was 87% sensitive and 97% specific for arterial injury. The authors concluded that in the absence of hard signs of arterial injury, ABI is a reasonable substitute for screening arteriography, particularly if continued observation can be ensured. Arteriography has a sensitivity of 95 to 100% and a specificity of 90 and 98% and is therefore considered the gold standard for evaluation or confirmation of arterial injury. In the setting of extremity injury, however, nonselective angiography has not been found to be cost-effective and is often overly sensitive, detecting minimal injuries that do not call for further management. Furthermore, arteriography can be time consuming, can delay definitive treatment, and can give rise to complications of its own, including renal contrast toxicity and pseudoaneurysm formation. Angiography should be reserved for patients with soft signs of vascular injury and an abnormal ABI; there is little reason to perform angiography in a patient with hard signs of injury, unless an intraoperative angiogram is needed to delineate the anatomy. In no case should transport to the OR for definitive treatment be delayed so that arteriography can be performed. Ongoing radiologic advances have led some centers to consider using CTA to evaluate arterial injury. To date, this approach has been formally evaluated in only a modest number of studies, although there is reason to believe that it will be more widely used in the future. A few small series found CTA to have a sensitivity and a specificity of approximately 90% in the evaluation of large arteries; other studies suggested that this modality might be a reasonable alternative to conventional arteriography for diagnosis of traumatic arterial injuries. Before CTA can be considered equivalent to the gold standard, large randomized trials will have to be performed.

A 42-year-old man is the unrestrained driver in a motor vehicle collision. Reports from the accident scene include significant damage to the steering wheel, and cardiac injury is suspected. Which of the following statements is false regarding cardiac trauma? A) Patients who make it to the hospital with vital signs have greater than 50% chance of survival B) Cardiac tamponade is preferably treated by performing a pericardial window in the OR C) Cardiac tamponade is accurately diagnosed by pericardiocentesis D) Pump failure from cardiac contusion is usually attributable to right heart failure E) Right atrial perforation is the most common injury in patients with cardiac disruption who make it to the hospital alive

Key Concept/Objective: To demonstrate understanding of traumatic cardiac injury Answer C is correct. The incidence of cardiac trauma continues to rise as a consequence of growing urban violence, improved detection of cardiac injuries, and an increase in the percentage of patients with such injuries who arrive at trauma centers alive. Improved prehospital transport, along with the continuing evolution of diagnostic, surgical, and anesthetic techniques, has contributed to an increase in overall survival in this population. Although the overall mortality associated with cardiac trauma remains high, survival rates of 50 to 95% are not uncommon in patients who arrive at the hospital with vital signs. Although cardiac ultrasonography has many advantages, the subxiphoid pericardial window remains the gold standard for diagnosis of cardiac injury. For otherwise stable patients with proximity wounds or suggestive signs and symptoms, a pericardial window should be considered when ultrasonographic findings are equivocal or when ultrasonography is unavailable. This procedure is usually performed in the OR with the patient under general anesthesia, often in combination with abdominal exploration. Some authorities advocate using pericardiocentesis to detect cardiac injuries, especially where rapid access to the OR, trauma surgeons, and anesthesiologists is not available. Drawbacks to this approach include the high rate of false positive and false negative results and the potential for iatrogenic cardiac injuries. Furthermore, pericardiocentesis is of limited use in treating tamponade because blood within the pericardial sac often is clotted and is not amenable to removal through a needle. Blunt cardiac injury typically involves a direct blow to the chest, usually sustained in a motor vehicle collision or a fall. Cardiac injuries generally are associated with sternal or rib fractures, although they may occur in the absence of any chest wall fracture; however, sternal fractures do not predict the presence of blunt cardiac injury. The most common location of blunt cardiac injury is the anterior heart, which consists primarily of the right ventricle. A blow that causes the sternum to exert a direct impact on the myocardium may result in direct injury to myocardial cells, sometimes leading to cell death, mechanical dysfunction, or dysrhythmias. The diagnosis of myocardial contusion is elusive. Many tests have been proposed, but none have proved definitive, except for direct visualization of the heart at surgery or autopsy. For practical purposes, the clinically significant sequelae of myocardial contusion are myocardial dysrhythmias and pump failure. Pump failure associated with cardiac contusion is usually the result of right heart failure in that most hemodynamically significant cardiac contusions are caused by injury to the anterior right ventricular free wall. Treatment of right heart failure from cardiac contusion consists of inotropic support and reduction of right ventricular afterload. Dysrhythmias secondary to cardiac contusion are treated in the same manner as dysrhythmias of any other etiology. In the rare patient with cardiorrhexia who presents to the hospital with signs of life, the most common injury is right atrial perforation.

A 19-year-old female involved in a motor vehicle collision is found to have a left tibial fracture and soft tissue injury requiring operative repair. In the postanesthesia care unit, she complains of pain that is unable to be relieved with narcotic analgesics. On palpation, she has a tense and swollen left leg. Passive flexion and extension of her foot increase her pain. Her dorsalis pedis and posterior tibial pulses are intact. She is subsequently taken back to the operating room for a decompressive fasciotomy. In a patient who is not awake or alert enough to complain of symptoms, what is the most accurate method of diagnosing compartment syndrome? A) Measurement of a compartment pressure of 10 mm Hg in the affected extremity B) Motor nerve stimulation to evaluate for muscle paralysis in the affected extremity C) Handheld Doppler ultrasonography to evaluate for pulselessness in the affected extremity D) Measurement of a compartment pressure of 15 mm Hg in the affected extremity E) A difference of less than 30 mm Hg between arterial diastolic pressure and compartment pressure in the affected extremity

Key Concept/Objective: To identify the signs, symptoms, and diagnostic criteria of extremity compartment syndrome Answer E is correct Compartment syndrome is defined as high-pressure swelling within a fascial compartment. Many physicians still believe, incorrectly, that compartment syndrome cannot develop in conjunction with an open fracture, because the break in the skin provides decompression. This is a dangerous assumption: compartment syndrome occurs in a significant number of patients with open fractures—for example, as many as 10% of patients with open tibial fractures. The most common cause of compartment syndrome is hemorrhage and edema in the damaged soft tissues seen with fractures. Other causes include a too-tight dressing or cast, disruption of the limb's venous drainage, advanced ischemia, and eschar from a circumferential burn. Multiply injured patients with hypovolemia and hypoxia are predisposed to compartment syndrome. The key to diagnosis of compartment syndrome is to maintain a high level of suspicion in any situation involving an extremity injury where there is a significant chance that this syndrome might develop (e.g., tibial fractures, forearm fractures, and all comminuted fractures associated with severe soft tissue injury). The diagnosis is primarily a clinical one, with the five Ps—pain, pallor, paresthesia, paralysis, and pulselessness—constituting the classic signs. The surgeon should not wait until all of these signs are present; the prognosis is much better if they are not. Severe ischemic muscle pain occurs that is unrelieved by the expected amounts of analgesia. On palpation, the compartment is tense and swollen, and passive stretching of the digits of the extremity increases the pain. Paresthesia occurs early and should be actively watched for; paralysis develops when ischemia has caused permanent damage. Pulselessness occurs late and is a relatively rare sign; it has been shown that irreversible damage can occur in a patient who still has palpable pulses. Measurement of compartment pressures is also employed in the diagnosis of compartment syndrome. Monitoring can be particularly helpful in patients who are not alert or are difficult to examine. There is no agreement on what constitutes the critical pressure threshold for a definitive diagnosis. An absolute value of 30 to 35 mm Hg has frequently been adopted as a diagnostic indicator; however, the evidence suggests that the difference between the diastolic arterial pressure and the pressure in the involved compartment (delta pressure, or Δp) is more important than any particular absolute value. Currently, a diagnosis of compartment syndrome is usually made if the Δp is less than 30 mm Hg, depending on the clinical signs and the level of suspicion. If compartment syndrome is suspected, the first step is to remove all circumferential bandages to relieve any pressure. If a plaster cast is present, it should be split, spread, or removed; if necessary, maintenance of reduction should be sacrificed. If the clinical picture does not improve after these measures are taken, then a decompressive fasciotomy is indicated.

A 32-year-old man was the unrestrained driver in a motor vehicle collision. He has gross hematuria. His workup reveals a fracture of the superior and inferior pelvic rami on the right, and a bladder injury is suspected. Which of the following is not true regarding traumatic bladder injuries? A) The majority of bladder injuries are extraperitoneal B) Retrograde CT cystography has a high sensitivity and specificity for bladder injury in blunt trauma C) Extraperitoneal bladder injuries can be treated with urinary catheter drainage D) If a patient undergoes laparotomy for another abdominal injury, extraperitoneal bladder injuries should be repaired in the stable patient E) Most intraperitoneal bladder perforations occur in the trigone

Key Concept/Objective: To learn evaluation and management of blunt bladder injuries Answer E is correct Bladder injuries occur in approximately 10% of blunt trauma patients with a pelvic fracture. Approximately two-thirds of these injuries are extraperitoneal. CT cystography has a 95% sensitivity and a 100% specificity for bladder injury. Extraperitoneal bladder injuries are typically treated with catheter drainage, but if a patient undergoes laparotomy for another injury and is hemodynamically stable, repair of extraperitoneal bladder injuries is recommended. Intraperitoneal bladder perforations typically occur in the dome of the bladder, not the trigone.

A 32-year-old man was the unrestrained driver in a motor vehicle collision. He has gross hematuria. His workup reveals a fracture of the superior and inferior pelvic rami on the right, and a bladder injury is suspected. Which of the following is not true regarding traumatic bladder injuries? A) The majority of bladder injuries are extraperitoneal B) Retrograde CT cystography has a high sensitivity and specificity for bladder injury in blunt trauma C) Extraperitoneal bladder injuries can be treated with urinary catheter drainage D) If a patient undergoes laparotomy for another abdominal injury, extraperitoneal bladder injuries should be repaired in the stable patient E) Most intraperitoneal bladder perforations occur in the trigone

Key Concept/Objective: To learn evaluation and management of blunt bladder injuries Answer E is correct Bladder injuries occur in approximately 10% of blunt trauma patients with a pelvic fracture. Approximately two-thirds of these injuries are extraperitoneal. CT cystography has a 95% sensitivity and a 100% specificity for bladder injury. Extraperitoneal bladder injuries are typically treated with catheter drainage, but if a patient undergoes laparotomy for another injury and is hemodynamically stable, repair of extraperitoneal bladder injuries is recommended. Intraperitoneal bladder perforations typically occur in the dome of the bladder, not the trigone.

A 48-year-old male suffers blunt abdominal trauma from a motorcycle accident. He is hemodynamically stable and undergoes an abdominal CT scan to assess for abdominal injuries. Which of the following is not true regarding management of renal injuries? A) Emergency revascularization should be performed for renal artery thrombosis if the other kidney is absent or injured B) Grade 1 and 2 injuries can typically be managed nonoperatively C) Exploratory laparotomy is required for all patients with urine extravasation on a CT scan D) Patients with grade 3 and 4 injuries should be closely monitored and followed with serial hematocrit levels E) Arterial embolization is an option for active arterial bleeding

Key Concept/Objective: To learn indications for operative and nonoperative management in renal injuries Answer C is correct. Renal injuries can typically be managed nonoperatively. All grade 1 and 2 renal injuries, regardless of the mechanism of injury, can be managed with observation alone because the risk of delayed bleeding is extremely low. Most grade 3 and 4 injuries, including those with devitalized parenchymal fragments and urinary extravasation, can be managed nonoperatively with close monitoring, serial hematocrit measurement, and repeat imaging in selected cases. Active arterial bleeding, in the absence of other associated injuries, can be treated with emergency arteriography and angioembolization. Thrombosis of the renal artery or its branches is treated expectantly unless the contralateral kidney is absent or injured, in which case, emergency revascularization is indicated.

A 62-year-old woman is involved in a motor vehicle collision and is hypotensive and tachycardic on arrival at the hospital. She has an obvious pelvic fracture on initial evaluation Which of the following is true regarding the evaluation of patients with pelvic trauma? A) Hypotensive patients with a pelvic fracture are unlikely to have a source of bleeding outside the pelvis B) A sheet wrap should not be placed on a patient with an unstable pelvic fracture because of the risk of worsening venous bleeding C) A FAST (focused assessment with sonography in trauma) examination should be performed in unstable patients to rule out intra-abdominal hemorrhage D) The treatment of arterial and venous pelvic hemorrhage is identical E) All of the above

Key Concept/Objective: To learn initial evaluation and management of hypotensive patients with unstable pelvic fractures Answer C is correct. Pelvic fractures are frequently associated with significant hemorrhage, not only because of the fracture itself but also because pelvic trauma is often accompanied by serious injuries to other parts of the body (e.g., the chest or the abdomen). The first step after diagnosing a pelvic fracture should be the immediate application of some type of external stabilization device (e.g., a sheet wrap). This approximates the fractured bones and decreases the volume of the pelvis, potentially reducing blood loss, particularly from the fractured bones and the lacerated venous plexus. This is not thought to increase venous bleeding. In hemodynamically unstable patients with clinical signs of a pelvic fracture, the next step should be the FAST examination to rule out a significant intra-abdominal bleeding source. If the FAST examination is negative and no other obvious sources of hemorrhage (e.g., chest or extremities) are found, the pelvis is the most likely source of the bleeding. Determining whether pelvic bleeding is arterial or venous is important as arterial bleeding can be treated with angioembolization.

62-year-old woman is involved in a motor vehicle collision and is hypotensive and tachycardic on arrival at the hospital. She has an obvious pelvic fracture on initial evaluation. Which of the following is true regarding the evaluation of patients with pelvic trauma? A) Hypotensive patients with a pelvic fracture are unlikely to have a source of bleeding outside the pelvis B) A sheet wrap should not be placed on a patient with an unstable pelvic fracture because of the risk of worsening venous bleeding C) A FAST (focused assessment with sonography in trauma) examination should be performed in unstable patients to rule out intra-abdominal hemorrhage D) The treatment of arterial and venous pelvic hemorrhage is identical E) All of the above

Key Concept/Objective: To learn initial evaluation and management of hypotensive patients with unstable pelvic fractures Answer C is correct. Pelvic fractures are frequently associated with significant hemorrhage, not only because of the fracture itself but also because pelvic trauma is often accompanied by serious injuries to other parts of the body (e.g., the chest or the abdomen). The first step after diagnosing a pelvic fracture should be the immediate application of some type of external stabilization device (e.g., a sheet wrap). This approximates the fractured bones and decreases the volume of the pelvis, potentially reducing blood loss, particularly from the fractured bones and the lacerated venous plexus. This is not thought to increase venous bleeding. In hemodynamically unstable patients with clinical signs of a pelvic fracture, the next step should be the FAST examination to rule out a significant intra-abdominal bleeding source. If the FAST examination is negative and no other obvious sources of hemorrhage (e.g., chest or extremities) are found, the pelvis is the most likely source of the bleeding. Determining whether pelvic bleeding is arterial or venous is important as arterial bleeding can be treated with angioembolization.

During an attempted robbery, a 42-year-old shop owner was stabbed in the midabdomen. He was hemodynamically unstable in the trauma bay and was taken emergently to the operating room and explored through a midline laparotomy. After packing all four quadrants and removing the packs systematically, he is noted to have a central hematoma inferior to the transverse mesocolon. Which of the following is not part of the exposure and vascular control of the infrarenal aorta? A) Cephalad retraction of the transverse mesocolon B) Mobilization of the ascending colon C) Dividing the midline retroperitoneum of the aortic bifurcation D) Opening of the retroperitoneum at the level of the left renal vein E) Evisceration of the small bowel to the patient's right

Key Concept/Objective: To learn steps in the exposure of the infrarenal aorta Answer B is correct. An injury to the inframesocolic abdominal aorta that is under a hematoma is controlled by performing the same maneuvers used to gain proximal control of an infrarenal abdominal aortic aneurysm. The infrarenal abdominal aorta is exposed by pulling the transverse mesocolon up toward the patient's head, eviscerating the small bowel to the right side of the abdomen, and opening the midline retroperitoneum until the left renal vein is exposed. A proximal aortic cross-clamp is then placed immediately inferior to the left renal vein. To expose the aortic bifurcation for distal control, the midline retroperitoneum is divided. Mobilization of the ascending colon does not provide exposure to the infrarenal aorta.

A 23-year-old male suffered a gunshot wound to the epigastrium and is taken emergently to the operating room. At laparotomy, a central hematoma is found medial to the kidneys and superior to the transverse mesocolon. Which of the following is correct? A) Temporary control should not be attempted with an aortic compression device because of the risk of esophageal injury B) The quickest method of gaining proximal control of the supraceliac aorta is by performing a left medial visceral rotation C) This is a zone 2 retroperitoneal hematoma D) Potential injured vessels include the aorta, celiac axis, proximal superior mesenteric artery, and proximal renal arteries E) To expose the proximal aorta through the lesser omentum, the esophagus should be retracted to the patient's right

Key Concept/Objective: To understand exposure to the proximal aorta Answer D is correct If active hemorrhage from the proximal abdominal aorta is encountered, the quickest way to perform aortic compression is by dividing the lesser omentum, retracting the esophagus and stomach to the patient's left, and compressing the aorta with an aortic compression device. Although there is some risk of esophageal injury, this is the most expeditious method of controlling acute hemorrhage. A left medial visceral rotation takes longer but does provide exposure of the entire abdominal aorta. Zone 1 hematomas are central hematomas, with zone 1 bounded laterally by the kidneys. Zone 2 comprises the lateral aspects of the retroperitoneum, from the kidneys lateral to the paracolic gutters. The retroperitoneal pelvis is zone 3. Potentially injured vessels in the supramesocolic area of zone 1 include the aorta, celiac axis, proximal superior mesenteric artery, and proximal renal arteries.

A 23-year-old male suffered a gunshot wound to the epigastrium and is taken emergently to the operating room. At laparotomy, a central hematoma is found medial to the kidneys and superior to the transverse mesocolon. Which of the following is correct? A) Temporary control should not be attempted with an aortic compression device because of the risk of esophageal injury B) The quickest method of gaining proximal control of the supraceliac aorta is by performing a left medial visceral rotation C) This is a zone 2 retroperitoneal hematoma D) Potential injured vessels include the aorta, celiac axis, proximal superior mesenteric artery, and proximal renal arteries E) To expose the proximal aorta through the lesser omentum, the esophagus should be retracted to the patient's right

Key Concept/Objective: To understand exposure to the proximal aorta Answer D is correct If active hemorrhage from the proximal abdominal aorta is encountered, the quickest way to perform aortic compression is by dividing the lesser omentum, retracting the esophagus and stomach to the patient's left, and compressing the aorta with an aortic compression device. Although there is some risk of esophageal injury, this is the most expeditious method of controlling acute hemorrhage. A left medial visceral rotation takes longer but does provide exposure of the entire abdominal aorta. Zone 1 hematomas are central hematomas, with zone 1 bounded laterally by the kidneys. Zone 2 comprises the lateral aspects of the retroperitoneum, from the kidneys lateral to the paracolic gutters. The retroperitoneal pelvis is zone 3. Potentially injured vessels in the supramesocolic area of zone 1 include the aorta, celiac axis, proximal superior mesenteric artery, and proximal renal arteries.

A 52-year-old man was involved in a motorcycle accident and suffered a femur fracture. In patients with long bone fractures of the lower extremity, what measure has been shown to decrease the incidence of developing fat embolism syndrome (FES)? A) Volume replacement B) Administration of corticosteroids C) Early stabilization of the fracture D) Administration of appropriate analgesics E) Respiratory support

Key Concept/Objective: To understand that the treatment of fat embolism is supportive and that it is best prevented by early fracture fixation Answer C is correct. Fat embolism syndrome (FES) is most commonly associated with fractures of long bones of the lower extremity. The classic clinical triad consists of respiratory distress, cerebral dysfunction, and petechial rash. The pathophysiology is not clear, but there is some evidence to suggest that extravasation of fat particles from long bone fractures may play an important part. Furthermore, early stabilization of long bone fractures has been shown to decrease the incidence of FES. Signs and symptoms of clinical FES usually begin within 24 to 48 hours after trauma. Treatment is primarily prophylactic and supportive, consisting of early fracture fixation, careful volume replacement, analgesia, and respiratory support. The role of corticosteroids in this setting is controversial.

An 18-year-old man was in an altercation and suffered multiple gunshot wounds to the abdomen. He is taken emergently to the operating room for an exploratory laparotomy without preoperative imaging. At laparotomy, a large hematoma is seen within the Gerota fascia on the left, and exploration of the left kidney is planned. Which of the following is true regarding operative management of renal injuries? A) Intraoperative IVP is not necessary in this situation B) Isolation and control of the renal artery and vein are best performed after opening the Gerota fascia C) If the kidney appears ischemic, surface cooling is indicated to preserve function D) Principles of management include control of hemorrhage and repair of any injuries to the collecting system E) To preserve renal function, the renal injury should be addressed before other intra-abdominal sources of bleeding are controlled

Key Concept/Objective: To understand the operative management of renal injuries Answer D is correct. Intraoperative IVP is indicated when exploration of a kidney is planned and no preoperative imaging is available. The main purpose of one-shot IVP in this setting is to confirm the presence of a contralateral functioning kidney; a potential benefit is the ability to rule out major injury. Although somewhat controversial, most evidence suggests that the renal artery and vein should be controlled prior to opening the Gerota fasica. Surface cooling is not recommended as it adds time to the operation and can contribute to hypothermia. The principles of operative management of renal injuries are control of bleeding followed by repair of injuries to the collecting system. The renal injury should not be addressed until other bleeding sources in the abdomen have been controlled.

A 56-year-old female pedestrian was crossing a busy intersection when she was struck by a motor vehicle traveling at a moderate rate of speed. She was unconscious at the scene, with a blood pressure of 82/51 mm Hg. She was intubated and brought to the emergency department, where a trauma evaluation identified the following injuries: a subarachnoid hemorrhage, C5-C7 transverse process fractures, multiple left-sided rib fractures, a left pneumothorax, a grade IV splenic laceration, and a closed, complex midshaft femur fracture. A left-sided chest tube was immediately placed, and she received 2 L of intravenous (IV) fluids as well as 2 units of packed red blood cells. Despite these measures, she remained hypotensive and was taken to the operating room for an exploratory laparotomy. In the operating room, she underwent a splenectomy and placement of an intracranial pressure monitor. During the initial operation, she developed acidemia and hypothermia. As a result, she was taken to the intensive care unit with an open abdomen. In the intensive care unit, she underwent further resuscitation and was warmed, but she remained acidemic. The next day, she developed hypotension and was found to have a significant drop in her hematocrit. She returned to the operating room, where 1 L of blood clot was evacuated from her left upper quadrant and a bleeding short gastric vessel was identified and ligated. Following this operation, her condition stabilized. At what point should this patient's femur fracture undergo intramedullary nailing for definitive repair? A) During the initial operation B) 3 to 7 days after her initial injury C) 7 to 10 days after her initial injury D) 2 to 3 weeks after her initial injury E) 4 to 6 weeks after her initial injury

Key Concept/Objective: To understand the timing of operative treatment of injuries involving the extremities in patients with multiple trauma Answer C is correct. In multiply injured patients, the timing of operative treatment of injuries to the pelvis and extremities depends on both the condition of the patient and the particular combination of skeletal and soft tissue injuries sustained. In such cases, the threshold for adoption of a damage-control strategy to minimize operating time and tissue insult should be low. The performance of multiple definitive osteosyntheses is an option only in patients who have been successfully resuscitated (as indicated by stable hemodynamic status without a need for inotropes; absence of hypoxia, hypercapnia, and acidosis; normothermia; normal coagulation parameters; and normal diuresis). In other cases, the timing and extent of operative treatment of musculoskeletal injuries may be more complex. Generally, if open reduction and internal fxation are indicated, the sooner the operation is performed, the better. With early operation, fracture surfaces are more easily cleaned of blood clots and other material, and reduction is facilitated by the absence of prolonged dislocation and shortening. After 6 to 8 hours, swelling develops, making both the operation and the subsequent closure more difficult and thereby increasing the risk of infection and other wound complications. There are some cases, however, in which it might be preferable to postpone a complex articular reconstruction to ensure reasonable provisional reduction; however, complete joint dislocations are not acceptable. Clinically, sufficient reduction of swelling has occurred when the skin has regained its creases and is wrinkled over the operative site. Early definitive surgery may also be contraindicated when abrasion or degloving injury is present at the fracture site on admission. Swelling is less of a problem with shaft fractures that will be treated with intramedullary nailing. If, however, it is not possible to perform nailing within 24 to 48 hours after the injury, it is better to postpone the operation for 7 to 10 days; patients operated on between days 3 and 7 are at higher risk for acute respiratory distress syndrome (ARDS). If the procedure must be delayed for more than 2 to 3 weeks (e.g., because of sepsis and organ failure), reconstruction of bones and joints will be substantially more difficult. If such delay leads to suboptimal axial alignment and nonanatomic reconstruction of the articular surface, the long-term prognosis will be worse.

A 62-year-old woman is involved in a motor vehicle collision and is hypotensive and tachycardic on arrival at the hospital. She has an obvious pelvic fracture on initial evaluation. The patient stabilizes with volume resuscitation and is taken to the computed tomographic (CT) scanner. A CT scan shows contrast extravasation in the pelvis. Which of the following is not true regarding pelvic arterial bleeding? A) Angiographic embolization is frequently successful in controlling arterial hemorrhage B) Contrast extravasation on a CT scan is highly sensitive and specific for the need for angiographic embolization C) Angiography is typically performed through the femoral artery D) External fixation devices rarely result in cessation of arterial bleeding E) Angiography should be performed before laparotomy in a patient with both pelvic and intra-abdominal arterial hemorrhage

Key Concept/Objective: To understand the treatment of pelvic arterial bleeding Answer E is correct Angiographic embolization has a success rate of up to 90% for controlling pelvic arterial hemorrhage. It is typically performed through the femoral artery. Contrast extravasation on a CT scan is highly sensitive and specific for pelvic arterial injury requiring angiographic embolization. External fixation devices and surgical retroperitoneal packing are useful in treating venous bleeding but typically do not adequately control arterial bleeding. In a patient with hemorrhage from both intra-abdominal and pelvic sources, the intra-abdominal source should be controlled at laparotomy followed immediately by angiographic embolization of the pelvic arterial source.

A 46-year-old man went ice fishing but drank a significant amount of alcohol and passed out on the ice for an hour until he was found by a friend. He is brought to the hospital and admitted to the burn intensive care unit with second-degree frostbite. Which of the following interventions should not be undertaken in the initial management of this patient? A) Administration of prophylactic antibiotics B) Sterile cotton placed between toes or fingers to prevent skin maceration C) Immersion of frostbitten tissue in a large water bath of 40° to 42°C (104° to 108°F) D) Avoidance of dry heat E) Administration of toxoid booster as indicated by the immunization history

To demonstrate understanding of the key principles of the initial management of frostbite Answer A is correct The emergency department treatment of a frostbite victim should first focus on the basic ABCs of trauma resuscitation and on identifying and correcting systemic hypothermia. Frostbitten tissue should be immersed in a large water bath of 40° to 42°C (104° to 108°F). The bath should be large enough to prevent rapid loss of heat, and the water temperature should be maintained. This method of rapid rewarming may significantly decrease tissue necrosis caused by full-thickness frostbite. Dry heat is not advocated, because it is difficult to regulate and places the patient at risk for a burn injury. The rewarming process should take about 30 to 45 minutes for digits. The affected area appears flushed when rewarming is complete and good circulation has been reestablished. Narcotics are required because the rewarming process may be quite painful. The skin should be gently but meticulously cleansed and air-dried, and the affected area should be elevated to minimize edema. A tetanus toxoid booster should be administered as indicated by the immunization history. Sterile cotton should be placed between toes or fingers to prevent skin maceration, and extreme care should be taken to prevent infection and to avoid even the slightest abrasion. Prophylactic antibiotics and dermal blister débridement are both controversial; most clinicians débride blisters and reserve antibiotics for identified infections. A 2005 study reported a lower than expected digit amputation rate when rapid rewarming was followed by treatment with heparin and tissue plasminogen activator (tPA), but, to date, this finding has not been confirmed by other studies.

An 18-year-old ejected driver of a high-speed rollover motor vehicle accident presents with stridor and subcutaneous emphysema of the neck. What is the preferred method of securing an airway in this patient? A) Intubation over a flexible bronchoscope B) Blind intubation C) ED cricothyrotomy D) ED tracheostomy E) OR tracheostomy

To demonstrate understanding of airway management with tracheal injury Answer E is correct With tracheobronchial injuries, as with all injuries, airway management is the first priority in treatment. If the patient is maintaining his or her own airway and is adequately ventilated, a cautious, noninterventional approach is probably the best initial choice until further diagnostic workup is performed or other life-threatening injuries are stabilized. Careless handling or mishandling of the airway (e.g., inadvertently placing an endotracheal tube through a transected or ruptured airway and into soft tissue) can be disastrous and may compound the injury. ED tracheotomies are difficult and may be dangerous, to say the least. How best to secure an airway in a patient with neck trauma and possible tracheal injury is a matter of debate. With blind endotracheal intubation, the path of the tube distal to the larynx is unknown, and it is possible to lose the lumen or create a false passage. With intubation over a flexible bronchoscope, the tube can be visualized as it passes beyond the site of injury, and some of the dangers of blind intubation are thereby mitigated; however, some degree of sedation is usually required, and if the patient is oversedated, the airway that was being spontaneously protected may be lost. Paralytic medications should generally be avoided in this setting for the same reason. Urgent tracheotomy or cricothyrotomy, performed in the OR, is advocated by many as the safest and securest way of obtaining airway control. If the trachea is completely transected, the distal trachea can usually be found in the superior mediastinum and grasped for insertion of a cuffed tube. The approach taken to airway control must vary with the resources and expertise available at each institution. One must also keep in mind that even after the airway is secured, it may still be possible to exacerbate the injury by means of aggressive ventilation with high airway pressures. Tube thoracostomies should be appropriately placed at this time and connected to suction, even though dyspnea may worsen.

Shortly after arrival at the trauma bay, the patient from question 4 becomes hypotensive and tachycardic, and you are concerned about intra-abdominal injury. Which of the following is least likely to be injured in a pregnant patient? A) Liver B) Spleen C) Gastrointestinal tract D) Uterus E) Bladder

To demonstrate understanding of blunt injury in pregnant patients Answer C is correct In the mother, blunt trauma (as in motor vehicle accidents) may cause multiple life-threatening injuries. Pelvic fractures are a particular concern: because of the extensive vascular supply in this area, there is a significant risk of substantial hidden blood loss. Because the uterus consumes approximately 20% of the cardiac output (i.e., 500 to 600 mL/min), a uterine injury can place a pregnant woman at substantial risk for massive hemorrhage within a short period. Uterine expansion displaces the bladder into the abdomen, thereby increasing the risk of traumatic bladder injury. The upper urinary tract is generally spared from injury, however, because the gravid uterus shields the retroperitoneum from direct injury. Hepatic, splenic, and uterine injuries are common in high-speed motor vehicle accidents, but injuries to the gastrointestinal tract, surprisingly, appear to be uncommon, largely because of the protective effects of the gravid uterus. Direct injury to the fetus as a result of blunt trauma is unusual because of the protection afforded by the maternal abdominal wall and the uterus. Blunt trauma may result in fetal skull fractures, fetal intracranial hemorrhage, or abruptio placentae; nevertheless, fetal demise is rare in this setting and usually is secondary to maternal demise.

Shortly after arrival at the trauma bay, the patient from question 4 becomes hypotensive and tachycardic, and you are concerned about intra-abdominal injury. Which of the following is least likely to be injured in a pregnant patient? A) Liver B) Spleen C) Gastrointestinal tract D) Uterus E) Bladder

To demonstrate understanding of blunt injury in pregnant patients Answer C is correct. In the mother, blunt trauma (as in motor vehicle accidents) may cause multiple life-threatening injuries. Pelvic fractures are a particular concern: because of the extensive vascular supply in this area, there is a significant risk of substantial hidden blood loss. Because the uterus consumes approximately 20% of the cardiac output (i.e., 500 to 600 mL/min), a uterine injury can place a pregnant woman at substantial risk for massive hemorrhage within a short period. Uterine expansion displaces the bladder into the abdomen, thereby increasing the risk of traumatic bladder injury. The upper urinary tract is generally spared from injury, however, because the gravid uterus shields the retroperitoneum from direct injury. Hepatic, splenic, and uterine injuries are common in high-speed motor vehicle accidents, but injuries to the gastrointestinal tract, surprisingly, appear to be uncommon, largely because of the protective effects of the gravid uterus. Direct injury to the fetus as a result of blunt trauma is unusual because of the protection afforded by the maternal abdominal wall and the uterus. Blunt trauma may result in fetal skull fractures, fetal intracranial hemorrhage, or abruptio placentae; nevertheless, fetal demise is rare in this setting and usually is secondary to maternal demise.

A 59-year-old woman is brought to the ED after being the victim of a mugging. She is hemodynamically stable but confused. On examination, she has a large hematoma over her left frontal skull and linear ecchymoses over the left neck. Which of the following statements is true regarding blunt injuries to the vasculature of the neck? A) Most injuries are the result of direct trauma to the neck B) Traditional angiography should be used as a method of screening for vascular injury C) All high-grade injuries need surgical intervention D) Complications of such injuries include pseudoaneurysm, dissection, and emboli E) All patients with such injures should receive antiplatelet or anticoagulation agents

To demonstrate understanding of management of blunt injuries to the vasculature of the neck Answer D is correct Blunt injuries to cervical arteries, although relatively rare, can be difficult to diagnose and can lead to devastating complications (e.g., stroke and bleeding). Most such injuries result from stretching of the vessels as a consequence of hyperextension of the neck. Motor vehicle collision is a common cause of this sort of stretch injury. Injury may also be caused by direct trauma to the artery, such as with a seat belt or by fracture fragments, particularly cervical transverse foramen or basilar skull fractures. Arterial stretching leads to endothelial tearing, with subsequent intimal flaps, dissections, or emboli. Four-vessel angiography has historically been the gold standard for imaging the cervical vasculature but is invasive and expensive and has largely been replaced as a screening modality by CT angiography. CT angiography is noninvasive, can be performed relatively rapidly, and can be done in the course of CT scanning of the head and other parts of the body. Improvements in CT technology, particularly the use of multidetector CT scanners, have resulted in sensitivity and specificity rates similar to those of formal angiography. Biffl and colleagues developed a grading scale for blunt cervical arterial injury based on the angiographic appearance of the lesion. On this scale, grade I and II lesions show less than 25% and greater than 25% luminal narrowing, respectively; grade III lesions are pseudoaneurysms; grade IV lesions demonstrate thrombosis; and grade V lesions are transections with extravasation. Grade I to III lesions may progress to a higher grade over time. Treatment of blunt cervical arterial injury reduces morbidity and mortality. Groups in Denver and Memphis have shown a dramatic decrease in stroke rates (54% to 2.6% and 64% to 6.8%, respectively) for carotid artery injuries in treated versus untreated patients. Although treatment of some sort seems to be beneficial, the exact means of treatment is debated. The primary goal of treatment is to decrease embolic events, and the most common treatment is some means of interfering with clotting, which can be challenging in trauma patients with contraindications to anticoagulation, such as intracranial hemorrhage, solid-organ, or pelvic injuries. The risks and benefits of anticoagulation versus simple observation and expectant management need to be weighed. Asymptomatic patients with small intimal flaps or minimal dissection who have no contraindication to antiplatelet agents should be given aspirin. If a contraindication to aspirin exists, the patient should be treated expectantly. More than half of low-grade injuries heal within 2 weeks, and anticoagulation can be stopped after 14 days. If the patient is asymptomatic but has an acute dissection with no contraindications to anticoagulation, then either heparin or antiplatelet agents should be given. Patients with neurologic symptoms need to be anticoagulated.

A 45-year-old man sustained multiple stab wounds to his neck, chest, and abdomen in an altercation. On arrival at the trauma bay, he is hemodynamically unstable. Emergency release blood is called for, and he is being intubated. On examination, he has pulsatile blood spurting from a zone II neck wound when the paramedic removes his hand. In addition, he does not have left-sided breath sounds and his abdomen is tensely distended. Which is the proper management of a left common carotid laceration in this situation? A) Exploration through a supraclavicular incision and ligation of the common carotid artery at its takeoff from the brachiocephalic artery B) Exploration through an incision along the anterior border of the sternocleidomastoid (SCM) muscle and ligation of the common carotid artery C) Exploration through an incision along the anterior border of the SCM muscle and repair of the laceration D) Exploration through an incision along the anterior border of the SCM muscle and repair using a saphenous vein patch angioplasty E) None of the above

To demonstrate understanding of management of penetrating injuries to the common carotid artery Answer B is correct. Management of common or external carotid artery injuries is governed by the extent of the injury and the overall status of the patient. Simple injuries to the external carotid artery should be repaired, whereas complex injuries should be ligated given that the external carotid artery normally has such good collateral flow. Even in cases with complex injuries to the common carotid artery, if the patient has no neurologic deficits, ligation can be done. The internal carotid will be reconstituted with flow from the external carotids. If an injury to the common carotid artery or the internal carotid artery has interrupted flow in the vessel, repair creates a theoretical disadvantage. Interruption of flow may lead to focal brain ischemia and partial disruption of the blood-brain barrier. Sudden restoration of blood flow may cause hemorrhage in the area of the ischemia and worsen the extent of brain injury; an anemic, or white, infarct of the brain may be converted to a hemorrhagic, or red, infarct. Whether this pathophysiology is important after traumatic injury is unclear and controversial. Deciding whether to repair or ligate when flow has been interrupted is often difficult. One approach is to base the decision on the patient's preoperative neurologic status. If there is no neurologic deficit, it is presumed that there are no areas of brain ischemia and that repair is safe. Conversely, a focal neurologic deficit is presumed to be related to ischemia, and in such cases, the risk of worsening the patient's neurologic status with restoration of blood flow is increased. Even though this approach is rational, it is not applicable in cases in which a detailed neurologic examination before surgery is not possible. Furthermore, this approach may be applicable only to patients in coma or with severe neurologic deficits. There are indications that milder neurologic deficits respond favorably to revascularization. Yet another approach is to gauge the appropriateness of repair according to the nature of the injury itself. In this approach, large, complicated injuries requiring involved and lengthy procedures for repair are ligated, whereas simple injuries requiring only simple and quick repairs are repaired. Similarly, repair is not indicated in patients with severe or multiple associated injuries. There is also a difference between the management of injuries of the common carotid artery and the management of injuries of the internal carotid artery. Common carotid injuries are more accessible and easier to repair, and repair is generally associated with a good outcome. Continued antegrade flow in the internal carotid artery is more likely after injury to the common carotid artery than after injury to the internal carotid artery because of the possibility of collateral flow via the external carotid artery. A reasonable way to deal with repair dilemmas is to make the decision on the basis of distal back-bleeding. Interruption of blood flow to the brain is tolerated only for a short time, and restoration of flow is unlikely to be accomplished quickly enough to improve outcome. It is therefore logical to base the decision about revascularization on the state of back-bleeding from the internal carotid artery. If back-bleeding is brisk, the patient is presumed to have good collateral flow, and the chances that there is an area of ischemia are low. Repair rather than ligation is safe in such circumstances. If internal carotid artery back-bleeding is minimal or absent, an ischemic infarct is more likely and restitution of arterial inflow is more dangerous. A corollary to this reasoning is that if back-bleeding is poor, a clot distal to the area of injury may be present, and return of flow with repair may dislodge the clot distally.

A 38-year-old woman was the victim of domestic abuse when she was stabbed in the right mid- and upper neck. She is transported by the emergency medical service to the ED and rapidly moved to the operating room after examination demonstrated findings consistent with a vascular injury. During an internal carotid artery exploration for bleeding, which of the following structures should not be sacrificed? A) Mastoid process B) Occipital artery C) Ascending ramus of the mandible D) Hypoglossal nerve E) Ansa cervicalis

To demonstrate understanding of the anatomy of carotid artery exploration Answer D is correct For wounds to the distal internal carotid artery, exposure can be difficult. A 3 to 5 French Fogarty balloon catheter can be placed through the area of injury or through a proximal arteriotomy to help with distal control. The catheter should be advanced distally and the balloon inflated to provide a dry field for arterial repair. Repair can be done around the catheter; the balloon is deflated near the conclusion of the repair, and the catheter is removed before the final several sutures are tied. If exposure of the distal internal carotid artery is needed, a number of important structures should be identified and protected. The hypoglossal nerve is usually encountered within several centimeters of the carotid bifurcation and should be dissected free of the internal carotid and retracted upward. This is facilitated by division of the occipital artery, which crosses superficial to the hypoglossal nerve on its course from the external carotid artery toward the occiput. It is also helpful to divide the ansa cervicalis branches that run inferiorly from the hypoglossal nerve to supply the muscles of the neck. Injury to the hypoglossal nerve results in impaired motor function of the tongue and can lead to dysarthria and dysphasia. Injury to or sectioning of the ansa cervicalis causes little or no morbidity. Further distal exposure of the internal carotid artery may require unilateral mandibular subluxation or division of the ascending ramus of the mandible. Such maneuvers are somewhat easier when the patient is nasotracheally intubated. They increase the size of the small area immediately behind the condyle and allow easier division of the stylohyoid ligament and the styloglossus and stylopharyngeus muscles. These three structures can be divided together adjacent to their common origin at the styloid process. If this is done, care should be taken to preserve the facial nerve, which lies superficial to these muscles and must be dissected free of the muscles before they are divided. The underlying glossopharyngeal nerve, which lies deep to these muscles and superficial to the internal carotid artery, should also be protected by dissecting it free of the muscles before their division. Injury to the facial nerve results in loss of function of the muscles of facial expression. If the glossopharyngeal nerve is injured, loss of motor and sensory supply to parts of the tongue and pharynx increases the risk of aspiration. Once the muscles originating from the styloid process have been divided, the styloid process itself can be resected to gain a further short distance of distal exposure. In very rare instances, it may prove useful to remove portions of the mastoid bone to provide even more distal exposure of the internal carotid artery as it enters the carotid canal. For more distal lesions, it is necessary to place a posterolateral scalp incision, reflect a medially based scalp flap, and divide the ipsilateral external auditory canal. This approach results in better exposure of the mastoid process and allows exposure of the intrapetrous portion of the internal carotid after removal of the overlying bone of the mastoid with a high-speed bone drill.

A 33-year-old female is concerned about a small "growth" on her left forearm. On examination, you find a 6 mm bluish-black flat lesion with irregular borders. The appropriate next step in management is: A) Close observation B) Incisional biopsy with depth of excision reaching the subcutaneous fat layer C) Incisional biopsy with depth of excision reaching the dermal layer D) Excisional biopsy with depth of excision reaching the subcutaneous fat layer E) Excisional biopsy with depth of excision reaching the dermal layer

To demonstrate understanding of the appropriate technique for biopsy of skin lesions Answer D is correct Any suspicious skin lesion should be biopsied. This can be performed by excisional biopsy if the lesion is small or incisional biopsy if the lesion is large. Excisional biopsy should be performed by incorporating a 1 to 4 mm margin of normal skin surrounding the lesion depending on the clinical characteristics. This may eliminate the need for subsequent reexcision for some types of lesions (dysplastic nevi). However, no attempt should be made to perform a definitive radical excision until a diagnosis is established by biopsy. Full-thickness excision into the subcutaneous fat should be performed and the margins should be marked for orientation.

You are the on-call surgery resident in the pediatric emergency department of a regional children's hospital. A 6-year-old boy is brought in as the victim of blunt trauma sustained during a motor vehicle collision. When considering pediatric trauma, which of the following general principles is false? A) The nutritional requirements of injured children are greater than those of injured adults B) Gastric dilatation, which can result in vomiting and pulmonary aspiration, is very common in young children after all forms of trauma C) Children can compensate for hypovolemia effectively and keep their vital signs in the normal range, even in the presence of shock D) Falls are the most common cause of trauma in the pediatric population E) A small blood loss that would be insignificant in an adult can result in marked hemodynamic changes in a small child

To demonstrate understanding of the basic principles of pediatric trauma Answer D is correct Although the general principles of trauma care are essentially the same for children as for adults, several significant differences must be taken into account in the care of pediatric accident victims. For example, children do not react to trauma in the same way as adults do. They often have difficulty in expressing pain and in articulating their complaints. They are often extremely frightened after an accident, and this fear may cause them to give misleading signals (e.g., by exhibiting signs of an acute abdomen even though no intra-abdominal injury has occurred). Another key difference between children and adults is that children are still growing. Metabolic management after any form of stress, whether from a surgical procedure, trauma, or some other event, must take this difference into account. Children can compensate for hypovolemia effectively and keep their vital signs in the normal range, even in the presence of shock. However, a small blood loss that would be insignificant in an adult can result in marked hemodynamic changes in a small child. Moreover, water and heat loss can be far more extensive in small children than in older children and adults because smaller children have a greater surface area in relation to their weight and have a relative lack of insulating subcutaneous fat. Hypothermia aggravates acidosis and makes hemodynamic resuscitation much more difficult. Gastric dilatation, which can result in vomiting and pulmonary aspiration, is very common in young children after all forms of trauma. Finally, the nutritional requirements of injured children are greater than those of injured adults because children, as growing organisms, naturally have a high metabolic rate. Consequently, total parenteral nutrition often must be started earlier in the treatment of a child than it would be in the treatment of an adult in a comparable condition. Not only are there significant physiologic and psychological differences between children and adults after trauma, but there also are differences in accident patterns. Most childhood injuries result from blunt trauma. Head trauma is far more common in children than in adults; in fact, it accounts for most of the morbidity and mortality in the pediatric population. After motor vehicle accidents, which are the major cause of trauma in both children and adults, the next most frequent causes of trauma in children are events that are less important causes in adults: falls, bicycle accidents, drowning, poisoning, and burns from fires. Nonaccidental trauma is a unique and important cause of trauma in children.

A 45-year-old gentleman is undergoing excision of a melanoma lesion of the lower extremity, as well as an SLNB. Frozen section of the lymph node returns positive for melanoma. Which of the following treatment statements is true? A) SLNB has clearly been shown to demonstrate an overall survival benefit in patients with melanoma B) The evidence for completion lymphadenectomy in a patient with a positive SLNB clearly supports a therapeutic benefit C) There is a role for therapeutic lymph node dissection in patients with bulky lymphadenopathy D) Patients with in-transit lymph node metastases have a favorable prognosis E) Adjuvant therapy with interferon alfa is not associated with improvement in disease-free survival

To demonstrate understanding of the management of lymph node metastases in melanoma Answer C is correct. Therapeutic lymph node dissection is currently recommended for management of the regional lymph node drainage basin in the presence of a positive sentinel node and in clinical stage III disease. Some challenge this recommendation as four randomized trials of elective lymph node dissection failed to demonstrate an overall survival benefit. Yet the majority of patients in these studies did not have nodal disease; thus, the trials were not sufficiently powered to detect a small survival benefit. Other trials have supported this practice, including the World Health Organization trial, which on subgroup analysis demonstrated a significantly improved 5-year survival rate in patients with occult nodal metastases detected at elective lymph node dissection compared with patients who had delayed lymphadenectomy at the time when they developed palpable nodal metastases (48% versus 27%, respectively; p = .04). Nonetheless, the impact of completion lymphadenectomy on overall survival is still a matter of debate. The results of the first Multicenter Selective Lymphadenectomy Trial (MSLT-I) suggest that SLNB with immediate completion lymphadenectomy if the sentinel node is positive improves disease-free survival but not overall survival for these patients. Although there were suggestions based on the data that a survival benefit might be gained by lymphatic mapping and SLNB followed by completion lymph node dissection in the event of a positive node, their statistical analysis has been criticized. There is, however, clearly a role for therapeutic lymph node dissection in patients with bulky nodal disease, and it can be performed with minimal morbidity and good palliation. Patients with in-transit metastases have an unfavorable prognosis. This reflects a disseminated stage of disease, with a 5-year survival rate of 25 to 30%. Surgical excision is the mainstay of therapy when the size and the number of lesions permit, although there is no formal recommendation regarding the extent of surgical margin. Adjuvant therapy with interferon alfa 2b is associated with improvement in disease-free survival but is also associated with significant toxicity.

34-year-old woman 20 weeks pregnant presents to the hospital as a victim of blunt trauma sustained in a motor vehicle collision. Which of the following general principles concerning trauma in this pregnant patient is not true? A) Trauma is believed to occur in approximately 6 to 7% of gestations, complicating as many as one in 12 pregnancies B) Motor vehicle accidents are the most common cause of traumatic maternal death C) When the mother survives, fetal death is most commonly related to abruptio placentae D) Penetrating trauma during the latter portions of the pregnancy has a high risk of fetal injury E) Twenty percent of abused pregnant women attempt suicide or abuse alcohol or drugs

To demonstrate understanding of the basic principles of trauma in pregnant women Answer B is correct Trauma is believed to occur in approximately 6 to 7% of gestations, complicating as many as one in 12 pregnancies. Trauma remains the leading cause of maternal death. Homicide is the most common cause of traumatic maternal death, followed by motor vehicle accidents, accidental injury, and suicide. The more severe the injury to the mother, the greater is the risk of injury to the fetus. When the mother survives, fetal death is most commonly related to abruptio placentae. Major trauma causes placental abruption in 40 to 66% of cases; minor trauma causes placental abruption in 5% of cases. Signs and symptoms of abruption typically become manifest within 4 hours of observation; however, a delayed presentation may also occur. Penetrating trauma during the latter portions of the pregnancy invariably causes fetal damage. Domestic or partner violence is common during pregnancy, affecting 7 to 23% of all pregnancies, and thus is a common source of traumatic injuries. If the patient is in an abusive relationship, the severity and frequency of assault typically increase during pregnancy. In addition, 20% of abused pregnant women attempt suicide or abuse alcohol or drugs. Unfortunately, partner violence often goes undiagnosed and hence untreated. If a suspicion of partner violence arises, the physician should ask direct questions about threats and should provide appropriate contacts and support. In all 50 states, domestic violence is a crime, and in some, failure to report suspected violence is a crime as well.

A 34-year-old woman 20 weeks pregnant presents to the hospital as a victim of blunt trauma sustained in a motor vehicle collision. Which of the following general principles concerning trauma in this pregnant patient is not true? A) Trauma is believed to occur in approximately 6 to 7% of gestations, complicating as many as one in 12 pregnancies B) Motor vehicle accidents are the most common cause of traumatic maternal death C) When the mother survives, fetal death is most commonly related to abruptio placentae D) Penetrating trauma during the latter portions of the pregnancy has a high risk of fetal injury E) Twenty percent of abused pregnant women attempt suicide or abuse alcohol or drugs

To demonstrate understanding of the basic principles of trauma in pregnant women Answer B is correct. Trauma is believed to occur in approximately 6 to 7% of gestations, complicating as many as one in 12 pregnancies. Trauma remains the leading cause of maternal death. Homicide is the most common cause of traumatic maternal death, followed by motor vehicle accidents, accidental injury, and suicide. The more severe the injury to the mother, the greater is the risk of injury to the fetus. When the mother survives, fetal death is most commonly related to abruptio placentae. Major trauma causes placental abruption in 40 to 66% of cases; minor trauma causes placental abruption in 5% of cases. Signs and symptoms of abruption typically become manifest within 4 hours of observation; however, a delayed presentation may also occur. Penetrating trauma during the latter portions of the pregnancy invariably causes fetal damage. Domestic or partner violence is common during pregnancy, affecting 7 to 23% of all pregnancies, and thus is a common source of traumatic injuries. If the patient is in an abusive relationship, the severity and frequency of assault typically increase during pregnancy. In addition, 20% of abused pregnant women attempt suicide or abuse alcohol or drugs. Unfortunately, partner violence often goes undiagnosed and hence untreated. If a suspicion of partner violence arises, the physician should ask direct questions about threats and should provide appropriate contacts and support. In all 50 states, domestic violence is a crime, and in some, failure to report suspected violence is a crime as well.

A 44-year-old male presents to your office with the complaint of a mass in his right thigh. On examination, you find a hard 8 cm mass in the medial compartment of the thigh that appears deep, nonmobile, and nontender. What is the most appropriate strategy for diagnosis? A) Excisional biopsy B) Fine-needle aspiration C) Core-needle biopsy (CNB) D) Incisional biopsy E) Magnetic resonance imaging (MRI)

To demonstrate understanding of the diagnosis of soft tissue masses Answer C is correct Soft tissue sarcomas (STSs) frequently present as painless large masses. Approximately 38% are larger than 10 cm at presentation. The optimal imaging modality remains a matter of debate, but it is accepted that for extremity lesions, MRI provides more information than CT. Diagnosis is attained by pathology. A mass that is larger than 5 cm, is growing, or has persisted for longer than 4 weeks constitutes an indication for biopsy. In choosing the method, location, and orientation of the biopsy, it is critical to consider possible future surgical interventions. For masses larger than 5 cm, either an incisional biopsy or a CNB is appropriate. Incisional biopsies increase the possibility of tumor spread through the tissue planes of the incision. CNB is therefore preferable for evaluation of soft tissue masses. Several studies have shown CNB to have very high sensitivity and specificity with respect to both malignant and benign soft tissue masses. For most masses smaller than 5 cm, an excisional biopsy with a clear surgical margin is indicated. Fine-needle aspiration is not a useful modality in the workup of soft tissue masses, because it does not provide a large enough sample to permit determination of histologic architecture and tumor grade.

A 24-year-old man is transported to the emergency department (ED) after being stabbed in the chest two times. He was intoxicated and hemodynamically stable at the scene, but on arrival at the ED, a pulse cannot be found. Examination reveals two 3 cm lacerations over the left chest at the level of the nipple in the anterior and midaxillary line. After securing an airway and while resuscitation is being performed, an emergency department resuscitative thoracotomy (EDRT) is performed. Which of the following is not one of the six therapeutic goals of EDRT? A) Relief of cardiac tamponade B) Cross-clamping the pulmonary hilum to control hemorrhage C) Repairing an injury to the left pulmonary artery D) Cross-clamping the descending aorta for lower extremity hemorrhage E) Confirming endotracheal intubation

To demonstrate understanding of the goals of emergency department (ED) resuscitative thoracotomy Answer C is correct The technique of EDRT ideally requires that an antiseptic solution be splashed on the chest, but skin preparation is not required. A left anterolateral thoracotomy incision is made from the sternal border to the midaxillary line in the fourth intercostal space. A chest retractor is inserted and opened widely. The costochondral junctions of the fifth, the fourth, and sometimes the third rib are divided quickly with the scalpel to provide exposure. The six main therapeutic goals are (1) confirming endotrachal intubation; (2) control of hemorrhage; (3) relief of cardiac tamponade if present; (4) effective cardiac compression; (5) cross-clamping the pulmonary hilum in the case of major lung hemorrhage, air embolism, or massive bronchopleural fistula; and (6) cross-clamping of the descending aorta for lower torso hemorrhage control. Attention is directed first to the injury. If there is exsanguination from a great vessel, the hemorrhage is controlled with pressure. If air embolism is the cause of the arrest, the hilum is clamped and air is evacuated from the aorta. Otherwise, the pericardium is opened anterior and parallel to the phrenic nerve. The hemopericardium is evacuated, the cardiac injury is controlled with digital pressure, and a temporary repair is performed. After the cause of the arrest has been addressed, the descending thoracic aorta is occluded with a vascular clamp or digital pressure and intrathoracic cardiac compression is initiated. The patient's intravascular volume is restored, and electrolyte imbalances are corrected. If the patient can be saved, he or she is transported to the operating room (OR) for definitive repair and closure.

A 46-year-old man went ice fishing but drank a significant amount of alcohol and passed out on the ice for an hour until he was found by a friend. He is brought to the hospital and admitted to the burn intensive care unit with second-degree frostbite. Which of the following interventions should not be undertaken in the initial management of this patient? A) Administration of prophylactic antibiotics B) Sterile cotton placed between toes or fingers to prevent skin maceration C) Immersion of frostbitten tissue in a large water bath of 40° to 42°C (104° to 108°F) D) Avoidance of dry heat E) Administration of toxoid booster as indicated by the immunization history

To demonstrate understanding of the key principles of the initial management of frostbite Answer A is correct The emergency department treatment of a frostbite victim should first focus on the basic ABCs of trauma resuscitation and on identifying and correcting systemic hypothermia. Frostbitten tissue should be immersed in a large water bath of 40° to 42°C (104° to 108°F). The bath should be large enough to prevent rapid loss of heat, and the water temperature should be maintained. This method of rapid rewarming may significantly decrease tissue necrosis caused by full-thickness frostbite. Dry heat is not advocated, because it is difficult to regulate and places the patient at risk for a burn injury. The rewarming process should take about 30 to 45 minutes for digits. The affected area appears flushed when rewarming is complete and good circulation has been reestablished. Narcotics are required because the rewarming process may be quite painful. The skin should be gently but meticulously cleansed and air-dried, and the affected area should be elevated to minimize edema. A tetanus toxoid booster should be administered as indicated by the immunization history. Sterile cotton should be placed between toes or fingers to prevent skin maceration, and extreme care should be taken to prevent infection and to avoid even the slightest abrasion. Prophylactic antibiotics and dermal blister débridement are both controversial; most clinicians débride blisters and reserve antibiotics for identified infections. A 2005 study reported a lower than expected digit amputation rate when rapid rewarming was followed by treatment with heparin and tissue plasminogen activator (tPA), but, to date, this finding has not been confirmed by other studies.

You have attained an MRI of the lower extremity from the patient in questions 1 and 2 that suggests that the major lower extremity vessels, nerves, and bones are spared by the tumor. Which of the following is the most appropriate treatment strategy? A) Complete resection to achieve a clear margin, followed by postoperative radiation B) Complete resection to achieve a clear margin without postoperative radiation C) Definite chemotherapy and radiation D) Amputation and adjuvant chemotherapy E) Amputation alone

To demonstrate understanding of the management of STS of the extremity Answer A is correct Surgery is the foundation of the treatment of STS. Amputation was once considered the only option for cure, but this view changed with the 1982 publication of the landmark trial carried out by Rosenberg and colleagues from the National Cancer Institute (NCI). In this trial, 43 patients with extremity sarcomas were randomly assigned to undergo either amputation of the affected limb (n = 16) or LSS plus radiation therapy (n = 27); all patients received adjuvant chemotherapy. The 5-year local recurrence rate was slightly higher in the LSS-radiation group, although the difference was not statistically significant. The 5-year survival was nearly identical in the two groups: 70% in the LSS-radiation group and 71% in the amputation group. In the years since the NCI trial, LSS has become the standard of care. Currently, fewer than 10% of all patients with surgically treated extremity sarcomas undergo amputation, and local recurrence rates after LSS are in the range of 5 to 15%. Amputation now is usually reserved for patients with extremity sarcomas that involve major vessels, nerves, or bones. Adequate surgical resection involves excising a margin of normal tissue around the tumor, along with any areas through which biopsies have previously been performed. Compartmental resection or resection of entire muscle groups provides no additional benefit over wide local excision. The ideal extent of a wide local excision has not yet been defined; however, it is usually considered that a margin of at least 1 to 2 cm, when possible, should be the goal. Regional lymphadenectomy is not usually indicated in the treatment of sarcomas, because only 2.6% of sarcomas metastasize to lymph nodes.

A 19-year-old male is in a high-speed motorcycle crash. On arrival at the ED, he is hypotensive, with flaccid paralysis of all four extremities and incontinence of urine and stool. Noncontrast cervical spine CT demonstrates a displaced fracture of C4 with retropulsion into the spinal canal. Which of the following statements is true regarding the management of spinal cord injury (SCI)? A) Craniocervical traction can be performed in highly unstable fractures with the use of fluoroscopy B) Specialists agree that steroids should be given in call cases of SCI C) Complete SCI is a contraindication to surgical intervention D) Surgical intervention does not have a role in the acute management of SCI E) Surgical intervention is limited to patients with otherwise nonreducible spine fractures

To demonstrate understanding of the management of acute spinal cord injury (SCI) Answer A is correct Traction should always be applied under strict neurologic monitoring. If the patient's condition deteriorates when the weight is increased, additional weight should be removed and the patient should immediately undergo imaging (e.g., with plain films, magnetic resonance imaging, or both). In the case of a highly unstable fracture, traction should be guided by fluoroscopy rather than serial x-rays. The challenge is to identify the most effective treatment or combination of treatments with the fewest severe side effects—a task requiring many experiments for each treatment tested. Methylprednisolone, which is thought to act by scavenging free radicals, has been reported to be neuroprotective in patients with SCIs. Considerable controversy remains, however, regarding the clinical benefit of methylprednisolone administration after acute SCI. There is general agreement among physicians that immobilization of the patient to prevent further injury and early stabilization of fractures and dislocations of the spine are necessary. The single widely accepted indication for early urgent surgical treatment is ongoing neurologic deterioration in the presence of spinal canal compromise from bone and disk fragments, hematoma, or unreduced subluxation. Surgical indications still under debate include incomplete SCI (with persistent spinal cord compression) and complete SCI with the possibility of some neurologic recovery.

You are the on-call surgery resident in the pediatric emergency department of a regional children's hospital. A 6-year-old boy is brought in as the victim of blunt trauma sustained during a motor vehicle collision. The child-has a closed head injury (CHI). When considering the diagnosis and treatment of pediatric CHI, which of the following actions is not indicated? A) Immediate intubation for a Glasgow Coma Scale (GCS) score of less than 12 B) Computed tomography (CT) of the head in any patient with a witnessed loss of consciousness, mental alteration, or an abnormal GCS score C) Judicious use of sedatives, anticonvulsants, pressors, intraventricular drainage, or even surgical decompression to optimize cerebral perfusion D) Monitoring of sodium levels E) Hospital admission for postconcussive symptoms (e.g., vomiting, seizures, or headache), even if the head CT reveals no structural injury

To demonstrate understanding of the management of closed head injuries (CHIs) in the pediatric population Answer A is correct CHIs are the most common cause of death in children and account for close to 7,000 fatalities in the United States each year. Any child who had a witnessed loss of consciousness, is mentally altered, or has an abnormal GCS score should undergo urgent CT of the head. If the GCS score is lower than 8, intubation is necessary to protect the airway. The main focus of therapy for severe CHI is on minimizing secondary insult to the brain by optimizing oxygen delivery. Management involves close monitoring of cerebral perfusion pressure and judicious use of sedatives, anticonvulsants, pressors, intraventricular drainage, or even surgical decompression to optimize cerebral perfusion. Evidence-based practice guidelines for the management of pediatric CHI patients are now available. Because children with even minor structural injuries to the brain are at risk for hyponatremia, close monitoring of sodium levels is required. Patients who have postconcussive symptoms (e.g., vomiting, seizures, or headache) should be admitted, even if the head CT reveals no structural injury.

A 29-year-old professional pianist is brought to the emergency department after suffering a rollover motor vehicle accident while driving at high speed. He is alert, oriented, and hemodynamically stable. On examination, his only injury is a 4 cm longitudinal laceration to his distal left forearm over the radial artery. Blood spurts from the wound when direct pressure is not held, and the distal radial pulse at the wrist is not palpable. Motor and sensory examinations of the left hand are normal. What is the appropriate management of this patient? A) Apply a pressure dressing and perform serial examinations B) Explore the wound in the OR and ligate the injured radial artery C) Explore the wound in the OR and perform a primary repair of the radial artery D) Explore the wound in the OR, perform a primary repair of the radial artery, and place the patient on a heparin drip E) Explore the wound in the OR and perform a repair of the artery using a vein patch angioplasty

To demonstrate understanding of the management of distal extremity arterial injuries Answer B is correct. On occasion, arteries in the distal extremities (e.g., the radial, ulnar, or tibial arteries) may have to be repaired or ligated after trauma. In most patients, there is little need for repair of these arteries, which can typically be ligated without deleterious effects. The safety of ligation is predicated on the presence of adequate arterial flow from the nonaffected arteries, as well as retrograde blood flow from an intact palmar or plantar arch. Repair of injuries to these arteries is associated with the possibility of embolization or other surgical problems. In addition, the patency rate for grafts in the distal extremities tends to be low.

A 24-year-old male is brought to the ED after a single gunshot wound to his left lower abdomen. He is severely hypotensive with a distended abdomen and a pulseless left lower extremity. On exploration in the OR, an injury to the left common iliac artery and vein is identified. During the case, the patient receives one round of advanced cardiac life support after developing pulseless electrical activity; he regains a carotid pulse, and with massive resuscitation, his blood pressure has stabilized. What is the most appropriate management of this patient? A) Ligate both the left common iliac artery and vein and resuscitate in the ICU B) Ligate the left common iliac vein, place a Pruitt shunt in the artery, resuscitate in the ICU, and perform definitive operation at a later date C) Perform primary repair of the vein and then subsequent primary repair of the artery D) Perform primary repair of the vein and then subsequent interposition PTFE graft repair of the artery E) Perform primary repair of the vein and then subsequent repair of the artery using the ipsilateral internal iliac artery

To demonstrate understanding of the management of injury to the iliac artery and vein Answer B is correct. Ligation of either the common or the external iliac artery in a hypotensive trauma patient leads to a greater than 50% amputation rate in the postoperative period; consequently, injuries to these vessels should be repaired if at all possible. The standard options for repair—lateral arteriorrhaphy, completion of a partial transection with an end-to-end anastomosis, and resection of the injured area with insertion of a conduit—are feasible in most situations. On rare occasions, it may be preferable either to mobilize the ipsilateral internal iliac artery to serve as a replacement for the external iliac artery or to transpose one iliac artery to the side of the contralateral iliac artery. When a patient is in severe shock from exsanguination caused by a complex injury to the common or the external iliac artery, damage-control laparotomy is indicated. The injured area should be resected and a temporary intraluminal Argyle, Javid, or Pruitt-Inahara shunt inserted to maintain flow to the ipsilateral lower extremity during resuscitation in the surgical ICU. One unique problem associated with repair of the common or the external iliac artery is the choice of technique when significant enteric or fecal contamination is present in the pelvis. In such cases, there is a substantial risk of postoperative pelvic cellulitis, a pelvic abscess, or both, which may lead to blowout of any type of repair. When extensive contamination is present, it is appropriate to divide the common or external iliac artery above the level of injury, close the injury with a double row of continuous 4-0 or 5-0 polypropylene sutures, and bury the stump underneath uninjured retroperitoneum. If a stable patient has obvious ischemia of the ipsilateral lower extremity at the completion of this proximal ligation, one may perform an extra-anatomic femorofemoral crossover bypass with an 8 mm externally supported PTFE graft to restore arterial flow to the extremity. If the patient is unstable, one should take several minutes to perform an ipsilateral four-compartment below-the-knee fasciotomy; this step will counteract the ischemic edema that inevitably leads to a compartment syndrome and compromises the early survival of the leg. After adequate resuscitation in the surgical ICU, the patient should be returned to the OR for the femorofemoral graft within 4 to 6 hours. Injuries to the internal iliac arteries are usually ligated even if they occur bilaterally because young trauma patients typically have extensive collateral flow through the pelvis. The survival rate in patients with isolated injuries to the external iliac artery in older series exceeded 80% when tamponade was present. If the injury was large and free bleeding had occurred preoperatively, however, the survival rate was only 45%. Current studies report overall survival rates of approximately 45 to 55% for injuries to the common iliac artery and 62 to 65% for injuries to the external iliac artery. Injuries to the common or the external iliac vein are best treated by a lateral venorrhaphy with continuous 4-0 or 5-0 polypropylene sutures. Significant narrowing often results, and a number of reports have demonstrated occlusion on postoperative venography. For patients with narrowing or occlusion, as well as for those in whom ligation was necessary to control exsanguinating hemorrhage, the use of elastic compression wraps and elevation for the first 5 to 7 days after operation is mandatory.

A 20-year-old man is brought to the ED with a single gunshot wound to his central abdomen between the xiphoid and the umbilicus. Examination demonstrates a distended and tender abdomen. He is taken to the OR for exploration while being resuscitated with blood and plasma. On exploration, hemoperitoneum is encountered and a zone I retroperitoneal hematoma is noted. Medial visceral rotation is performed, and an injury to the pancreas and superior mesenteric artery (SMA) is identified. Which of the following is the best management of the SMA injury in a relatively stable patient with this combination of injuries? A) Ligate the artery B) Perform primary repair of the artery C) Use a great saphenous vein (GSV) patch to repair the artery D) Ligate the artery at its origin and use a polytetrafluoroethylene (PTFE) graft or GSV patch to reconstruct the SMA takeoff from the distal abdominal aorta away from the site of injury E) Use a PTFE interposition graft to reconstruct the artery

To demonstrate understanding of the management of injury to the superior mesenteric artery (SMA) Answer D is correct Injuries to the SMA are managed according to the anatomic level of the perforation or thrombosis. On rare occasions, in patients with injuries beneath the neck of the pancreas, one may have to transect the pancreas to obtain proximal control. Another option is to perform left medial visceral rotation and apply a clamp directly to the origin of the SMA. Injuries to the SMA in this area or just beyond the base of the mesocolon are often associated with injuries to the pancreas. The potential for a postoperative leak from the injured pancreas near the arterial repair has led numerous authors to suggest that any extensive injury to the artery at this location should be ligated. Because of the intense vasoconstriction of the distal SMA in patients who have sustained exsanguinating hemorrhage from more proximal injuries treated with ligation, the collateral flow from the foregut and hindgut is often inadequate to maintain the viability of the organs in the distal midgut, especially the cecum and the ascending colon. Therefore, it is safest to place a saphenous vein or PTFE graft on the distal infrarenal aorta, away from the pancreatic injury and any other upper abdominal injuries. Such a graft can be tailored to reach the side or the anterior aspect of the SMA, or it can be attached to the transected distal SMA in an end-to-end fashion without significant tension. Soft tissue must be approximated over the aortic suture line of the graft to prevent the development of an aortoenteric fistula in the postoperative period. In patients with severe shock from exsanguination caused by a complex injury to the SMA, damage-control laparotomy is indicated: the injured area should be resected and a temporary intraluminal Argyle, Javid, or Pruitt-Inahara shunt inserted to maintain flow to the midgut during resuscitation in the surgical intensive care unit (ICU). When ligation is indicated for more distal injuries to the SMA, segments of the ileum or even the right colon may have to be resected because of ischemia. The survival rate in patients with penetrating injuries to the SMA is approximately 55 to 60% overall but only 20 to 25% when any form of repair more complex than lateral arteriorrhaphy is necessary.

While discussing her treatment plan, the patient from questions 1 to 3 begins to ask you about her prognosis and the rationale for your treatment plan. Which of the following is not true? A) Tumor thickness correlates with overall survival B) Clinical risk factors that have been shown to have significant prognostic value include age, sex, location, number of involved lymph nodes, presence of metastases, and serum lactate dehydrogenase (LDH) C) Of patients with positive lymph nodes, those with macroscopic disease (i.e., those with palpable lymphadenopathy that is confirmed histologically) have a survival similar to that of those with microscopic lymph node involvement D) A 0.5 cm margin is adequate for in situ melanoma, a 1 cm margin is adequate for melanomas less than 1 mm thick, and 2 cm margins should be obtained for lesions that are greater than 1 mm in depth E) SLNB should be considered in the management of this patient

To demonstrate understanding of the prognostic factors and management of early-stage melanoma Answer C is correct Extensive research has been done over the past several years regarding prognostic factors in both early- and late-stage melanoma. Overall, clinical factors shown to have significant prognostic value include patient age, sex, location of the melanoma, number of involved lymph nodes, presence of distant metastasis, and serum LDH level. Prognosis is generally better for patients under 65 years of age, women, and those with tumor location on the extremities, absence of lymph node involvement, absence of distant metastasis, or a normal serum LDH level. See also the explanation for question 3.

The patient from questions 1 and 2 returns to the office to discuss treatment options. On detailed physical examination, no other suspicious lesions are found, and no palpable lymphadenopathy is present. An appropriate treatment plan includes: A) Surgical excision with 1 cm margins and no SLNB B) Surgical excision with 1 cm margins and SLNB C) Surgical excision with 2 cm margins and no SLNB D) Surgical excision with 2 cm margins and SLNB E) Surgical excision with 4 cm margins and SLNB

To demonstrate understanding of the surgical treatment of melanoma Answer D is correct Surgical excision remains the mainstay of treatment for melanoma. The width of the recommended surgical margins depends on the thickness of the lesion and has been well defined by a series of prospective randomized clinical trials. The most recent recommendations suggest that a 0.5 cm margin is adequate for in situ melanoma, whereas margins of 1 cm are suggested for melanomas less than 1.0 mm in thickness. Two-centimeter margins should be obtained for lesions measuring 1 to 2 mm in depth. Although a reduction in surgical margin width to 1 cm for melanomas 1 to 2 mm in thickness may slightly increase the risk for local recurrence, it would not reduce overall survival statistics. Two-centimeter margins should be obtained for melanomas greater than 2 mm in thickness. Sentinel lymph node status is the single most important predictor of survival in patients with melanoma and is considered a standard approach in this country. SLNB is recommended for intermediate thickness (1 to 4 mm) and thick (> 4 mm) melanomas. It is generally not recommended for thin melanomas (< 1 mm) unless the lesion is of high risk (presence of ulceration, extensive regression, high mitotic rate, Clark level IV, or positive deep margin) or depending on the patient's age (SLNB may be performed in young patients).

A 17-year-old was the unhelmeted driver of an all-terrain vehicle that was involved in a rollover accident at high speed. Evaluation revealed an epidural hematoma on the left with midline shift. An intraventricular drain was placed to monitor ICP. Which of the following statements regarding the tiered approach to the management of elevated ICP is false? A) Decompressive craniectomy should be instituted prior to barbiturate coma B) Repeat noncontrast head CT scan should be obtained prior to elevating therapy from one tier to the next C) Mannitol and hypertonic saline infusion can be repeated as needed independent of the specific tier D) Neuromuscular paralysis compromises the entire neurologic examination E) A neurosurgeon or neurointensivist should see the patient prior to elevating treatement

To demonstrate understanding of the tiered approach to managing elevated intracranial pressure (ICP) Answer D is correct Moderate hypothermia and barbiturate metabolic coma are highly effective means of reducing ICP but are associated with significant morbidity. As a result, many centers are considering surgical intervention prior to inducing barbiturate coma. Therapies within a tier should be maximized before moving to a higher tier, and careful consideration should be given to repeat head CT scans when a patient requires escalation of therapy (to evaluate for intracranial lesion progression). Mannitol and hypertonic saline can be delivered in boluses for acute control of ICP, but in adults, they are not typically used as a regular, intermittent therapy. They can be used across all tiers of therapy but are limited by their side effects of induced serum hyperosmolarity and induced hypernatremia. Although muscular paralysis compromises most of the neurologic examination, it is important to remember that neuromuscular blockade does not interfere with pupillary response, which is mediated by smooth muscle contraction.

A 21-year-old man is the nonhelmeted driver of a motorcycle who swerved off the road in an attempt to avoid hitting another vehicle. The GCS score at the scene is 7, so he is intubated and brought to the ED, where a head computed tomographic (CT) scan demonstrates a 1 cm left-sided subdural hematoma and 3 mm of midline shift. Neurosurgery evaluation is performed, and nonoperative intervention is recommended; an intraventricular catheter is placed to monitor ICP. Despite addressing all basic measures, the ICP is elevated at 26 mm Hg. Repeat noncontrast head CT is stable from prior images. Which management is most appropriate? A) Continue mannitol and hypertonic saline despite a serum osmolarity of 332 mOsm/L B) Moderate hyperventilation to PaCO2 of 30 mm Hg C) Decompressive craniotomy D) Fentanyl and propofol sedation to reduce cerebral metabolic demand E) Induce phenobarbital coma

To demonstrate understanding of the tiered approach to managing elevated intracranial pressure (ICP) Answer D is correct Patients who experience elevated ICP despite the above prophylactic measures should be managed by a protocol-directed, tiered approach. Therapies within a tier should be maximized before moving to a higher tier, and careful consideration should be given to repeat head CT when a patient requires escalation of therapy (to evaluate for intracranial lesion progression). It is generally not advisable for escalation of tiered therapy to occur without careful assessment by the treating neurosurgeon or neurointensivist as some etiologies of elevated ICP may result from reversible events (e.g., hypoventilation) that require more specific treatment strategies. Mannitol and hypertonic saline can be delivered in boluses for acute control of ICP, but in adults, it is not typically used as a regular, intermittent therapy. They can be used across all tiers of therapy but are limited by their side effects of induced serum hyperosmolarity and induced hypernatremia. Mannitol is usually administered in intravenous boluses of 0.25 to 1 g/kg over 10 to 15 minutes, until either ICP is controlled or serum osmolarity reaches 320 mOsm/L. Many neuroscience intensive care units prefer the use of fentanyl and propofol for ICP control as they are relatively shorter-acting agents that have demonstrated success in the management of severe TBI. Propofol is a sedative hypnotic with a rapid onset and a short duration of action. In patients requiring more than 48 hours of sedation, propofol infusion results in significantly fewer ventilator days than treatment with intermittent lorazepam and has a more favorable economic profile and shorter weaning time than midazolam. In moderate and severe TBI patients, a prospective trial found significantly improved ICP control with propofol infusion (versus morphine). After maximizing sedation and supporting CPP, many clinicians will institute muscular paralysis for ICP control. Although its efficacy may vary among patients, pharmacologic paralysis can be used in conjunction with maximal sedation to further reduce ICP. Paralysis is also achieved using continuous intravenous infusion and should be titrated to effect (using percutaneous peripheral nerve stimulation). Although muscular paralysis compromises most of the neurologic examination, it is important to remember that neuromuscular blockade does not interfere with pupillary response, which is mediated by smooth muscle contraction. Hyperventilation can be used as a strategy for ICP lowering despite maximal sedation and chemical paralysis but carries the risk of iatrogenic cerebral ischemia. Therefore, mild to moderate hyperventilation should be used only in patients with a continuous brain oxygenation monitor, such as a brain tissue oxygen probe or jugular bulb oximetry monitor. Decompressive craniectomy is an important component of protocol-driven therapy for patients with severe TBI and should be performed in patients who have exhausted tier I and II therapies and do not have adequate ICP control with intermittent hyperosmolar therapies. Moderate hypothermia and barbiturate metabolic coma are highly effective means of reducing ICP but are associated with significant morbidity. As a result, many centers are considering surgical intervention prior to inducing barbiturate coma.

A 51-year-old female was referred to you with a large mass of the stomach that is a biopsy-proven gastrointestinal stromal tumor (GIST). She has been told of a "targeted" therapy option using a drug called imatinib. What is the molecular target of this drug? A) c-myc B) ret C) Fas ligand D) Epidermal growth factor receptor E) c-kit

To demonstrate understanding of the treatment of GISTs Answer E is correct The characterization and targeting of the tyrosine kinase receptor c-kit has generated a new approach to treating GISTs. Phase III trials comparing standard and high-dose imatinib mesylate in the treatment of these lesions have been completed in the United States and Europe. In these two trials, the progression-free response rate ranged from 43 to 53%, and the estimated 2-year survival ranged from 69 to 78%. The use of imatinib mesylate in adjuvant and neoadjuvant settings is still evolving, but recent data suggest a survival advantage of using imatinib in a neoadjuvant approach.

In discussing the treatment plan, the patient in questions 1 to 3 asks you about other options for the treatment of STS. Which of the following statements is not true? A) Preoperative chemotherapy may reasonably be considered in attempting to preserve limb function, but given the lack of sufficient evidence, its use should be limited to clinical trials B) The use of postoperative chemotherapy is clearly associated with a survival benefit C) Hyperthermic isolated limb perfusion may allow patients who are not candidates for LSS to undergo surgery with a negative margin without amputation D) Patients with STSs with recurrence after resection may benefit from further resection. E) For sarcoma metastases to the lungs, patients with previously resected extremity STS and control of the primary tumor may benefit from pulmonary metastasectomy

To demonstrate understanding of the various treatment strategies for STS Answer B is correct Postoperative chemotherapy for the treatment of STS has been studied in numerous prospective, randomized trials, but the small sample sizes and the various differences among the trials have made it difficult to determine how efficacious such therapy is. At present, given the uncertainty regarding its efficacy, postoperative adjuvant chemotherapy for treatment of STS is probably best employed in the context of appropriate clinical trials. Similarly, given the lack of sufficient evidence for any survival benefit, preoperative chemotherapy may reasonably be considered in attempting to preserve limb function, but otherwise, its use should be limited to clinical trials. The largest trials of isolated limb perfusion for extremity sarcoma to date were conducted in the Netherlands by Eggermont and colleagues, who used tumor necrosis factor (TNF) and melphalan. A total of 246 patients with extremity sarcomas who were not candidates for LSS underwent one or two sessions of isolated limb perfusion. After an interval of 2 to 4 months, 76% of these patients were able to undergo a resection with a negative margin, and 71% underwent successful LSS. Patients with extremity STSs experience local recurrences at a rate of 8 to 20% despite appropriate primary resection, and those with resected retroperitoneal sarcomas experience local recurrences at a rate of 38 to 50%. These patients are often candidates for re-resection of the locally recurring sarcoma. A retrospective review from the Brigham and Women's Hospital found that 67% of patients with recurrent STSs were able to undergo salvage surgery with excellent long-term survival. Pulmonary metastases are present in approximately 20% of patients with trunk or extremity sarcomas. If the patient is medically fit, the primary tumor is controlled, no extrathoracic disease is present, and the metastases are resectable, pulmonary metastasectomy may be attempted.

A 19-year-old female involved in a motor vehicle collision is found to have a left tibial fracture and soft tissue injury requiring operative repair. In the postanesthesia care unit, she complains of pain that is unable to be relieved with narcotic analgesics. On palpation, she has a tense and swollen left leg. Passive flexion and extension of her foot increase her pain. Her dorsalis pedis and posterior tibial pulses are intact. She is subsequently taken back to the operating room for a decompressive fasciotomy. In a patient who is not awake or alert enough to complain of symptoms, what is the most accurate method of diagnosing compartment syndrome? A) Measurement of a compartment pressure of 10 mm Hg in the affected extremity B) Motor nerve stimulation to evaluate for muscle paralysis in the affected extremity C) Handheld Doppler ultrasonography to evaluate for pulselessness in the affected extremity D) Measurement of a compartment pressure of 15 mm Hg in the affected extremity E) A difference of less than 30 mm Hg between arterial diastolic pressure and compartment pressure in the affected extremity

To identify the signs, symptoms, and diagnostic criteria of extremity compartment syndrome Answer D is correct Compartment syndrome is defined as high-pressure swelling within a fascial compartment. Many physicians still believe, incorrectly, that compartment syndrome cannot develop in conjunction with an open fracture, because the break in the skin provides decompression. This is a dangerous assumption: compartment syndrome occurs in a significant number of patients with open fractures—for example, as many as 10% of patients with open tibial fractures. The most common cause of compartment syndrome is hemorrhage and edema in the damaged soft tissues seen with fractures. Other causes include a too-tight dressing or cast, disruption of the limb's venous drainage, advanced ischemia, and eschar from a circumferential burn. Multiply injured patients with hypovolemia and hypoxia are predisposed to compartment syndrome. The key to diagnosis of compartment syndrome is to maintain a high level of suspicion in any situation involving an extremity injury where there is a significant chance that this syndrome might develop (e.g., tibial fractures, forearm fractures, and all comminuted fractures associated with severe soft tissue injury). The diagnosis is primarily a clinical one, with the five Ps—pain, pallor, paresthesia, paralysis, and pulselessness—constituting the classic signs. The surgeon should not wait until all of these signs are present; the prognosis is much better if they are not. Severe ischemic muscle pain occurs that is unrelieved by the expected amounts of analgesia. On palpation, the compartment is tense and swollen, and passive stretching of the digits of the extremity increases the pain. Paresthesia occurs early and should be actively watched for; paralysis develops when ischemia has caused permanent damage. Pulselessness occurs late and is a relatively rare sign; it has been shown that irreversible damage can occur in a patient who still has palpable pulses. Measurement of compartment pressures is also employed in the diagnosis of compartment syndrome. Monitoring can be particularly helpful in patients who are not alert or are difficult to examine. There is no agreement on what constitutes the critical pressure threshold for a definitive diagnosis. An absolute value of 30 to 35 mm Hg has frequently been adopted as a diagnostic indicator; however, the evidence suggests that the difference between the diastolic arterial pressure and the pressure in the involved compartment (delta pressure, or Δp) is more important than any particular absolute value. Currently, a diagnosis of compartment syndrome is usually made if the Δp is less than 30 mm Hg, depending on the clinical signs and the level of suspicion. If compartment syndrome is suspected, the first step is to remove all circumferential bandages to relieve any pressure. If a plaster cast is present, it should be split, spread, or removed; if necessary, maintenance of reduction should be sacrificed. If the clinical picture does not improve after these measures are taken, then a decompressive fasciotomy is indicated.

A 52-year-old man was involved in a motorcycle accident and suffered a femur fracture. In patients with long bone fractures of the lower extremity, what measure has been shown to decrease the incidence of developing fat embolism syndrome? A) Volume replacement B) Administration of corticosteroids C) Early stabilization of the fracture D) Administration of appropriate analgesics E) Respiratory support

To understand that the treatment of fat embolism is supportive and that it is best prevented by early fracture fixation Answer C is correct Fat embolism syndrome (FES) is most commonly associated with fractures of long bones of the lower extremity. The classic clinical triad consists of respiratory distress, cerebral dysfunction, and petechial rash. The pathophysiology is not clear, but there is some evidence to suggest that extravasation of fat particles from long bone fractures may play an important part. Furthermore, early stabilization of long bone fractures has been shown to decrease the incidence of FES. Signs and symptoms of clinical FES usually begin within 24 to 48 hours after trauma. Treatment is primarily prophylactic and supportive, consisting of early fracture fixation, careful volume replacement, analgesia, and respiratory support. The role of corticosteroids in this setting is controversial.

A 27-year-old helmeted male fell from his motorcycle while traveling on a roadway partially covered with sand and gravel. He did not lose consciousness and was awake at the scene complaining of pain in his right hip. He was taken to the emergency department, where he underwent a trauma evaluation. He was speaking and had bilateral breath sounds and a strong carotid pulse. His heart rate was in the 90s, his blood pressure was 143/87 mm Hg, and he had a normal oxygen saturation. During the evaluation, gentle rocking of his pelvis evoked tenderness. A sheet wrap was placed. He then underwent chest and cervical films that did not demonstrate an injury. An anteroposterior pelvis film demonstrated an injury pattern consistent with an anteroposterior compression injury. A contrast-enhanced computed tomographic (CT) scan of the abdomen and pelvis showed the aforementioned pelvic fracture as well as extravasation of contrast from a branch of the right internal iliac artery. What is the most appropriate next step in the management of this patient? A) Immediate transfer to the intensive care unit for further resuscitation B) Immediate transfer to the operating room for an exploratory laparotomy C) Immediate transfer to the operating room for open reduction and internal fixation of the pelvic fracture D) Immediate transfer to the interventional radiology suite for angiographic embolization E) Perform the FAST examination (focused assessment for sonographic evaluation of the trauma patient)

To understand the management algorithm for patients with unstable pelvic fracture Answer D is correct Pelvic fractures are frequently associated with significant hemorrhage, not only because of the fracture itself but also because pelvic trauma is often accompanied by serious injuries to other parts of the body (e.g., the chest or the abdomen). Given that bleeding in pelvic fracture patients can occur in other body compartments besides the pelvis and can be arterial as well as venous, it is of the utmost importance to identify its source and, ideally, determine its nature as soon as possible. This information is crucial in determining what the next steps in management should be. The first step after diagnosing a pelvic fracture should be the immediate application of some type of external stabilization device (e.g., a sheet wrap or a device such as the Pelvic Binder [Pelvic Binder Inc., Dallas, TX]). The rationale behind this step is that approximating the fractured bones and thereby decreasing the volume of the pelvis may reduce blood loss, particularly from the fractured bones and the lacerated venous plexus. In addition, stabilization may minimize further damage to blood vessels and prevent dislodgment of recent clots. It is doubtful, however, whether this procedure actually reduces arterial hemorrhage to a significant degree. In hemodynamically unstable patients with clinical signs of a pelvic fracture, the next step (immediately after—or, preferably, while—x-rays of the chest, the pelvis, and the cervical spine are obtained according to advanced trauma life support [ATLS] protocols) should be the FAST (focused assessment for sonographic evaluation of the trauma patient) to rule out a significant intra-abdominal bleeding source. If the FAST is negative and no other obvious sources of hemorrhage (e.g., chest or extremities) are found, the pelvis is the most likely source of the bleeding. The question then arises as to whether the pelvic hemorrhage is predominantly arterial or venous. An arterial bleeding source in the pelvis is found in 73% of hypotensive patients who do not respond to initial fluid resuscitation. Contrast-enhanced CT is extremely helpful in determining the presence of arterial hemorrhage in cases of pelvic fracture but can be performed only if the patient is stable enough to undergo the time-consuming transfer to the imaging suite. Extravasation of contrast medium, a large retroperitoneal hematoma, or abrupt cutoff of an artery on CT indicates that angiographic embolization is necessary. Contrast extravasation (so-called contrast blush) is a particularly good predictor of arterial hemorrhage necessitating embolization, having a sensitivity and specificity of well over 80%. Thus, CT is an ideal means of identifying patients who should be treated with angiographic embolization and, ideally, should be performed in all pelvic fracture patients who are stable enough to undergo this procedure. Arterial hemorrhage should preferably be treated by angiographic embolization, which has shown excellent results in current studies. Success rates exceed 90%, and major complication rates are below 5%. This technique does, however, require a skilled, experienced, and permanently available interventional radiology service. In patients with unstable fractures, venous hemorrhage is treated with operative placement of an external fixation device. This measure requires specific expertise on the part of the trauma surgeon; in experienced hands, it should take no longer than 20 minutes to perform. Patients with more severe pelvic fractures probably benefit most from this procedure. Retroperitoneal packing may be employed as an adjunct to external fixator placement. It is unlikely that external fixation has a significant impact on arterial hemorrhage; consequently, it is vital to decide whether angiographic embolization should precede placement of an external fixation device in the operating room.

The pathology from the patient in question 1 returns high-grade STS, malignant fibrous histiocytoma subtype. Which of the following regarding STSs is not true? A) Sarcomas may occur anywhere in the body and comprise more than 50 distinct histologic subtypes B) The majority of STSs occur in the retroperitoneum C) Genetic factors, irradiation, chemical exposure, and lymphedema have all been shown to correlate with evolution of STSs D) The best indicator of a tumor's biologic aggressiveness and metastatic potential is its grade, regardless of its histologic subtype E) The American Joint Committee on Cancer staging system for STS integrates tumor grade, size, depth of tissue invasion, degree of nodal involvement, and presence or absence of metastases

To understand the pathologic classification and staging of STS Answer B is correct STS is a collective term for an unusual and diverse group of malignancies that arise from cells of the embryonic mesoderm. Although tissues derived from the mesoderm contain approximately 75% of the cells in the human body, sarcomas represent only 1% of adult tumors and 15% of pediatric tumors. Sarcomas may occur anywhere in the body and comprise more than 50 distinct histologic subtypes. Approximately 43% of STSs occur in the extremities, 15% in the retroperitoneum, 10% in the trunk, 19% in the viscera, and 13% in other locations. In addition, some sarcomas occur in the gastrointestinal tract. It has been estimated that in 2007, there will be approximately 9,220 cases of sarcoma in the United States, with 3,560 deaths. STS-related mortality has been quite constant over the years, indicating that relatively little progress has been made in the treatment of most sarcomas. The etiology of STS is unclear and somewhat controversial. Genetic factors, irradiation, chemical exposure, and lymphedema have all been shown to have a strong correlation with the evolution of these lesions. Regarding its staging, answers 4 and 5 are true.


Conjuntos de estudio relacionados

Chap 1: Collecting and Analyzing Data

View Set

Sample Questions Examination 5. Chapters 23-24

View Set

Lifespan Growth & Development - Unit 5 Middle Childhood

View Set

Psych Statistics - Module 2 Test

View Set